Philosophy <- StackExchange top 100

1: What is the difference between Fact and Truth? (score 201298 in 2013)

Question

I’m curious about the difference between Fact and Truth. I was searching on the internet if I could find it. But still I’m confused about the exact meaning.

I first read the forum discussion here Fact and Truth where an author has given two examples for each like below

A fact is a reality that cannot be logically disputed or rejected. If I say “fire is hot,” I don’t care how great your reasoning skills are, if you touch fire your skin will burn (and don’t give me that “but people can walk on hot coals!” bull. There’s a difference between the transfer of heat through conduction and training one’s body to deal with the agonizing pain of said conduction). Now when I say this, I am not speaking a truth, I am speaking a fact. If you say “fire is not hot,” you are not lying, you are incorrect. Facts are concrete realities that no amount of reasoning will change. When one acknowledges a fact, they are doing just that. Facts are not discovered, facts are not created, facts are simply acknowledged.

A truth on the other hand, is almost the opposite. Truths are those things that are not simply acknowledged, but must be discovered, or created. If I say “God exists,” and I possess strong reasoning for the affirmative of that statement, then God really does exist, that is a reality. However, if another individual possesses strong reasoning for the negative, and because of this reasoning they believe that God does not exist, then that is also a reality. If we were to debate our ideologies, and my reasoning appeared stronger than theirs, they may choose to adopt my belief that God does exist. If they do, then the existence of God is just as true as the nonexistence of God which they believed a week ago. Truths, as opposed to fact, are much more fluid and malleable than their empirical counterparts.

and followed by further discussion.

Then I found this Reference.
Article from above link says like below:

Facts are notes and lyrics on sheet music. Truth is what the singer gives to the listener when she’s brave enough to open up and sing from her heart.

But still curious about the difference between both of them.

In our daily life, in general conversation, we generally use these both terms interchangeably. Then what is the difference? Are they synonym or have specific difference?

Answer accepted (score 19)

The quote about facts gets it pretty right. A fact is, for many philosophers, a part of reality (Russel, for example). So as there are people and tables and chairs in our world, there is also the fact that I am sitting on the chair. It is as real as the chair itself. You often see some kind of brackets when someone speaks about fact, so for example: < I am sitting on a chair> converts to “The fact that I am sitting on a chair”.

Truth is a property of sentences, propositions, utterances, whatever you like. Facts can therefore not be true, in the same way as a chair cannot be true. Stating a fact, however, and depending on your opinion, has a truthvalue.

I think the second quote about truth is a bit problematic. It sounds as if good arguments alter reality. But arguments cannot be true, they can be valid, and they can be truthconserving. So if I have an argument for the existence of god, it is at best valid. That does not mean, however, that suddenly, in virtue of the good argument, god came into existence.


Edit: More on truth

So on one common view those things that can be true are propositions. So a meaningful exression would be: The proposition that snow is white is true.

If you believe that sentences are the things that can be true, then this would be an example: The sentence “Grass is green” is true.

Most people believe that facts cannot be true: They think that “(The fact that grass is green) is true” is a weird thing to say. (I use brackets to make clear that the predicate “is true” refers to the fact. Because otherwise there could be a second reading about the (fact that grass is green is true), if there is such a fact)

To conclude:(i) There is the fact that grass is green, and (ii) the proposition that grass is green is true.

Also it is worth pointing out that there are philosophers who say that there are no facts, because facts are weird ontological things and maybe you can do without them. So this is just one way to answer this question.

Answer 2 (score 1)

I want to make some general points about the OP.

Firstly, you appear to be asking for how the words truth and fact are used, but you capitalize these words. That already tends to obfuscate the issue, suggesting there is some very special, possibly metaphysical, usage you are alluding to.

Secondly, in asking for the meaning of individual words, you are suggesting that the unit of meaning is a single word. This is not true, as any cursory look in a dictionary will demonstrate. There are multiple entries for both truth and fact, not in the the least because the meaning of the words is modified by their context, and that therefore truth and fact can have multiple meanings in different contexts.

Now, it happens to be the case that one such dictionary entry for truth is “conforming to the facts” and for fact “a particular truth known”. This is from the Oxford Dictionary, but I assume any dictionary would have similar definitions. This only goes to show that in one important sense truth and fact are interchangeable.

Answer 3 (score 1)

I want to make some general points about the OP.

Firstly, you appear to be asking for how the words truth and fact are used, but you capitalize these words. That already tends to obfuscate the issue, suggesting there is some very special, possibly metaphysical, usage you are alluding to.

Secondly, in asking for the meaning of individual words, you are suggesting that the unit of meaning is a single word. This is not true, as any cursory look in a dictionary will demonstrate. There are multiple entries for both truth and fact, not in the the least because the meaning of the words is modified by their context, and that therefore truth and fact can have multiple meanings in different contexts.

Now, it happens to be the case that one such dictionary entry for truth is “conforming to the facts” and for fact “a particular truth known”. This is from the Oxford Dictionary, but I assume any dictionary would have similar definitions. This only goes to show that in one important sense truth and fact are interchangeable.

2: What is the difference between Rule Utilitarianism and Act Utilitarianism? (score 200486 in 2015)

Question

Based on the definitions given by Wikipedia , Rule Utilitarianism and Act Utilitarianism both seems to imply the same meaning

Rule Utilitarianism

Action is right as it conforms to a rule that leads to the greatest good, or that “the rightness or wrongness of a particular action is a function of the correctness of the rule of which it is an instance.”

Act Utilitarianism

Person’s act is morally right if and only if it produces at least as much happiness as any other act that the person could perform at that time

The supposed difference between Rule Utilitarianism and Act Utilitarianism

For rule utilitarians, the correctness of a rule is determined by the amount of good it brings about when followed. In contrast, act utilitarians judge an act in terms of the consequences of that act alone

The justification of action for the above rule mentioned are about the greater good of the society , How are Act Utilitarianism and Rule Utilitarianism different from each other ???

Answer accepted (score 15)

A rule utilitarian thinks, before acting, about the consequences of people following that rule. If the outcome is regarded as positive, she might decide that it’s good to follow that rule in general, and will apply it in future.

An act utilitarian doesn’t generalise the act, but regards it as a single action with a single outcome. She will have to weigh the possible consequences each and every time she acts.

Therefore, rule utilitarianism is considered to be more practicable, countering the anti-utilitarian argument that weighing each and every possible outcome each and every time is just not the way we want to (or can) spend our time.

On the other hand, act utilitarians consider rule utilitarians somewhat dull-witted, for a smart person might think of herself to be able to decide what to do without just applying rules time and time again. Also, blindly applying rules to specific situations can have unforeseen negative consequences that might have been averted by somebody who paid more attention instead of executing a programme.

At the same time, act utilitarians are criticised for their double standards, for they think it is useful if everybody follows “good” rules while they take for themselves the right to decide whether or not it is clever to stick to those rules in a specific situation.

An example: A rule utilitarian drives at night and sees a red intersection light. Thinking “it would have good consequences if people would stick to the rule and not cross red lights, so everyone is safe while waiting for a short while”, she would apply that rule to herself and wait for it to turn green. Meanwhile, the act utilitarian might think “well, I certainly hope that people, who aren’t me, in general follow that rule and stay put, but as there’s no one around who might get influenced by my act, since there’s no police around to fine me, and since I would see an approaching car as it’s dark, I might as well cross right now.”

Sources: There is a paper by Smart which you can find here; I’m pretty sure that’s what we read in the seminar where I learned what I wrote. Smart’s the act utilitarian.

Answer 2 (score 6)

Originally, I was going to comment on iphigenie’s answer, but I decided there were enough parts I wanted to comment on that it’s worth supplying a second answer – even though the other one is good as it is.

John Stuart Mill’s Utilitarianism seeks to maximize happiness (something he inherits from Jeremy Bentham and his father). Contemporary versions are usually a bit more nuanced in what they are trying to do and are probably better called consequentialism. There are several flaws in Mill’s original idea for utilitarianism which center on two pairs of epistemic concerns.

First, there’s a question about whether we are to do what we expect would maximize happiness or whether we are to in fact maximize happiness. If we are expected to really maximize happiness, we face an impossible task, because we can get it completely wrong (maybe the cake I baked for your birthday contains an allergen that kills you and depresseds thousands of people). But then if we are just maximizing what we expect will make people happy, then we need to have some rules in place to balance how hard we need to be learning about this or else its just as good to cut off your hand (if I mistakenly believe this will make you happy) as it is to sing you a song. To some extent, rule utilitarians and act utilitarians disagree about the degree to which we need rules to overcome this problem with Mill’s approach.

Second, there’s a worry about how adaptive our ethical theory should be. Do we adopt principles that would maximize happiness (e.g. R.M. Hare) or do we adjust continuously towards what will maximize happiness? If you answer the former, then it is rule utilitarianism. Otherwise, it is act utilitarianism. I take it the two worries that need to be balanced here are this. On the one hand, if you wind up with rules and complex considerations of when you can amend them, you sound a lot like like a deontologist – something other than utility by itself is guiding you. On the other hand is the worry you raise about the concerns of calculation joined with an anti-theory position and a haphazard reality.

Answer 3 (score 1)

Act utilitarianism’s main question, with a quote from https://www4.uwsp.edu/philosophy/dwarren/IntroBook/ValueTheory%5CConsequentialism%5CActVsRule%5CActVsRule.htm, is “does this specific action maximize happiness more than any other that could be performed at the time?” Act utilitarianism is basically a specific interest that is justifiable if, and only if, it makes others happy, too. Rule utilitarianism is a more general form. It’s main question is “what general rule would this action follow, and would it maximize happiness if generally followed?” So if this action was made a universal law, would it create good? Such as lying; lying, under certain conditions, could be justifiable. However, the rule utilitarian would say that lying, if followed as a general law/rule, would not create good.

Act=specific interest that is able to be justified to maximize happiness.

Rule=general happiness that would be maximized and is able to create maximized happiness if followed as a general rule. :)

3: What is the purpose of life? Why should we live or even be created? (score 86315 in 2015)

Question

I have this empty feeling in life and no matter how many times I try to question myself, the answers don’t seem to emerge. I feel like I don’t have a purpose to live and consider myself as just one of many in this world running a rat race.If you have any clue to this answer, Can you explain?

Answer accepted (score 7)

First, your emotions around this question are something that I would advise you to seek help with - whether it is with a trusted friend or relative, or someone trained to help people deal with such a sense of despair. I can tell you from personal experience, it is not an academic matter, and can be a matter of life or death. So talk to someone about how you feel, irrespective of the philosophical ramifications.

Now to the question. Generally, people seek meaning for their lives from two types of sources: objective or personal.

Objective purposes for life can be provided by the orthodox religions, some social movements, any number of collective constructs that adhere to a particular world view. In these cases, this is not a personal search of yours, but a matter of accepting a doctrine and committing to a life lived accordingly. Your part of it is deciding how you can fulfill this given purpose. Many wise and intelligent people have lived their lives according to a purpose provided to them by a school of thought, religion or other institution.

Those who are not convinced by such extrinsic influences, who have searched their own minds, explored the universe around them and found no extrinsic purpose that they believe is valid - these people have two options.

On one hand, they can adopt a lack of purpose and a kind of hopelessness as their world view, and live accordingly. Some see this as a ticket to perfect freedom. How they use this freedom is entirely up to them.

On the other hand, they can decide that a purposeless life is not worth living, and there is no power in the universe besides themselves to provide it for them. In this case, they survey the landscape of their associations - to themselves, to their family and friends, to their profession, the groups, national or ethnic to which they belong - and decide to forge a purpose for themselves out of these relationships.

This is heavy going, a tremendous responsibility. However, I will simply name for you such luminaries as Albert Einstein, Bertrand Russell, probably Eleanor Roosevelt, and many others as merely famous examples.

As you discuss your feelings with others on this topic - which again, I urge you to do - you will discover less famous persons who have struggled with this responsibility. They will have suggestions on how to go about finding a purpose, from either objective sources, or from within yourself.

So in the short term, reading your question, you HAVE forged a purpose - of finding a purpose. You have set your feet on a journey traveled by many of your fellow humans. Step one completed. Congratulations.

Answer 2 (score 2)

Before giving my two cents, I’ll echo the original response: you may want to seek some kind of help if you are suffering deep emotional pain.

With that said, here is a take I agree with. I think it’s the Existentialist view, a view that comes off as depressing, but in reality can be deeply liberating if we let it…

What is the purpose of a hammer? To drive nails. What is the purpose of a car? To go places. In each case the purpose of X is the greater role it serves.

This means asking about the purpose of life is asking what greater thing it serves. Therefore, unless you believe there is something beyond life to which it can stand in such a relation (e.g.: God), then the question of purpose is incoherent.

I’m not saying life has no purpose. I’m saying the question of purpose doesn’t apply. So life neither has nor has no purpose.

Now, this can be deeply liberating or deeply depressing, depending on your view. The liberating part is that purpose is a double edged sword. The moment we abandon the notion of purpose, life becomes more open. Now, instead of certain narrow tracks we can tread depending on how “important” they are, we realize life simply is, and are more free to wander the field at will.

Answer 3 (score 1)

For me there is no porpouse in live. You basicly live because of evolution. There are no responsibilities that come with live (you ‘need’ to adopt to your environment: country, family, etc.). So basicly you have a finite amount of time you can spend however you want. After this time there is nothing for you. All your acomplishments stay on this world and disappear after a short amount of time.

The most logical thing would be to live a happy live. If you think a bit further there are 2 types of thinks that make you happy. One is good the other is bad.

If you dont want to be blind, always choose the moraly good thing.

Source: Plato

Why live: Because you can. Even if you live a bad life you dont get anything by dying.

4: Who are the most influential living philosophers? (score 81508 in 2011)

Question

Who are the most important, widely-read or influential living philosophers still actively working and contributing to the field today? Which thinkers are recognized for doing the most interesting and urgent work?

This is clearly subjective borderline territory so please try to justify your claim. As per the discussion it’s probably not that important to limit yourself to one thinker per answer, but the best answers already justify their decisions based on a reasonable assessment of the importance of the new concepts the thinkers produced. It isn’t necessary that the thinker be the most prolific, but she or he should probably rank fairly high in terms of number of citations (Gettier, for instance.) Thank you!

Answer accepted (score 5)

In analytic philosophy, I would say Saul Kripke, without a doubt.

Although his output has by no means been prolific, everything he has written has had a huge and lasting impact. He is quite a character too.

See especially, Naming and Necessity and Wittgenstein on Rules and Private Language.

Answer 2 (score 4)

The Wikipedia article on Saul Kripke, referenced by @Chuck, actually links to a survey done of 600 philosophers asking Who is the most important philosopher of the past 200 years?

The top 10 are:

  1. Ludwig Wittgenstein
  2. Gottlob Frege
  3. Bertrand Russell
  4. John Stuart Mill
  5. W.V.O. Quine
  6. G.W.F. Hegel
  7. Saul Kripke
  8. Friedrich Nietzsche
  9. Karl Marx
  10. Soren Kierkegaard

I can’t speak to the rigor of this survey, but perhaps it’s a little more objective than just naming people, at the very least.

Answer 3 (score 3)

Noam Chomsky

Although known widely as a political dissident and an anarchist, Noam Chomsky’s opinions are widely read and he is definitely one among the most influential living philosophers.

5: What did Nietzsche mean by monsters and the abyss? (score 73267 in 2014)

Question

What do you think Nietzsche meant by “Whoever fights monsters should see to it that in the process he does not become a monster. And when you look long into an abyss, the abyss also looks into you.” (Beyond Good and Evil, 146)? What kind of monster? What does it mean to look into an abyss?

Answer accepted (score 16)

This is one of the aspects of Nietzsche that is easily overlooked by people who want to see him as simply nihilistic and destructive.

For Nietzsche, the construction of the self is not a religious act, an obligation, or an act of submission to nature, as variously seen by ‘moralities’ – it is an art form. In The Gay Science he says something to the order of ‘One must make of one’s Self a work of art, carving away something here, growing something there, repurposing some mass of unavoidable ugliness elsewhere to present a more pleasant view from the distance…’ (I do not have a copy here, and I cannot find it online, if someone can give me the words…)

A monster is one whose ‘self’ lacks ‘art’.

Power may be the medium of morality, and its goal, but tasteless use of power is like tasteless use of any other medium. To see his aesthetic, you can look at his own artistic process, which he displayed over and over again by choosing mythological or poetic representations, or you can look at his critiques of other’s work. Particularly, I think it is why he bothered to publish ‘contra Wagner’.

He accuses Wagner’s music of being an assault on the audience, brandishing its scale in a way that shocks the senses and bruises the organs, and of having too little consistency and comprehensibility – winding an endless melody, rather than a theme.

In this context, I think the quote about monsters indicates there are aesthetic choices that we should restrain ourselves from making even though they would be effective. We should choose scale, elegance and consistency. If others’ use of power lacks art, we should not simply confront them with more power, if that involves less art. We should restrain ourselves.

In particular, I think ‘an abyss’ is a sort of monster, the monster of complete cynicism and true nihilism – the completely empty man that early ‘beatnik’ post-modernism seems to favor. There is always power to be uncovered by renunciation of boundaries, but pursuing an utter lack of restraining form leaves one ‘powerfully empty’, and perhaps incapable of recovering one’s artistic nature.

Answer 2 (score 2)

“Whoever fights monsters should see to it that in the process he does not become a monster.”

If you engage in any kind of activity, you begin to embrace the viewpoints and facts related to the activity. If you keep on immersing yourself, the more all-encompassing the viewpoint becomes: “if the only tool you have is a hammer, to treat everything as if it were a nail.”

I believe that for Nietzsche, facts, interpretations and activities are always tied together, which can be illustrated in terms of how people in different occupations see the world around them.

E.g. If you are a doctor, you solve health-related problems daily (activity), you observe facts which are related to health (e.g. pulse, blood pressure, breathing, general wellbeing) and often think of different things how they relate to health (interpretation). On the other hand, if one works as a manufacturer, one knows how produce certain kinds of things (activity and viewpoint). One also knows how much materials cost (fact) and how much people are willing to pay for the goods (facts). I believe this is the idea behind another famous Nietzsche quote: “There are no facts, only interpretations”.


I assume that N. means by ‘facts’ things which confront the individual as external constraints and the individual has no power over them.

However, the facts are tied to certain kind of activities: If I drive a car, I must abide the driving regulations, the technical constraints of the car, and the financial realities of keeping the car in shape and fixing it whenever faults emerge. I can use the car as I please but I have to abide to these constraints.

On the other hand, if I sell the car and decide to go with a bike instead, the constraints/ facts of car-driving no longer apply to me. I am then bound by the constraints which control riding a bike (different kind of regulations, regions where I ride and so on). I no longer need to care about whether gas costs 1,3 euros or 2,6 euros because I am no longer engaged in car-driving. When I switch from car driving to riding a bike, the activity changes and so do the constraints.

I cannot alter the facts associated to particular activity, but I can find freedom in choosing what kind activity I engage in. The types of activities which are available for me at the moment, are determined by the society in which I have been born into. And engaging, I also gain the particular types of freedoms associated to that particular activity.

Facts always require seriousness from people and try to convince that they are eternal and never change. N. is pointing out with this example that the facts of witch hunting were tied to interpretation which was prevalent at certain time but as times have changed, people no longer dabble in witch hunts and he is claiming that this applies to all human activities.

Answer 3 (score 1)

Whoever fights monsters should see to it that in the process he does not become a monster.

It doesn’t sound to me like Nietzsche is saying people shouldn’t fight evil. I don’t know what Nietzsche’s beliefs regarding justice and revenge were, but I interpret his words as a question of balance. If you’re going to fight evil, be careful you don’t become the very evil you’re fighting.

And when you look long into an abyss, the abyss also looks into you.

This can be interpreted in various ways.

One interpretation is that people who hate evil should remember that there’s a little evil in all of us. Rather than put ourselves on a pedestal, we should carefully examine our own lives.

Another interpretation is that focusing too intently on evil can either twist one’s mind or simply induce a depression so great it drags us down into the abyss. Some might simply call it “burnout.”

6: Deontology: Perfect vs. Imperfect Duties (score 73234 in 2014)

Question

I have a question regarding Kant’s deontological Maxim of both Perfect and Imperfect duties.

What I know about them is that Perfect duties require a person to perform a certain “action” all the time, while Imperfect duties require a person to do that “action” sometimes.

Could someone please help define the duties in layman’s terms, and the difference? It’s getting more confusing. Thanks!

Answer accepted (score 4)

Kant raises a distinction between what he calls perfect duties and imperfect duties in the Groundwork of the Metaphysics of Morals and again in the Metaphysics of Morals: Doctrine of Virtue. You have the basic definition in hand: a perfect duty is one which one must always do and an imperfect duty is a duty which one must not ignore but admits of multiple means of fulfillment.

Kant specifies two imperfect duties: the duty of self-improvement and the duty to aid others. To understand why Kant thinks of these as imperfect duties, we need to first understand the nature of duty for Kant. The literature on this is vast, so I’m going to skip over some parts of the mechanics and summarize it as follows: a duty is something that we are obligated to by the Categorical Imperative. In other words, it is something that that we can see as a universal rule for all of humanity necessary for a morally just society (mixing together all three major types of formulations of the Categorical Imperative).

It’s difficult to come up with completely non-problematic duties due to some issues related to the basis on which we act “maxims” and the means through which we universalize these, but I will skip over this for the purposes of this question (If interested, I suggest reading Allen Wood’s Kant’s Ethical Thought).

Let’s say that I want to lie to someone. If we universalize this, then every rational creature will lie whenever it is convenient. This will turn out to be self-defeating because no one will believe what anyone says. Since we have a constant need of truth in our dealings, this is something we must practice at all times. (i.e., we cannot add an exception "except when telling the truth is inconvenient). This makes this a perfect duty in the Kantian system. Most perfect duties turn out to be negative duties – i.e. don’t do X. On Kant’s system, every rational being is obligated by perfect duties.

Imperfect duties reflect the nature of human rational existence. We are born weak and frail, we cannot do everything by ourselves, and we die. These realities create interesting non-rational features of our reality: I needed someone to feed me when I was a baby. I need someone to help me when my car is stuck. I need a surgeon when my liver fails. These needs are not universal either in time or duration nor are they purely rational laws. To make these desires moral, Kant needs us to universalize them. Thus, we transform I need help at times into every [limited] rational creature has a duty to help other rational creatures at times. Thus, I have a requirement to aid others at times reflective of my own need for help at other times. This is one of the two imperfect duties for Kant.

The second imperfect duty is to perfect myself. This duty arises because when I need help, I need experts. Thus, the only way that rational creatures can have their needs met is if rational creatures are developing their talents. So, I too have a need to develop my talents in order to create a universalizable rule that would make it so aid is available when I need it of sufficient ability.

Moved to the level of the particular, imperfect duties are things like: study chemistry, practice the violin, learn Japanese, volunteer at an orphanage. These are duties I don’t need constantly and that I somewhat pick from among. Thus, they are imperfect duties since they are not constant obligations, but they remain obligations.

What Kant does not answer is how often. He puts this question somewhat in the Doctrine of Virtue. You can read more about this in Creating a Necessity out of Virtue by Nancy Sherman.


I also gave a more thorough argument for this account in my dissertation.

Answer 2 (score 0)

I do not agree that perfect duties, for Kant, are those which you must always do, while imperfect duties are duties you don’t always have to fulfill. This way of putting the distinction encourages people to accep the view of Kant as an absolutist in the sense that he does not allow exceptions to moral rules. This interpretation renders Kant’s theory useless in attempting to deal with moral issues involving conflicts of duties. Kant defines perfect duties as those not allowing any exceptions to the inclinations. He does not say that they allow of no exceptions at all. In the introduction to the Meta. Of Morals Kant allows that when “grounds of obligation conflict,one cancels the other one out, in whole or in part.” The essay ,“On the alleged right to lie from beneficent motives” which also seems to support the absolutist view, concerns the matter of how the law can respond to a lie intended to save an innocent person, but it does not state clearly that it would be immoral to lie, except in a merely. formal sense.

7: What did David Hume mean when he said that “reason is a slave to the passions”? (score 67670 in 2013)

Question

I don’t understand the meaning of this oft-quoted quotation of Hume’s in On Reason, namely his saying that “reason is a slave to the passions.” What exactly does he mean by that ? Is it simply that reason is subsequent to a deeper moral sense? Is it equivalent to the maxim today that “science cannot answer moral questions”? One thing that may be confusing is me is that I sense he’s being somewhat rhetorical; would it be better to summarize his the arc of work that reason can only guide the passions, and that the truths we think it is uncovering us are ultimately a product of what our fickle passions urge it to investigate?

Answer accepted (score 41)

Hume’s quotation is from a famous passage discussing the “motivating influence of the will” in his Treatise on Human Nature and reads in full:

Reason is, and ought only to be the slave of the passions, and can never pretend to any other office than to serve and obey them. (T 2.3.3 p. 415)

The context is his discussion of what is sometimes called “moral psychology”, the study of how we are motivated to act morally. In particular, he raises a question about the role of practical reason in moral motivation. Hume vehemently opposes the view, held by philosophers before him (and after him), that to act morally is have a rational grasp of moral truths. He defends an instrumental conception of practical reason, according to which the role of reason is only to find out which means helps achieve a given goal. Reason (or the intellect) plays no part in determining the goals. Our goals are set exclusively by what Hume calls the passions and what today is most often called desires. Desires cannot be evaluated as true or false or as reasonable or unreasonable - they are “original existences” in our mind and arise from unknown natural causes. We cannot be criticized rationally for our desires (As Hume remarks, it is “not contrary to reason to prefer the destruction of the whole world to the scratching of my finger” (p 416)).

Reason is the slave of the passions in the sense that practical reason alone cannot give rise to moral motivation; it is altogether dependent on pre-existing desires that furnish motivational force. For Hume, this is not a fact we should lament (as moralists do) but a basic fact about our psychology.

Answer 2 (score 11)

You can apply this quotation in many different contexts as far as Hume’s thought is concerned - in general I think the best way to read it is as an outgrowth of his radical empiricism which in the case of ethics descends into his famous advocacy of emotivism. The point is that reason will never reach out into the world - the passions are what we get when the world reaches into us. And therefore, reason will never be able to control them or understand them because they (the passions/sensations/impressions) are the raw materials of reason. So here are some ways to understand this general tendency, as expressed in the famous maxim you quote:

  1. Emotivism in Ethics: Reason cannot enter into our ethical judgements because these judgements are based on sentiments (i.e. passions.) An act of cruelty will cause in us a feeling of injustice, and that feeling (sentiment/passion) will be the reason why we pass an unfavourable judgement on an act of cruelty. This is the polar opposite of a Kantian view of ethical judgements to which one arrives at by pure a priori reasoning.
  2. Taste in Aesthetics: Here the situation is more complicated. Hume admits that beauty can only ever be as it were ‘in the eye of the beholder’ because beauty cannot be in the object but must be wholly contained in the pleasurable sentiment it causes us. Indeed Hume explicitly states that pleasure is the essence of beauty (we define as beautiful that which gives us pleasurable sensations.) So here you see that reason will always be a slave of the passions, i.e. you will never be able to rationally convince your friend who thinks artworks by Dali are ugly that they are, in fact, beautiful. Nevertheless in On the Standard of Taste Hume tries to argue that there are in fact some objective aesthetic standards, by urging us to heed the advice of ideal critics which he goes on to define (i.e. critics possessed of a delicacy of taste, and sound understanding, sharpened by practice and comparison and who are free from prejudice.)
  3. Self and Causation: Hume also famously argued that there is no such thing as direct causation, only observable regularities; and that there is no indestructible self, only impressions which we call our own. If you take passions to include sensations of hot or cold or sense impressions, then you can interpret the maxim that reason will always be a slave to such passions as a further advocacy of this type of empiricism or anti-realism. The latter (anti-realism) in the sense that there are no shared constraints that ever effectively decide such questions as ‘Am I here?’ or ‘Is this colder than this?’

Answer 3 (score 4)

Hume’s argument is that all preferences and motives are emotional. There is no such thing as an unemotional or purely rational decision, because to decide, by its nature, is to have a preference for, i.e. an inclination toward or aversion to, something.

Reason’s (i.e., cognition’s) role is to structure the world for us: it lays out a schematic of how objects and ideas are connected. But this is merely cold information, devoid of any significance on its own. Rational categories have no value or priority without feeling.

The passions (i.e., emotions or affects) are necessary to evaluate any object or idea as valuable, problematic, virtuous, immoral, good, or bad, because these forms of evaluation are all based on either a positive or negative impression. Decisionmaking is neither an rational nor “irrational” (whatever the hell that means) process: it is based on preferences, which must arise from emotional states and can never just magically be produced by reason alone.

What reason does is enable things to become positive or negative by association. I have a goal that I care about, and reason suggests to me that a previously unimportant object will help me achieve it. Thus the object itself becomes important to me. Reason creates the association–it’s a telescope that allows me to see the distant or indirect emotional consequences of the object–but the emotion itself remains my only motive and only decider.

8: How will learning about philosophy impact real-life? (score 67448 in 2015)

Question

Examining the practical, impactful, real-life benefits of getting a degree in philosophy
It is often asked, “What is the practical use of philosophy?” or “How will learning about philosophy impact real-life?”

It is easy to see why there is such mystery surrounding the topic. Most schools up through high school may only offer 1 or 2 courses on it, if any. It is also often talked about in general conversation as a “wishy-washy” subject, often negatively, as though it contains no answers but only raises an endless series of questions.

To add to this, those who hear of the subject but know very little of it may be surprised to see it offered at their university as a major. It is clear to most why some people would want to major in Computer science, Mathematics, or Physics. But what would a degree in philosophy get you?

Answer accepted (score 47)

The Practical Use of Anything

It is difficult to talk about the “practical use” of almost any piece of knowledge out of context. In fact, I can conceive of no single piece of knowledge is universally beneficial. For example, on the face of it you might think that mathematics is a more practical area of study than philosophy, but can you honestly say that you’ve actually ever used the formula for the volume of a cylinder in real-life? Did you ever whip out the quadratic formula on a napkin in a restaurant to help calculate the tip? Most knowledge in every discipline is helpful only in specific contexts, so we must be careful when we talk about the “practical use” of any field of study. The only relevant distinction here is which pieces of knowledge are more helpful in everyday life vs. those which are less so. From this perspective, both mathematics and physics—which are in some senses seen as the foundational/core sciences—are probably among the least practical. Ranking up there would be psychology and sociology if you interact with other humans on any regular basis, biology if you want to know how your own body works, heck, maybe your understanding can be used to save people’s lives. As you can probably see this is somewhat of a subjective judgement, but if you were to ask me I would hold that there is one field that rises above them all, and that is philosophy.

What in the world does philosophy teach you that could possibly be so useful, you wonder. That’s a good question, and I will elaborate below, but there is another important distinction to remember:

Skills vs. Facts

I think it’s important to remember—as it is also true in many other fields—that learning in philosophy can be distinguished between skill knowledge and factual knowledge. That is, there is a big difference between philosophy as in critical thinking skills and reasoning ability, versus philosophy as in facts regarding the history of philosophy and conceptual ideas. Virtually all philosophy courses address both sides of this coin (they encourage and help develop good reasoning in the context of popular concepts or the history of philosophy), but where I think philosophy is most useful (when it comes to real life) is the former notion. As a college graduate who has taken multiple courses in a wide range of disciplines, one of the skills that philosophy seems to emphasize most is critical thinking. Of course, I can only speak from the perspective of one university (others may not emphasize this as much), but I would wager that many people who have studied philosophy at the college level would concur with this statement. And having this ability to think critically is of immense benefit for a number of real-life situations such that it’s not even reasonable to begin listing them all, because it would otherwise be disingenuous to the entire scope of benefits. At any moment in life where you have to make a decision about something, that decision will always depend on your reasoning abilities. In this way, being a good thinker can help you become a better manager, a better consumer, a better writer, a better reader, a better voter, a better leader, a better father or mother, brother or sister, a better friend…

Outside of the critical thinking skills you learn in philosophy, you have the history of philosophy. Knowing that René Descartes was born on March 31st or the definition of determinism is as equally useful and domain specific as just about any other piece of knowledge. I can honestly say that I have never needed to know that the American Revolutionary War went from 1775–1783 after learning it in 11th grade AP US History. Lots of random pieces of knowledge like this don’t often present any particular use in our daily lives, but that’s to be expected. When you decide that you want to learn something, you have to ask yourself why you want to learn. Is it for a job? Is it to increase your knowledge about the world? It is these questions which help you decide what to learn. You have to figure out your goals before you can decide what to learn and how useful it will be for you.

Uses of a Philosophy Degree

These days, typically the only people who get advanced degrees in philosophy are those who want to teach philosophy (become professors). I’m sorry if this comes as bad news to some people, but alas, there aren’t very many jobs available for those of us who would like to just sit around and philosophize. If you have no plans on becoming a philosophy professor, I would not recommend making philosophy your sole major (either double major or take philosophy as a minor). That said, I know many people who solely have a degree in philosophy. This is not the end of the world by any means, and many great jobs require no specific degree, and that’s assuming you don’t plan on going back to school.

In terms of graduate school, a lot of people who plan on going into law start with philosophy degrees, and many schools will offer a philosophy of law program in addition to the standard philosophy track. People with philosophy degrees also seem to have no trouble getting into business or journalism programs.

In the end, you should learn what interests you, but it’s appropriate to keep your future in mind as well. If you are concerned, consider double-majoring so you can keep your options open. Double-majoring wasn’t hard as I had to take many electives/humanities anyways, and philosophy courses often filled those gaps. With decent planning you should be able to finish a double major for two Bachelor’s degrees in the normal time frame (4 years) without taking too many more extra courses at most schools (I completed a double-major and a minor in 4 years with no summer or winter classes).

Further Reading:
Why study philosophy?

A very good overall guide for why philosophy can help you in everyday life, and what you can do with a philosophy degree.

Philosophy majors outperform all other majors

A great collection of articles and statistics which show the benefit of studying philosophy.

“In the US, where the number of philosophy graduates has increased by 5 per cent a year during the 1990’s, only a very few go on to become philosophers. Their employability, at 98.9 per cent, is impressive by any standard….Philosophy is, in commercial jargon, the ultimate ‘transferable work skill’.”

The Philosophy Major’s Career Book

The aim of this handbook is to help you think about the relation between your degree in philosophy and getting a job or planning a career (or life) after graduation, and to help you to prepare intelligently for it.

USA Today: What can you do with a philosophy degree?

“So many people think philosophy isn’t practical,” says Shoener, who also is studying biomathematics for a double major and plans to be a women’s health advocate. “It’s the most practical thing I’ve ever done.”

Answer 2 (score 26)

Gilles Deleuze’s thoughts on this, from Nietzsche and Philosophy (my emphasis):

When someone asks “what’s the use of philosophy?” the reply must be aggressive, since the question tries to be ironic and caustic. Philosophy does not serve the State or the Church, who have other concerns. It serves no established power. The use of philosophy is to sadden. A philosophy that saddens no one, that annoys no one, is not a philosophy. It is useful for harming stupidity, for turning stupidity into something shameful. Its only use is the exposure of all forms of baseness of thought. Is there any discipline apart from philosophy that sets out to criticise all mystifications, whatever their source and aim, to expose all the fictions without which reactive forces would not prevail? Exposing as a mystification the mixture of baseness and stupidity that creates the astonishing complicity of both victims and perpetrators. Finally, turning thought into something aggressive, active and affirmative. Creating free men, that is to say men who do not confuse the aims of culture with the benefit of the State, morality or religion. Fighting the ressentiment and bad conscience, which have replaced thought for us. Conquering the negative and its false glamour. Who has an interest in all this but philosophy? Philosophy is at its most positive as critique, as an enterprise of demystification.

Answer 3 (score 11)

The practical use of philosophy of ethics:

To provide a new reference frame for the negotiation of basic values in a society where “because God wants it” or “because it is unnatural” or “because we have always done it this way” is not convincing to many people.

The practical use of philosophy of science:

Most scientists don’t want to waste too much of their precious research time on philosophical issues which leads to many scientists practicing a different point of view than they preach. Having philosophers exploring the discrepancies can lead to scientists who are actually aware of what they do which will improve their science overall.

It is similar for other branches for philosophy. And just as in other domains, the practical use does not turn up by focussing on the practical use, but by focussing on what is interesting.

9: What exactly do ‘objective’ and ‘subjective’ mean in contemporary philosophy? (score 65962 in 2014)

Question

I’m pretty new to philosophy and I just have a quick question in regards to about how people use the terms ‘objective’ and ‘subjective’.


Does objective value mean anything that is independent of one’s mind? Meaning, no matter what one agrees or disagrees of the existence of something, it still exists? Whereas subjective value is mind-dependent?

Answer accepted (score 7)

(new account, not enough rep to comment reply to Michael’s follow up question)

There are four apples and therefore the objectively correct answer is four. The third observer is wrong to claim there are 5 apples when there are four. Of course, it might be that there really are 5 apples and that the third person is correct and the first two wrong. Another possibility is that they are all wrong and there are six apples. Regardless, this is a situation where there is one correct answer.

The subjective question is about personal taste. Since it is about taste there is no conflict inherent in having different answers to the question. It is a known property of humans that they differ in their tastes. The question “Is the cake yummy” really should not be taken literally that way. The question is actually reinterpreted by people to me “Do YOU find the cake yummy”. Which when asked of different people will generate multiple objective questions, one for each person.

In this case lets call the girl Sue and the boy Joe. The subjective question, “Is it yummy” converts into two objective questions, “Is it yummy for Sue” and “Is it yummy for Joe”. The respective answers are yes and no. If Joe were to turn to sue and say “No it is not yummy” then he could be making one of two mistakes. He is either claiming that Sue thinks the cake is not yummy. In which case he is wrong. Or, he thinks that Sue was claiming the cake was yummy for everyone and was disputing that because he doesn’t like it. Depending on what Sue meant he could be right or wrong, but only about his interpretation of what Sue was saying. After all if Sue meant it was yummy for everyone she is objectively wrong.

Another distinction that can be made is whether something is metaphysically or epistomologically subjective or objective. Metaphysically objective things are things we call can see. For example the apples. Things that exist in the real world independent of any one person. Metaphysically subjective things are that which only each person experiences and can verify against each other. Exactly how Joe experiences the taste of the cake is not something Sue can experience nor directly verify. This is usually referred to as qualia. This exists in the real world but only to a single person.

Hallucinations are metaphysically subjective and NOT metaphysically objective. That is they exist subjectively but not out in the real world. Metaphysics is about what exists. Certainly hallucinations exist, and the objects of the hallucinations don’t exist or they wouldn’t be hallucinations.

When speaking epistomologically we are talking about truth claims. Those truth claims that can be decided by metaphysically objective facts are epistomologically objective. Those claims that can be decided by metaphysically subjective facts are epistomologically subjective. The statement “There are four apples” is something all parties can determing by counting. The statement “This cake is yummy” is something that is determined subjectively by tasting it.

I could tell if my cat finds foods yummy by whether she eats them. That’s an objective standard for measuring a subjective experience.

Answer 2 (score 6)

As a starting point, I can tell you that, generally:

  • the term “objective” refers to things which we deem as true/existing independent of our observations of them, and
  • subjective” refers to a things which we deem as true/existing contingent on our observation of them.

The cake and apples example in the other user’s answer below is an example of this, although it fails to take into account the fact that our count of the apples is itself subjective (maybe there’s 100’s of apples in a big bowl and we miss counting one). True, the actual amount of apples on the plate is an objective fact (by ‘fact’ here I simply mean a ‘piece of knowledge that I hold as true’), and if we were able to calculate this without error then sure, the example works. But I think a better example is the idea that my mind (myself) exists — this is more readily and more universally agreed to be an objective fact. Under no circumstances can we conceive of a situation where the thinking self (“I”) does not exist, for if we are thinking we must necessarily exist. Whether this thinking self is “smart”, however, is subjective — to an ant I may be considered smart but to a super-intelligent alien species I may be considered as unintelligent as the ant is to me. My smartness thus is a subjective fact.

The problem however is that even what we might call as objective facts we arrived at only through the processing of our own minds, which are inherently subjective. This is the problem with the apples example, as well as for mine. Objectivity for us on a human-level, it seems, is not actually intrinsic objectivity (which we can’t seem to know) but rather a form of collective subjectivity. Just because everyone agrees on the “truthness” of a fact doesn’t mean that it’s automatically an objective fact.

As you can see these concepts quickly become complicated, which is why I originally wrote that this question bordering on “too broad to be reasonably answered”. Subjectivity and objectivity mean many things to different philosophers depending on their particular views on a wide range of concepts. A good, solid answer in my opinion would really touch on all the major lines of thought that deal with it, but that’s perhaps too much work for a single question. There are many important concepts that come into play, pretty much everything in philosophy of mind, but specifically concepts such as:

Note, I linked only Wikipedia articles above. Do also check out the equivalent articles at the Stanford Encyclopedia of Philosophy.

Answer 3 (score 3)

The terms are somewhat ambiguous as to whether the sense of their use is ontological or epistemological. Professor John R. Searle explains as much in his paper, “Consciousness

Here is the ambiguity: We need to distinguish two different senses of the objective-subjective distinction. In one sense, the epistemic sense (“epistemic” here means having to do with knowledge), science is indeed objective. Scientists seek truths that are equally accessible to any competent observer and that are independent of the feelings and attitudes of the experimenters in question. An example of an epistemically objective claim would be “Bill Clinton weighs 210 pounds”. An example of an epistemically subjective claim would be “Bill Clinton is a good president”. The first is objective because its truth or falsity is settleable in a way that is independent of the feelings and attitudes of the investigators. The second is subjective because it is not so settleable. But there is another sense of the objective-subjective distinction, and that is the ontological sense (“ontological” here means having to do with existence). Some entities, such as pains, tickles, and itches, have a subjective mode of existence, in the sense that they exist only as experienced by a conscious subject. Others, such as mountains, molecules and tectonic plates have an objective mode of existence, in the sense that their existence does not depend on any consciousness.

As for objective and subjective “value” roughly yes, but this also depends upon how “value” is being used. For example, “the old worn sock puppet may not have been worth it’s weight in cotton, but its sentimental value to the child was beyond measure” is a subjective value; “the arithmetic expression evaluates such that x has a value of seventeen” is an objective value.

In the second example you may want to also look into distinguishing observer-independent and observer-relative (see section 1) as the computation of the arithmetic expression as done by a (non-human) computer is observer-relative, tho when the arithmetic expression is evaluated by a human, it is observer independent. For example, a map my show the directions from where you are to where you want to go - from point A to B. The “information” (also an ambiguous term) on the map is relative to an observer that can interpret it. The map does not know how to go anywhere. This is distinct from you actually knowing how to get from where you are to where you want to go - the information “in your head” is not relative to an observer, the conscious thought is psychologically real, actual and observer-independent.

From “Theory of Mind & Darwin’s Legacy” by Searle:

Related to the distinction between objectivity and subjectivity is the distinction between those features of the world whose existence depends on human attitudes and those features that exist independently of anyone’s attitudes. I call the former “observer relative” and the latter “observer independent” or “absolute.” Observer relative phenomena include money, property, marriage, nation states, universities and summer vacations. Observer independent phenomena include mountains, molecules, galaxies and tectonic plates. In general the natural sciences deal with observer independent phenomena; the social sciences deal with observer relative phenomena. The observer relativity of a phenomenon introduces an element of ontological subjectivity into its very existence. So the existence of money and language, for example, is observer relative and consequently contains an element of ontological subjectivity.

10: Accepted Interpretation of Machiavelli’s “the ends justify the means.”? (score 62148 in 2012)

Question

Prior to reading The Prince, I had heard of the aphorism:

The ends justify the means

This was Machiavelli’s identifying line to many people, and in my conversations with them, I got the impression that Machiavelli meant the following: that, given an ends profitable enough, any means, even a very immoral one, is justifiable.

Thus, I lived with this impression for a while, until I actually read The Prince and got to the line in question (at the end of Chapter 18). In specific context, its meaning seems very different:

In the actions of all men, and especially of princes, where there is no court to appeal to, one looks to the end. So let prince win and maintain his state: the means will always be judged honorable, and will be praised by everyone.

Here, Machiavelli does not seem to be making an ethical argument, but rather a sort of psychological one. It appears (to me) he is arguing that to the people, the ends justify the means, not that “the ends justify the means” is some moral code. Specific phrases that give me this impression are “the means will always be judged honorable, and will be praised by everyone.”

So, my question is this: among scholars, what is the generally accepted interpretation of this line? Is Machiavelli saying that the ends really do justify the means, or that the people often perceive it to? Is there some other common interpretation? What evidence is there for each perspective?

Answer accepted (score 8)

Beginning with Constantine (2007), we have the passage in question rendered in English thus:

Everyone sees what you seem to be, but few feel what you are, and those few will not dare oppose the opinion of the many who have the majesty of the state behind them: In the actions of all men, and particularly the prince, where there is no higher justice to appeal to, one looks at the outcome.

The prince ought to do what he needs to do in order to maintain his position as the prince. This is his virtue. What this requires him to do will depend upon circumstance. Sometimes he will be able to appease both his personal conscience while keeping himself in power, during other times he will have to sacrifice his own sense of what is good just to keep himself in good standing with whomever is powerful at the time.

Notice that this in no way reduces to a devil may care attitude to justice, both what the common people consider to be just, or to what the noble class may consider to be just, should the two parties have a difference in opinion. One quotation in chapter eight bears mentioning:

If one weighs Agathocles’s actions and skill, there is not much that can be attributed to Fortune. As I have pointed out, he did not gain his principality through anyone’s favor, but rose to it through the ranks of the army with a thousand privations and dangers, and then kept possession of the principality through many bold and dangerous feats. And yet we cannot define as skillful killing one’s fellow citizens, betraying one’s friends, and showing no loyalty, mercy or moral obligation. These means can lead to power, but not glory. Because if one considers Agathocles’s skill at plunging into and out of danger, and the greatness of his spirit in enduring and overcoming adversity, he cannot be judged inferior to the most excellent leaders. In other words, one cannot attribute to Fortune or skill what he attained without either of them.

In this long excerpt we find M. calling for restraint towards the same virtues that may bring a prince to power. The Roman sense of virtue: of manliness, of martial skill and ability, are on some occasions subordinated to ostensibly higher values of “loyalty, mercy or moral obligation.” From this I conclude that a general may make a fine prince, but princely duties are not to be reduced to martial ones.

So what obligation does a prince have, in general? It’s simple, really. To keep himself in power.

Answer 2 (score 1)

Analysis of Machiavelli’s Word Choice – The Ends Define The Means

I choose to analyze the phrase, “the end justify the means,” to determine why Machiavelli would have chosen the word “justify.” Based on my interpretation, Machiavelli is more likely to have believed that, the ends “define” the means, and given Machiavelli’s ends, it is clear why in the final of the version book dedicated to Lorenzo di Piero de’ Medici, he chose to print “the ends justify the means.”

Machiavelli had achieved some success under the Florentine Republic (after the Medicis were expelled), but was to a certain extent exiled upon the return of the Medicis. When writing this book, and dedicating it to Lorenzo di Piero de’ Medici, we cannot forget that Machiavelli’s “ends” were to appease and flatter the Midicis and achieve an end to his exile.

Given that the Medicis had just used mercenaries to conquer a republic, of course they would want to “feel justified” in their rule.

Machiavelli’s ends to flatter the Medicis defined how he wrote his book and his word choice. Therefore, he told the Medicis exactly what the wanted to hear, “The ends justify the means.”

By analyzing Machiavelli’s word choice, we can truly see that: The Ends Define The Means

Answer 3 (score 1)

Analysis of Machiavelli’s Word Choice – The Ends Define The Means

I choose to analyze the phrase, “the end justify the means,” to determine why Machiavelli would have chosen the word “justify.” Based on my interpretation, Machiavelli is more likely to have believed that, the ends “define” the means, and given Machiavelli’s ends, it is clear why in the final of the version book dedicated to Lorenzo di Piero de’ Medici, he chose to print “the ends justify the means.”

Machiavelli had achieved some success under the Florentine Republic (after the Medicis were expelled), but was to a certain extent exiled upon the return of the Medicis. When writing this book, and dedicating it to Lorenzo di Piero de’ Medici, we cannot forget that Machiavelli’s “ends” were to appease and flatter the Midicis and achieve an end to his exile.

Given that the Medicis had just used mercenaries to conquer a republic, of course they would want to “feel justified” in their rule.

Machiavelli’s ends to flatter the Medicis defined how he wrote his book and his word choice. Therefore, he told the Medicis exactly what the wanted to hear, “The ends justify the means.”

By analyzing Machiavelli’s word choice, we can truly see that: The Ends Define The Means

11: What is the difference between metaphysics and ontology? (score 53204 in 2011)

Question

I know that ontology is a sub-field of metaphysics. But I can’t see the difference between them. I mean ontology is defined as “The study of being and existence”, and metaphysics is defined as “fundamental nature of being and the world”; is there a difference between these two definitions?

Answer accepted (score 33)

This is an excellent question, and deserves more discussion than I can really provide here, but I’ll try to give a simple and clear delineation between the two fields.

Metaphysics is a very broad field, and metaphysicians attempt to answer questions about how the world is. Ontology is a related sub-field, partially within metaphysics, that answers questions of what things exist in the world. An ontology posits which entities exist in the world. So, while a metaphysics may include an implicit ontology (which means, how your theory describes the world may imply specific things in the world), they are not necessary the same field of study.

Let’s consider an example that might clarify the distinction a little more: gravity. Gravity is certainly not an object, but I assume that physics would be in a pretty bad spot if we say that gravity isn’t real. So what should we make of gravity? Well, we roughly know that gravity is a physical law that affects matter.

So, a metaphysical conclusion we can draw from this is:

“The world is such that matter is governed by physical laws.”

This is a metaphysical conclusion because it describes a way that reality is - laws are a property of reality (and further, reality is the kind of reality that can have laws). The distinct ontological conclusion to draw from our discussion so far is:

“There is a physical law of gravity.”͏͏͏͏͏͏

This is ontological because it is about a posited entity - the entity that we call “the law of gravity”. Now, where most people seem to get tripped up here is in the fact that our ontological conclusion is also a metaphysical conclusion; any posited entities are also part of (or describe) how the world may or may not be. But the ontological conclusion differs from the non-ontological metaphysical conclusion insofar as it may imply metaphysical conclusions, but is not itself about reality. “Meta” means roughly “about” or “after”, and physics means physics, so metaphysics means “about (or after/beyond) reality”. The ontological conclusion is about a given entity (or kind of entities).

I hope that helps!

Answer 2 (score 13)

Metaphysics

generally covers topics such as cosmology (space and time), determinism and free will, mind and matter, ontology (being, existence, reality), necessity and possibility, identity and change, among others.

Ontology

is just one of those subtopics of metaphysics; it focuses on the categories of being and whether things can be said to exist or not.

The Wikipedia articles on Metaphysics and Ontology are not entirely clear but you can get an idea of the differences by looking over each. The SEP article on Metaphysics (and ontology) is more in-depth and I would definitely recommend it if you want a thorough overview, especially since someone seems to want to vandalize the Wikipedia Ontology page

Answer 3 (score 2)

I hope I’m not over-simplifying… According to what I understood from my philosophy teacher, metaphysics is comprised of three fields:

  1. Ontology - discourse of that which exists,
  2. Epistemology - discourse of what we can know,
  3. Ethics - discourse of that which follows (for human behavior)

This distinction is useful, such as for discussing that…

  • Heidegger accuses philosophers since Socrates of moving too fast past the “existential” (ontology, Being).

  • Levinas accuses Heidegger of placing ontology over ethics, with disastrous historical consequences.

Source: “Totality and Infinity” by E. Levinas, p.46f

12: What is the difference between knowledge and belief? (score 49823 in 2013)

Question

Sometimes this image is used to explain what agnosticism is and how it’s independent from belief:

enter image description here

It makes some sense but I still have confusion understanding it.

What is the difference between knowledge and belief?

Answer accepted (score 10)

Knowledge is a particular kind of belief, one that has (or has more) evidence, and justified at that (of course there is the classic Gettier problem with this definition).

The picture you gave shows two axes, one from theism to atheism (the subject matter about what one knows/believes), and an orthogonal one or gnosis to agnosis, or what I take it, to be the degree of belief with gnosis being knowledge (certain belief) and agnosis being…

Well, that’s the problem. What is that axis ‘measuring’? Is it the certainty (which would presumably go from ‘sure’ knowledge to …unsure. Is unsure knowledge the same as belief? I think of knowledge as one kind of belief, a very sure kind of belief, rather than in opposition to belief.

For the diagram, I’d say that the a/gnosis axis is really trying to quantify ‘certainty’. At one end one is -very- sure of one’s belief that a god exists (or doesn’t). At the other, one is completely unsure of the statement.

My problem with this diagram is that is seems perverse to say ‘I believe that X, but I am completely unsure of X’. Those seem contradictory. If you are completely unsure of X, then I would say you can’t believe it. I guess one could be a theist and be unsure about it, but if one were -completely- unsure of it, then that wold just be an agnostic, directly in the middle, rather than being an agnostic theist.

The meta-lesson that I learn from this diagram is that a nice clean diagram does not necessarily exhibit coherent or consistent concepts.

Is the ‘gnostic/agnostic’ axis about a continuum between proof and faith? That might be a more orthogonal and coherent thing, but there’s no evidence in the picture that that is the case.

Answer 2 (score 10)

Knowledge is a particular kind of belief, one that has (or has more) evidence, and justified at that (of course there is the classic Gettier problem with this definition).

The picture you gave shows two axes, one from theism to atheism (the subject matter about what one knows/believes), and an orthogonal one or gnosis to agnosis, or what I take it, to be the degree of belief with gnosis being knowledge (certain belief) and agnosis being…

Well, that’s the problem. What is that axis ‘measuring’? Is it the certainty (which would presumably go from ‘sure’ knowledge to …unsure. Is unsure knowledge the same as belief? I think of knowledge as one kind of belief, a very sure kind of belief, rather than in opposition to belief.

For the diagram, I’d say that the a/gnosis axis is really trying to quantify ‘certainty’. At one end one is -very- sure of one’s belief that a god exists (or doesn’t). At the other, one is completely unsure of the statement.

My problem with this diagram is that is seems perverse to say ‘I believe that X, but I am completely unsure of X’. Those seem contradictory. If you are completely unsure of X, then I would say you can’t believe it. I guess one could be a theist and be unsure about it, but if one were -completely- unsure of it, then that wold just be an agnostic, directly in the middle, rather than being an agnostic theist.

The meta-lesson that I learn from this diagram is that a nice clean diagram does not necessarily exhibit coherent or consistent concepts.

Is the ‘gnostic/agnostic’ axis about a continuum between proof and faith? That might be a more orthogonal and coherent thing, but there’s no evidence in the picture that that is the case.

Answer 3 (score 8)

Knowledge, of the kind you’re asking about, I think, usually requires evidence and reasoning. In extreme cases where such knowledge doesn’t require both evidence and reasoning, such as in parts of symbolic logic, knowledge requires only reasoning.

On the other hand, belief doesn’t require any reasoning or evidence whatsoever.

If I know that the sun burns at, or around, a certain temperature, then either there exists some perceptual data as evidence of this, or some perceptual data exists which, along with reasoning, implies the sun as burning at, or around, that certain temperature. So, a claim of the sun burning at, or around, that certain temperature comes as sufficiently grounded.

On the other hand, if one believes the sun burns at a certain temperature, there might not exist any evidence or reasoning which grounds such a claim. One could believe something in one’s sleep quite easily. Unless you believe dreams provide us with empirical information about the sun, I think this indicates beliefs as not needing evidence or reasoning. This isn’t to say that no beliefs can get grounded via reasoning or evidence. Plenty of knowledge, also is believed (I know I have a hand, and I believe it too). However, no belief purely as a belief need get grounded via reasoning or evidence to qualify as a belief. Knowledge does need at least some sort of ground, and if a claim is not grounded via reasoning or evidence, then it comes as a strongly believed speculation at best.

Unfortunately I don’t have any “atheist” or “agnostic” literature citations here, but as I recall reading “atheist” and “agnostic” literature they do seem to use the terms at least somewhat in that way.

The Wikipedia on “Descriptive Knowledge” says this: “The difference between knowledge and beliefs is as follows:. A belief is an internal thought or memory which exists in one’s mind. Most people accept that for a belief to be knowledge it must be, at least, true and justified.”

13: What is the relationship between philosophy and science? (score 46977 in 2012)

Question

While philosophy and science as held as separate disciplines (and often taught in completely different colleges within a university [i.e. College of Liberal Arts vs. College of Science]), it is patently clear that there is an immutable relationship between philosophy and science. Philosophy can often be seen as providing justification for particular scientific theories – but why/how is this so?

In science:

  • A theory must have observable consequences that can be tested and be falsified.
  • Theories also must agree with previous theories in the domains where they had been successfully tested.

For example, special relativity reduces to Newton mechanics in the domain of velocities much less than the speed of light.

In philosophy, while new theories must comply with existing theories, there is clearly much less testing/retesting.

So, how does philosophy fit into this framework while maintaining the rigor required for scientific inquiry? Are there any examples of philosophical discussions that have significantly influenced or changed scientific theory?

Answer accepted (score 9)

  1. Philosophy isn’t concerned with empirically testable theories
In philosophy, while new theories must comply with existing theories, there is clearly much less [empirical] testing/retesting.

This observation is quite correct (leaving aside the remark about new/existing theories, see below).

However, the claim that philosophy doesn’t (shoudn’t) deal with scientific (empirical/mathematical) questions and methods is rather new, compared with what philosophers have been actually doing for most of philosophy’s history. This new claim appeared ca. 250 years ago, which is a rather short time period compared to philosophy’s 2500 years of existence. Up to German idealism it was customary for philosophers to be not only deeply informed about scientific matters, but to engage in scientific questions as well.

(To be sure, there were always battles within philosophy between rationalistic and empiristic inclined philosophers, and so Descartes would contribute to mathematics, while Newton would contribute to physics. But all these philosophers wanted to advance, in different ways, our actual knowledge of the world.)

One important symptom in the modern self-(re)imaging of philosophy as non-empirical was a debate occured in the 19th century about whether psychology, which was at the time a subdiscipline of philosophy, should use properly experimental methods in research. The faction supporting this view could succeed only at the cost of leaving philosophy (institutionally) in order to become an experimental discipline. The so called Psychologismus-Streit (psychologism debate) which followed sanctioned that logic and, most importantly, epistemology shouldn’t have anything to do with psychology, thus consolidating the claim that philosophy shouldn’t employ empirical methods and should deal with conceptual issues only. This is pretty much the status quo today.

There are, however, current attempts to return to the older view, i.e. that philosophy should employ experimental methods (check out experimental philosophy).

  1. Does philosophy meet scientific rigor?

In science: A theory must have observable consequences that can be tested and be falsified. Theories also must agree with previous theories in the domains where they had been successfully tested.
For example, special relativity reduces to Newton mechanics in the domain of velocities much less than the speed of light.
In philosophy, while new theories must comply with existing theories, there is clearly much less testing/retesting.

You seem to imply that in order to be called a science, a discipline has to produce empirically testable theories. This certainly isn’t an evident premise, as formal sciences do not meet this criterion. Are they therefore not scientific? This would seem an odd conclusion (and I don’t think you would like to draw it).

(Results of formal sciences are clearly used in constructing and testing scientific models dealing with observable reality, but these sciences are certainly not tested themselves this way.)

So, how does philosophy fit into this framework while maintaining the rigor required for scientific inquiry?

Well, when philosophy aims to maintain a certain rigor, it does so by adopting instruments used in the formal sciences (such as logic, probability theory, theoretical linguistics, etc.). But some branches of philosophy do not actually aim to emulate the epistemic values found in science, they orient themselves more at epistemic values found in other fields, such as literature. But still, they are certainly bounded to common standards of academic inquiry.

(As to your characterization of scientific theories, see the answer of Michael Dorfman’s reply. There is a common problem when speaking about “what science is” or “what scientific theories are”, as it is not clear if one means to give a (widely shared) definition or a factual description. Most of the time it is meant both as a definition and a description. Historical and sociological research showed that some widely shared definitions are simply false when used as factual descriptions of science - which still doesn’t invalidate these claims qua definitions!)

  1. Do philosophical discussion influence scientific theories?
Are there any examples of philosophical discussions that have significantly influenced or changed scientific theory?

Yes!

As you might see from the above, formal and empirical research was actually a part of philosophy for more than 2000 years. So the answer is: philosophy itself!

But probably you mean “science” in the sense of contemporary science, i.e. as a clearly differentiated set of disciplines with no general institutional ties to philosophy. Even in this case the answer is still: Yes!

As you mentioned Special Relativity (SRT), I’ll take this example to mention two cases in which philosophical discussions were involved in creating and defending/advancing SRT as well as General Relativity Theory (GRT).

  1. Ernst Mach formulated repeatedly the (now so called) Mach’s principle, which was seminal for Einstein to conceive GRT. (See this page for more info.)

  2. Since the Lorentz ether theory (LET) and SRT were deemed to be empirically equivalent, most scientists were puzzled as to which criteria could be used to solve this problem of underdetermination. Physicist Max von Laue consulted on this matter a still young Moritz Schlick, later to become the founder of the Vienna Circle, logical empiricism and thus of most of modern philosophy of science. Schlick gave a new interpretation of the principle of simplicity, showing why on these grounds SRT should be preferred (the old interpretation of simplicity actually supported LET). Together with Schlick, other soon-to-be logical empiricists, such as Hans Reichenbach, joined the discussion and formulated the first interpretations of SRT. They were also really important in defending SRT against scientifically and not-so-scientifically minded attacks. (Follow this link for detailed info on their contributions to SRT.)

If you need more (or more impressive?) examples, please let me now in the comments.

Answer 2 (score 4)

The short answer here is that philosophy is not concerned with testable hypothesis and controlled observation; those matters are left to science. Philosophy is concerned, rather, with questions that cannot be answered through this process.

Scientific theories also must agree with previous theories in the domains where they had been successfully tested.

This is not the case; sometimes new scientific theories directly contradict (and replace) previous theories. I suggest you take a look at Kuhn if the history of scientific theories is of interest.

Please give a specific example of a philosophical discussion that has significantly influenced or changed some scientific theory?

The domain of Logic is usually considered to be philosophy; all science piggybacks upon the foundation of classical logic and inference. In this regard, we can say that all Science, therefore, relies on a body of prior philosophical discussion.

Answer 3 (score 1)

Generally philosophy must help us to see something to the farthest extent that we can possibly achieve gradually by deepening the knowledge from any fields and to see where something to be placed on, so we can be guided to achieve something at best gradually.

Philosophy stands on any fields of knowledge to further deepening knowledge with the purpose to be able to see where ourselves should be placed correctly.

Science must be supporter for philosophy to be working properly, and vice versa.

To be more specific:

  • Science to Philosophy:

    • philosophy is working on reasonable thinking, and science can be a witness for correctness of an axiom from philosophy, and further philosophy will use this proven axiom with another axiom (that may not be proven yet), to gain another axioms, and
  • Philosophy to Science:

    • axioms from philosophy may be used by scientist to see any possible extent as hypotesis that may be followed by scientific observation.

If this kind of relationship can be constructed nicely, it will be a mutual relationship to both. Both will gain assertions, corrections and possibilities wider and wider (applicable knowledge) than before, from both (philosophy and science), one to another.

14: Are the first and second forms of the categorical imperative actually equivalent? (score 45653 in 2012)

Question

I will put aside any question of possible imprecision of the formulations, I will assume that the Kant’s intentions can be understood intuitively reasonably precisely.

Kant gives two forms of the categorical imperative:

  1. Behave in such a way that a reasonable generalization of your action to a universal rule will lead to a benefit to a generic person under this universal rule.
  2. Always treat others as ends and not means.

Kant then claims that 1 is equivalent to 2. Is this correct?

Suppose there were a class of people who liked to be ends. Suppose these people, under certain special circumstances, would like to be treated as objects, for example, as tables. They enjoy being tables, and have tablecloths and wine glasses put on their backs, it does them no harm, and they enjoy the experience, and talk about the experience with joy and regard.

Under these circumstances, knowing that you too might want to be a table at some point, would it be ok to treat these people as means and not ends, at least temporarily?

Is there a precise sense in which 1 and 2 are equivalent, as they seem completely different to me.

Perhaps the answer is that by respecting the wish to be means, not ends, you are treating the people as ends, not means. But then it becomes very difficult to actually determine when you are acting correctly according to imperative 2.

To give a more realistic precise examples, here are some things that are ok under 1 and not ok under 2:

  • purchasing blood plasma from a poor, willing donor.
  • lying to someone about something painful (like whether this person has cancer, or whether she is attractive in that dress, etc).
  • prostitution, dwarf tossing, and other superficially exploitative professions.

In general, I find that formulation 2 is the ethical intuition people are comfortable with (seeing as all three points above are objectionable to a lot of people), and the claim that formulation 2 is equivalent to formulation 1 I think is essentially unfounded.

Can someone who knows Kant better explain?

Answer accepted (score 6)

There’s quite a bit of scholarly debate concerning what Kant means when he insists on the unity of the various formulations of the categorical imperative; the general consensus seems to be that they all would generate the same duties.

It’s worth remembering that Kant does not say that we should only treat people as ends, and not as means—rather, we should not treat them as mere means.

The secondary literature on Kant’s moral system is enormous, and ever-growing; Derek Parfit’s On What Matters is perhaps the most significant and recent development. A good starting point, however, would probably be the relevant Stanford Encyclopedia of Philosophy article.

Answer 2 (score 3)

I think there is a point or two to pull out of the original question and supporting expansion of the question. First, you state, ‘suppose there are people who like…’ If we take part 1 of Kant and apply a universal rule evaluation, we could say it is good to give people what they like. So satisfying part 1 eliminates the negative of part 2. In other words you are not treating those people like ‘means’, your treating them like ends. Second, ‘treating someone’ like a mean versus and end is a rather complicated by the more imprtant fact of allowing the other party to participate in the decision.

Sadly, I am not more well schooled in Kant, but I do enjoy pondering his universal rule.

Answer 3 (score 0)

As to the example about people who like to be tables - well, as this is their life choice, then treating them as tables is exactly who and what they are by their own admission, therefore we are doing what they want and like….. there would be no real gain for us, except perhaps the convenience of having an extra table to hand, but really the gain is for them.

So we are not treating them as ‘means’. We are in fact behaving in a way that promotes their desired goals, i.e. to be ‘used’ as tables, thus we are meeting their ends.

Moreover, it would fit with the first CI, that the action must be one to which universality can be applied, in that we all have ways in which we like to be treated, even if it is not to be used as a table! Thus we can claim that we are behaving in a way that satisfied the way in which one group of people prefers to be treated, which in itself is subsumed under the universal rule that we take note of everyone’s preference/s.

15: What is the difference between philosophy and religion? (score 44904 in 2016)

Question

How do you make the distinction between philosophy and religion? Are there some philosophies/religions that are hard to categorize as being one or the other?

Answer accepted (score 16)

Taking the latter part first: yes, there are definitely religions/philosophies which have proven difficult to categorize. The government of Australia, if I understand correctly, is currently trying to decide if Buddhism qualifies as a religion; the government of the US has decided that Scientology counts as a religion for tax purposes, while the government of Germany has decided the opposite. In Norway, the second-largest “faith/life-view” organization (after the state Lutheran church) is the Humanist-Ethical Society (i.e., a group of atheists) which qualifies as a “religion” for many purposes (including tax status.)

The fact that this confusion exists in practice shows that there isn’t any clear, reliable indicator one can point to. The obvious candidates (such as the belief in a deity, or a soteriological path, or the presence of rituals or dogma) break down when you actually try to apply them in practice: exceptions abound.

So, that being the case, I would turn the question around: why does it matter? What benefit would be gained by drawing such a distinction? If we could answer that, perhaps we’d have a clearer way to finding an appropriate criterion.

Answer 2 (score 13)

The terms are not synonyms. You should note that religion always refers to a specific set of beliefs, i.e. Christianity, Islam, Buddhism, etc. The term philosophy can be used in two senses: In the general sense it refers to:

the rational investigation of the truths and principles of being, knowledge, or conduct. (source)

In a narrower sense, you can use the word to refer to a specific set of philosophical theories, such as stoic philosophy or Kantian philosophy. It would be these narrower subsets of philosophy which might be considered parallel to a particular set of religious beliefs.

With that cleared up…

How do you make the distinction between philosophy and religion?

Answer: philosophy in general is the rational investigation of truth, whereas religion often makes the same kind of truth claims but doesn’t claim to base it on reason or rationality, but instead it is based on other things like faith. The key difference is that they are different epistemological positions — philosophy has a system of logical principles in place to arrive at conclusions whereas many religions (such as Christianity) allow for other sources of knowledge (i.e. faith).

Are there some philosophies/religions that are hard to categorize as being one or the other?

Answer: In principle, it could be a problem to categorize them but in practice it is not very difficult. Why? Because virtually all religions with even a modest following carry with it a set of traditions and rituals which philosophies do not. Put simply, religions have practices and philosophies do not. Theoretically, you could have what might be considered a religion without any practices, but it’s not very common.

Answer 3 (score 9)

Thomas Aquinas makes the distinction that philosophy is based on human reason alone, but religion also includes some kind of divine revelation.

16: What will happen if Pinocchio says: My nose will grow (score 42987 in )

Question

  • If his nose is not growing, he is telling a lie and his nose will grow but then he is telling the truth and it can’t happen.
  • If his nose is growing, he is telling the truth, so it can’t happen.
  • If his nose will grow, he will be telling the truth, but his nose grows if he lies so it can’t happen.
  • If his nose will not grow, he is lying and it will grow but then he would be telling the truth so it can’t happen.

What will happen? The universe will explode?

Answer accepted (score 4)

That looks like a paradox, but it isn’t.

The assertion is so vague that it is true even if Pinocchio’s nose doesn’t grow immediately.

Even if he says: “My nose will grow immediately” that’s a vague assertion, when does “immediately” start and end as a time lapse?

If he is more specific and states: “My nose will grow in the next five minutes” then the nose can make a suspension of disbelief, if he tells a lie in those five minutes then the nose will grow and the first assertion will be true, if he doesn’t then the first assertion will be proved as false and if falsehood is considered as a lie then the nose will grow after the five minutes.

Everything that is false is a lie? That depends on some definitions, but if Pinocchio says something that is false and a mistake then the nose may (or may not) consider that Pinocchio is implicitly stating to be sure about something that he is not sure and actually wrong, hence a lie. That may be a question for Geppetto.

Nothing is true or false until it is.

If Pinocchio succeeds in creating a paradox (either the liar’s paradox or some other paradox) then the nose will not react to that. A paradox is something that cannot be true and cannot be false and the nose grows on falsehood, right?

No matter which logic and assumptions (OWA and CWA for instance) that nose follows, in any logic where there can be paradoxes it is clear that the middle cannot be excluded.

Answer 2 (score 1)

The premise that Pinocchio’s nose grows if and only if he claims any falsehood is given as:

∀x: G ↔ C(x) ∧ ¬x

If Pinocchio’s claim is that his nose will grow then this is given as:

G ↔ C(G) ∧ ¬G

This is a contradiction. Therefore the premise that Pinocchio’s nose grows if and only if he claims any falsehood is necessarily false:

∃x: ¬(G ↔ C(x) ∧ ¬x)

Answer 3 (score 0)

Bad question. Stated differently, you have constructed a paradoxical world. Check out self-reference. I will take your question to be equivalent to the liar paradox:

This sentence is not true.

Something tricky is going on here: recursion:

This sentence is not true.

Let me do a substitution, “This sentence” → “This sentence is not true.”:

“This sentence is not true.” is not true.

But wait, what is being pointed to by “This sentence”? The substitution did not bring any clarity! The problem here is that there is an infinite regress, caused by self-reference. This kind of paradox shows up elsewhere, like Russell’s paradox, which attempts to construct the set R, with membership criterion: “all sets which do not contain themselves as a member”.

  1. If R contains itself, it is not a member of R.
  2. If R does not contain itself, it is a member of R.

This might take a while to completely wrap your head around, but it is a deep result of what is now called naive set theory. In order to circumvent this problem, axiomatic set theories where developed which could not produce such paradoxes. However, they lose something: they are axiomatic, and thus not ambiguous like natural language. And yet, there seems to be something deep to this ambiguity. I won’t go into that now, but it would make a good separate question.

There is a theory of computation aspect to self-reference, which shows up as Turing machines being able to print out their own description. This gets at the idea of self knowledge. And yet, Thomas Breuer’s The Impossibility of Accurate State Self-Measurements questions this whole perfect self-knowledge enterprise. This Turing machine self-reference thing is very important; it shows up in the Halting Problem, which presents a huge obstacle to provability of Turing complete systems, which means we cannot guarantee properties we’d like to guarantee (like that your phone won’t crash).

Douglas Hofstadter introduced the idea of strange loops in his Gödel, Escher Bach. The book is a layman’s introduction to some neat theory of computation issues. I do not pretend to understand this ‘strange loop’ idea, but it definitely has to do with self-reference. It may be that consciousness itself has to do with self-reference; indeed, it is hard not to. So there’s a lot to this liar paradox!

17: What does Einstein’s quote “If the facts don’t fit the theory, change the facts” mean? (score 42346 in 2013)

Question

What did Einstein really mean by saying:

If the facts don’t fit the theory, change the facts.

Answer accepted (score 3)

Apparently Einstein recognized the human tendency of allowing pride to overrule honesty. When the facts prove that your theory is wrong, change the facts so you can continue to have the admiration of your peers and cronies, and in many cases to acquire the funds necessary to continue your work. Hats off to Einstein for his insight.

18: Is ‘guns don’t kill people, people kill people’ a good argument? (score 42226 in 2018)

Question

I’m hearing the argument X doesn't do Y people do Y in quite a few guises. For instance in it’s original form

guns don’t kill people; people kill people

Presumably, therefore guns are OK

cars don’t kill people; people kill people

Again, the inference is that fast cars are OK

And more recently on the BBC business news

Securitising debt doesn’t destablise financial institutions; people destablise financial institutions

Again leading implicitly to … these kind of financial instruments are perfectly fine so lets start using them again.

Instinctively these kind of arguments feel invalid. Can anyone perhaps use some more rigorous or formal type (logical?) analysis to demonstrate their validity/invalidity. Is that even possible?

What I am interested in is an analysis of this particular form of argument using the tools of philosophy. When it was just used by the gun lobby it just seemed like a rhetorical tool. But it’s now bleeding over to other areas so I’m interested in the form of that argument and it’s validity. It’s almost incidental that it’s gun control. As an aside there is a really good podcast about the philosophy of gun control if anyone is interested in those particular issues.

Answer accepted (score 232)

This line of argument is basically a strawman fallacy, which is when you construct a weaker version of your opponent’s argument in order to then disprove it. In this case the weaker argument is that “guns kill people” all by themselves. No one actually believes that or argues that. Even the most committed gun control advocate acknowledges that firing a gun takes human intervention. The stronger argument being concealed by the strawman is “people with guns kill more people than do people without guns.”

It’s also a black or white fallacy in that it is an attempt to frame the debate as two mutually exclusive choices, when other options may exist. You’ll often find these two fallacies together, where the debater creates an artificially weak version of your position, and then tries to frame it as the only alternative to his or her position.

IMPORTANT: The fact that this particular argument is fallacious does not mean we can therefore be justified in rejecting the conclusion the argument is intended to support (that in itself would be an argument from fallacy error), it just means we cannot take the argument as providing any actual support for the conclusion.

Answer 2 (score 43)

To complement Chris’ answer I’ll try to deconstruct some of the reasoning in the arguments a bit.

Suppose we claim that “guns don’t kill people; people kill people.” The only reasonable way to parse this into a slightly more formal statement is:

Guns are not responsible for killing people; people are responsible for killing people.

I can think of perhaps one other way to interpret the sentence, which would be:

Guns do not cause people to die; people cause people to die.

But this second formulation is patently false - if people cause people to die [using guns] (which they do), then the guns they use cause people to die via bullets, which cause people to die via massive brain damage/haemorrhaging/etc. The only way we can claim that people cause other people to die without granting that guns cause other people to die is by rejecting transitivity of causation: that “A causes B” and “B causes C” does not imply “A causes C.” Certainly there are some philosophers (especially within the free will debate, e.g. Carolina Sartorio) who reject transitivity, but definitely not within this context.

Here, the only way the second statement can be true is if “PEOPLE cause GUNS” and “GUNS cause BULLETS” and “BULLETS cause DEATH” (some details left out, obviously), and from this we conclude “PEOPLE cause DEATH” but not “GUNS cause DEATH.” This is a very strange thing to do - the only way we could conclude that people cause death is if we trace the process of causation through guns. Then we’d be forced to agree that guns, too, cause death. The only way out of this is some extremely arbitrary rule somehow excluding guns from being a cause of death while still being used to prove that humans cause death.

Note that it is not a valid response to leave guns out of the causal chain altogether - it is undeniable that guns cause bullets to fly into living bodies and turn them into corpses.

Thus we must return to the first interpretation, one of responsibility or agency. Simply, the argument is that guns don’t kill people because they are not agents which can be held responsible for their actions. One does not charge a pistol with murder; one charges the human who used it to commit the homicide.

This is a fair enough claim, but as Chris points out it is fallacious in being used to arrive at any sort of conclusion. It indeed constructs a strawman which claims those in favour of gun control somehow hold guns responsible for murder. Of course this is not in general true - if one uses firearms murder statistics as evidence in favour of gun control, they are roughly making the claim that the humans responsible for murder have access to weapons that can easily cause death. Then, by the only reasonable reading, “guns don’t kill people, people kill people” is technically true but wholly irrelevant to the argument being made by gun control advocates.

Answer 3 (score 16)

“Guns don’t kill people; people kill people” is not an argument, it’s a slogan. It may be the case that this slogan is just a way to get people to discuss the role of individual responsibility in what policies the government ought to adopt with respect to guns or something like that. Or perhaps it is just a signal that a person has some particular position that can be described in more detail if others are interested. I think interpreting it as an argument in its own right is not a good idea.

19: , it is precisely facts that do not exist, only interpretations… As translated in The Portable Nietzsche (1954) by Walter Kaufmann, p. 458 (score 42089 in 2018)

Question

I came across this philosophical thought.

There are no facts, only interpretations

written by Friedrich Nietzsche (1844-1900).

I tried to understand it but cannot get a satisfying answer. What does he mean with this quote?

Answer accepted (score 18)

An important thing to keep in mind when reading Nietzsche is that most of the time he is trying to reveal things through insights.

The point of this particular quotation is to reveal the assumption at the base of many philosophies (in this case, most specifically positivism): that objective facts exist.

Positivism holds, roughly, that the phenomena we observe through our senses are physical in nature and that they actually happen in a material world. Thus positivists take these phenomena as objective fact and use it for their world-explanation, for example by making physical laws. Nietzsche’s statement is that fundamentally, positivists are interpreting observed phenomena as physical (instead of non-physical, e.g. Berkeley), and real, when in fact they have no definite justification to do so. Thus, facts are really the subjective result of information: there is nothing necessarily “true” about them, other than how they fit into a particular interpretation.

Just as the interpretation of a book is up to the reader, so too is Nietzsche pointing out that the interpretation of our world is up to the person observing. Now, here he does not make any claim as to whether, as with a book, there is an “author” who intended a particular meaning that we are supposed to pick up on, and that this meaning is the “correct” interpretation, but holistically that idea would probably be repugnant to Nietzsche: he effectively rejects all teleology, metaphysics, and meaningful notion of truth, which leaves no room for there to be a “correct” interpretation of the world.

Ultimately what this means is that, to Nietzsche, it is much less important whether we “understand” how the world “works” (if that is even a meaningful thing to say), and much more important that we develop an interpretation that works for us. One of Nietzsche’s primary themes is the advance of humanity into a stronger (intellectually, culturally, and maybe physically) race, and the proper interpretation of the world could aid in achieving this. In Nietzsche’s terms (and this is straying from this quotation against positivism to a generalized summary of Nietzsche), this would be the interpretation that best frees us from life-rejecting restraints such as Good vs Evil and lets us become stronger by exercising our will to power and working toward the overman.

Answer 2 (score 4)

The quote gave me some problems as well, but as I understand it, Nietzsche is here taking a very basic position in contemporary philosophy. Regardless of what facts exist “out there”, what we always have and can only have are interpretations of them in our mind. There is no presumed identity between the contents of our mind and the facts out there, and in fact we have no direct access to facts under this strict definition. This does not mean that we cannot do science, or that we must become antirealists. It just means that the principles whereby we are realists; consistency, stability, facticity etc… are necessarily our interpretations as humans, as cultures, etc…

Answer 3 (score 4)

The quote gave me some problems as well, but as I understand it, Nietzsche is here taking a very basic position in contemporary philosophy. Regardless of what facts exist “out there”, what we always have and can only have are interpretations of them in our mind. There is no presumed identity between the contents of our mind and the facts out there, and in fact we have no direct access to facts under this strict definition. This does not mean that we cannot do science, or that we must become antirealists. It just means that the principles whereby we are realists; consistency, stability, facticity etc… are necessarily our interpretations as humans, as cultures, etc…

20: How can one not believe in god as the root cause of the universe? (score 41752 in 2018)

Question

How can you lack belief in the existence of god?

I define god here as prime cause. As the world is a sum of collections of events, causally linked to the past through time, then there must be a prime cause.

As of now the Big Bang singularity has been discovered, but to say this Big Bang occured in nothingness, where there is no volume, no time, no energy, completely nothing…isn’t it a bit far fetched? There ought to be cause(s) to this singularity, and cause(s) to that cause(s). In the end it should still lead to god.

Answer accepted (score 220)

The alternative between existence and non-existence of a creator god cannot be decided by the argument of the first cause.

Whoever argues that a first cause is needed and that this first cause is god, has to answer the question:

What is the cause of the creator god?

Answer 2 (score 174)

The most compelling argument I’ve heard in this vein is that the existence of God just adds an extra step.

As Jo Wehler has pointed out, claiming God is the first cause raises the question: “What is the cause of the creator god?”

The most common response I’ve heard is that God requires no first cause; that’s part of what makes God God. However, this raises another question: if you’re willing to accept that there exists something which does not require a first cause, why could that something not just be the universe?

In other words, there are two possible ways the universe came to be:

  1. God simply was, requiring no first cause, and created the universe.

  2. The universe simply was, requiring no first cause.

Many non-believers look at these two possibilities, and by Occam’s Razor choose the latter. To them, there is no reason God is necessary to solve the issue of a first cause.

Answer 3 (score 109)

God here I defined as prime cause.

If you simply define god as the prime cause, then that is simply word-play. You obviously understand that the vast majority of people do not use the word ‘god’ simply as the definition of the first cause. They attach much more meaning to the word. The vast majority of people who believe in ‘a god’ believe in some being, which–currently or in human history–has had direct effects on the world. Your definition of god comes closest to the deistic version of god, although even deists believe that god exists (whatever that means. By the way, question: does the “first cause”, which you define as god, exist?).

When you ask

How can people disbelieve in god?

It is very loaded question (or at worst, a very dishonest question) because you use the word ‘god’ much differently than what people ordinarily understand the word to mean.

If I asked you

How can people disbelieve in unicorns?

Then you might respond with something like “because there is no positive evidence that unicorns exist”, and then I respond with “I define unicorns as being horses.” Of course, by my definition of the word, most people actually believe in ‘unicorns’, but it would be silly of me to expect people to go around and say that they believe in unicorns, because my definition does not match the common usage of the word.

Why not simply skip the word ‘god’, and be much clearer and simply say “first (or primary) cause”?

Lastly, there is a bigger philosophical issue at hand. That is that we simply do not know if there is such a thing as the “first cause”. Our understanding of the physical world basically breaks down near the creation of the (observable) universe, and our intuitions are often completely wrong about how physics work at the quantum level. Therefore, the honest thing to say is simply that we don’t know.

21: In which way does quantum mechanics disprove determinism? (score 41175 in )

Question

I’ve heard this pop up in a discussion with my physicist/engineer roommates, but didn’t care to ask at the time. Now I’m mighty curious about it. Wikipedia doesn’t really seem to say much on this issue.

From what I understand about the Uncertainty Principle, it says that there are certain properties of electrons and stuff that cannot be measured, and are therefore uncertain. Then Wikipedia (under indeterminism) states that Sir Arthur Eddington says that the Uncertainty Principle isn’t really so because we can’t measure these properties, but because turns out nature is indeterministic. At least that’s what I took from those paragraphs. Even without my biased wording, it sounds more like an assertion than evidence.

I’ve also read a few things about how other scientific conventions perceive the issue, like how a ball on the peak of a perfect mound might randomly roll down in any direction, and I’m still unconvinced. My belief of determinism is generally that if you knew every single variable that existed as a factor at the very beginning and birth of the universe, you could correctly determine all properties of any individual particle at any point in time.

Could anyone provide some more background about this? Especially regarding quantum mechanics?

Answer accepted (score 31)

I thought I would give a physicist’s perspective here.

There are two types of evolutions in quantum mechanics: unitary (or free) evolution and measurement. Free evolution is fully reversible and deterministic; a given operator takes a specific wave functions and maps it to a specific other wave function. The uncertainty comes from the non-unitary measurement evolution.

Unfortunately, if you want to approach this problem from a realist point of view (how most people think of classical mechanics, etc) it becomes difficult to solve the measurement problem: i.e. what constitutes a measurement, where is the system and where is the measurement device? Isn’t the measurement device + orginal system just a bigger system that should be undergoing unitary transformations? This question has puzzled many, with some notable scientists even linking measurement to the acts of conscious observers. But this is not a standard view.

Most researchers on the foundations of quantum mechanics, however, usually side-step this question by taking the operationalist point of view. Tagline: “all we have is some procedures for setting up an experiment and the results of experiments”. In this framework, you can derive Bell’s theorem, which says that any phenomenon that is both deterministic and local must satisfy the Bell inequality. Quantum mechanics violates the Bell inequality (and there have been many experiments that mostly confirm this violation, there are some technical loopholes that need to be addressed in some of the experiments). This means that you must give up at least one: locality or determinism. Since without locality it becomes impossible to talk about causality, most people prefer not to give it up, and instead give up determinism.

Answer 2 (score 27)

Quantum Physics doesn’t disprove determinism.

What Quantum Physics does do is significantly complicate the task of arguing for determinism.

Put in the simplest possible terms, the Uncertainty Principle indicates that: 1) our observation of an event has a significant effect on the event, and 2) it is impossible for a single observation to observe all relevant properties of an event. This means that any argument for determinism can no longer have simple recourse to the notion of observation.

So, when you say:

My belief of determinism is generally that if you knew every single variable that existed as a factor at the very beginning and birth of the universe, you could correctly determine all properties of any individual particle at any point in time.

you instantly run into trouble, because we can’t know every single variable that existed as a factor at any point in time (including the initial state) through any type of observation.

Answer 3 (score 10)

The Uncertainty Principle is not directly problematic for determinism; it just says you can’t measure your states that accurately. You could always assume that the states were there, but you just couldn’t measure them. Einstein preferred this view, and together with Podolsky and Rosen devised a paradox that would show that uncertainty is not fundamental. Unfortunately for Einstein, the experiments delivered the seemingly paradoxical result, showing that uncertainty is fundamental and determinism, if true, is not local. (Actually, it even shows that causality is not local.)

But the more telling blow to determinism is the success of entangled/superimposed states that are stochastically collapsed under certain conditions. The double-slit experiment is the most famous of these, but it’s really Bell’s inequality and experiments (that failed) to confirm it that made determinism look like a bad model of reality. The experiments are too technical and detailed to describe here, but so far Bell’s inequality has been routinely violated, and hence, there is no room for a deterministic model where the relevant state stored locally. (Of course, a computer simulation with all state stored globally can reproduce anything, in principle, but that doesn’t make it a parsimonious way to explain results in physics.)

22: What logical fallacy is “If you don’t like it, move!”? (score 37819 in 2018)

Question

When criticizing government, society or whatever, people often retort, “If you don’t like America, why don’t you move somewhere else?”

What kind of fallacy would this qualify as?

At first glance, it doesn’t appear to make an argument at all. However, there is an implied argument:

If one doesn’t like (whatever), one should move (rather than trying to fix the problem).

Can anyone suggest what kind of fallacy this is?


P.S. For anyone looking for a clever rebuttal, this is mine:

Because I’m not the one with the problem; why don’t YOU move?

Answer accepted (score 141)

It may very well be a poor argument, but it’s not a logical fallacy

People are too quick to jump on the “fallacy” bandwagon. There is no logical fallacy occurring here. It may very well be an argument that is not particularly convincing (In fact, I wouldn’t use the argument), but there is nothing logically fallacious about it.

If a person asks

“If you don’t like America, why don’t you move somewhere else?”

Bread states that “there are other options your opponents refuse to acknowledge,” however this seems somewhat of a stretch. This is clearly reading more into what was said, than what was actually being said (In fact, this appears to be a straw man argument, which is ironically a fallacy itself). From this phrasing there is nothing that says that moving is the only way to achieve change. It just raises moving as an option. Clearly, this is an option that many people take, because people migrate all the time to countries they prefer to live in. In some cases it may even be the preferred way to achieve the change that you wish in your life.

Let’s take a similar example. Consider the following statement:

“If you want to earn more money, why don’t you work more hours per week?”

Would anyone seriously think that this person is suggesting that working more hours per week is the only possible way to earn more money? I think that most people can see that they are simply raising one possible avenue of earning more money.

Instead, if someone said

“If you don’t like America, your only option is to move somewhere else.”

Then, sure, their statement would be logically fallacious.

On the other hand, Mark Andrews analyzes the statement as

“There is nothing seriously wrong with this society, so the problem must lie in your own attitudes.” That is the conclusion. When the proponent recommends that the other person leave the country, the validity of this conclusion is taken as a given. The question of the truth or falsehood of the conclusion (which has become the assumption) is sidestepped completely.

But again, there is nothing fallacious in thinking “I like the country as it is; I would prefer if you didn’t change it; therefore I would prefer that you leave rather than changing the country.” Again, I’m not telling you to be convinced by the argument. But there is really nothing logically fallacious going on. To the question “Why don’t you move somewhere else?” You are completely free to respond “Because I would rather change [what I perceive to be] the flaws of this country rather than moving.”

Answer 2 (score 125)

It is the False Dilemma or Bifurcation Fallacy.

If you don’t like it, then move.

Let’s say you have caused some problems by questioning the decisions, speech, and actions of someone who is actively seeking social, economic, and/or political power. And you’re confident that you have every right – or even duty – to do so. Those in power who are profiting from the situation aren’t willing to change anything, hence they feel threatened by any kind of criticism. So they choose to solve the problem by employing a logically twisted rhetorical tactic against you in order to get rid of you fast. They assume that you aren’t intelligent or educated enough to catch the fallacy in their argument. It goes like this:

  • Either you agree with the way we’re handling things, or you don’t.
  • If you like it, you’re no threat to us (you can stay).
  • But your dissent creates a dilemma for us (so you can’t stay). “Move.”

The point is, they offer no other options. Just two: agree and stay, or disagree and leave. They’re not willing to discuss it with you, because they know they’ll lose that argument, so they’re going to insist that you go away. But since they don’t want to argue with you, they want you to think it’s your choice. Thus they create a false dilemma.

“It’s too bad you don’t like it, because that means you should leave.”

Well, not really. It doesn’t have to mean that you should leave, because there are other options your opponents refuse to acknowledge. You might prefer to remain in the land of your forefathers, the land that you love, and work diligently to improve the socio-political climate where you are, along with any other important problems you and your loved ones might be facing there.

Logical Fallacies explains it well:

The bifurcation fallacy is committed when a false dilemma is presented, i.e. when someone is asked to choose between two options when there is at least one other option available. Of course, arguments that restrict the options to more than two but less than there really are, are similarly fallacious.

Example:

    1. Either a Creator brought the universe into existence, or the universe came into existence out of nothing.
      1. The universe didn’t come into existence out of nothing (because nothing comes from nothing). Therefore:
        1. A Creator brought the universe into existence.
The first premise of this argument presents a false dilemma; it might be thought that the universe neither was brought into existence by a Creator nor came into existence out of nothing, because it existed from eternity.

Texas State University, Department of Philosophy, under ‘False Dilemma’ (an informal fallacy), lists as the first example:

America: Love it or leave it.

Answer 3 (score 51)

There is no argument, therefore there can be no fallacy

There is no fallacy here, no logical error in argument as e.g. in affirming the consequent. This is so because there is no argument here at all - only an expression of viewpoint. Fallacy presupposes argument. No argument, no fallacy : the concepts are tied.

23: What is the difference between Philosophy and Theology? (score 37670 in 2015)

Question

In attempting to wrap my mind around the basic vocabulary, concepts, and methods of philosophy, I find myself wondering what the difference is between a philosopher and a theologian.

Theology (link to definition in Wikipedia) can have two meanings:
1. Theology is a rational study of the existence of God/gods and the nature of religious ideas.
2. Theology is simply a study of a particular religion (or all religions), really more the practice than the theory, but maybe a mixture.

It seems from an immediate reading that definition 1 would lead to the conclusion that a theologian is a philosopher (perhaps that theology is a branch or subset of philosophy), but definition 2 appears to be rather unphilosophical in nature.


So, how does philosophy answer this question?

  1. Is theology considered a proper subset/branch of philosophy overall?
  2. Does it depend entirely on which of the above 2 definitions of theology fits a particular theologian as to whether he is a philosopher or not?

I think this answer is simple and fairly self-evident, but somewhere in my gut I feel like I’m missing something. Is there a treatment of this question in the literature?


Edit: the answers so far seem to want to redefine theology or other terms - that is fine, but please be clear that you are redfining them, and provide, if possible, sources for why the term should be different than given (support from historical works, other sites, etc.).

Answer accepted (score 7)

As Nelson Alexander mentions, philosophy encompasses far more than just philosophy of religion, but I assume that’s what you mean when comparing the two.

Theology and Philosophy (of Religion) are two different disciplines, that none the less have historically had significant overlap. For a long time it was hard to distinguish the two. In India, philosophy was discussed within the context of Hindu and Buddhists beliefs, and in the West it was taken for granted that theologians such as Augustine of Hippo were also philosophers. Since Philosophy is described as being “the love of wisdom”, any form of knowledge was considered a branch of philosophy. Hence philosophy also encompassed math, biology, physics, etc…For this reason, theology was viewed as ‘naturally’ being part of philosophy, and all theologians were philosophers by definition.

There was also a sociological reason, in the fact that often the only (or most) literate people in a given society were priests and monks, and so there was a strong selection bias, in that the people who practiced philosophy were also the most religiously inclined.

The two start to be viewed as separate first in the Islamic sphere with Al-Ghazali (circa 1100), who wrote “The Incoherence of the Philosophers” arguing that the rules of logic and reason can not be applied to matters of faith and belief. For Al-Ghazali any attempts to do so would inevitably lead to incoherence, and eventually atheism. He doesn’t want to become an atheist, so instead he concludes that theology has to remain separate from all other sciences. This was later picked up in the West during the enlightenment, for example in Kant’s analysis of arguments for God’s existence, or more dramatically as viewed by Hume in his famous fork. The invention of the printing press also helped, with the fact that more and more laymen were able to read, leading to the gradual increase in secular scholarship.

Despite the historical overlap, a clear distinction can be made between the two:

  • Theology starts from a position of absolute certainty. A certain number of facts about God (the ‘theos’ in theology) and revelation are taken for granted, and the theologians task is then to analyze and elaborate the consequences of those facts. A theologist might use philosophical arguments and methods, but he/she is applying them to a set of presuppositions about God and the world. In this sense the relationship between theology and philosophy is similar to the relationship between physics and math or economics and math. A perfect example of this are the various approaches (for example by theologians such as Augustine or Aquinas) to solving the problem of evil: It is a fact that a) God is omnipotent, omniscient, and omnibenevolent and b) that there is evil in the world. A theologian then tries to reconcile the two facts, and might (or might not) use tools from Philosophy in doing so. A “true” philosopher on the other hand, would, given the problem of evil, simply dismiss (one or all of) the initial premises about God, instead of trying to reconcile them with the existence of evil in the world.

  • Philosophy has to start from a position of radical skepticism, even if it eventually arrives at a position of knowledge or certainty. Consider how Socrates (or Plato) in the Meno discusses virtue, but starting from a position that he doesn’t know what virtue is. This is probably why “true” philosophy is considered to have started with the Greeks. The classical Greeks were the first to start from positions of questioning all existing assumptions, especially religious ones. Another good example is Descartes: Although he ultimately arrives at positions that are in perfect accord with religious doctrine (i.e. that souls and God exist), he only does so after starting from a position of total doubt. Compared to solutions to the problem of evil, arguments for God’s existence are “inherently” philosophical, since they have to start from questioning basic assumptions.

Despite this clear distinction, even today people still tend to confuse the two terms, and I’ve seen several sources use “Philosophy of Religion” and “Theology” interchangeably. Again a certain selection bias applies here: People who study philosophy of religion are also more religiously inclined (a credit to Virmaior for this link). This is probably why many sources (including wikipedia), still include the problem of evil under the heading of philosophy of religion, when it should be properly included under the heading of theology.

Finally, an interesting case is that of Apologetics. Apologetics gets a bad rep because of its association with the Christian fundamentalist and Creationist crowds and explicitly Christian educational institutions (at least here in the US), but it is actually starting from a position that is more sound than that of mainstream theology (as taught in mainstream religious studies and divinities departments). Apologists do concede that while they believe, their presumed target audience has good reason to doubt religious dogma, and then proceed to try to convince them of their views. This strikes me as a more philosophical approach than theology qua theology, even if comes from dubious sources.

Answer 2 (score 5)

Aristotle when writing his First Philosophy or Philosophy of Nature wrote on the First-Mover; it’s his definition of God.

Plato in the Republic, says that the aim of philosophy is the knowledge of the Forms; and the highest Form is the Good, which shining forth makes all things visible in their true sense - this is his theology.

Spinoza, who I took to be a rationalist in the secular sense, begins with the neccessary Beings, and show that there can only be one - again this is his definition of God.

Philosophical theology is distinct from Western religous theology; where rituals and revelations are important, as well as a pantheon of saints or sages; but the distinction shouldn’t be taken as some essential difference, but a porous or dialectical difference.

Answer 3 (score 4)

Theology is different from philosophy. No one is a subcase of the other.

Theology is the systematic explanation of a certain religion. Hence theology always lives inside the scope predefined by the religion in question. Theology employs philosophical methods like hermeneutics but also historical studies. Theology in general presupposes belief in the religion under study. Hence theology is not open-ended as philosophical investigations are.

On has to discern theology from the discipline of Religious Studies. The latter studies the phenomen of religion itself, without giving precedence to one distinguished religion. Religious studies employ results from sociology and psychology.

Philosophy is a cultural enterprise to gain knowledge about our world simply by thinking and to argue for these thoughts. Philosophy can be a purely secular enterprise.

Added due to one comment of @LightCC:

Referring to theology I take the definition implicit in “Thomas Aquinas: Summa Theologiae”. He uses the Latin terms sacra doctrina and sacra scriptura as synonyms for theology. The first question of the first part deals with the characteristics of sacra doctrina. In part I, q.1 a3 Thomas says, see e.g., http://www.gutenberg.org/cache/epub/17611/pg17611-images.html:

Therefore, because Sacred Scripture considers things precisely under the formality of being divinely revealed, whatever has been divinely revealed possesses the one precise formality of the object of this science; and therefore is included under sacred doctrine as under one science.

Because Thomas presupposes the divine revelation as starting point of theology, the result of theology cannot be open-ended.

I am aware that quoting Thomas Aquinas speaks for Christian theology only.

24: What are prominent attacks of Rawls’ “veil of ignorance” argument? Which liberal philosophers have advanced it? (score 36867 in 2012)

Question

In John Rawls’ A Theory of Justice, he argues that morally, society should be constructed politically as if we were all behind a veil of ignorance; that is, the rules and precepts of society should be constructed as if we had no a priori knowledge of our future wealth, talents, and social status, and could be placed in any other person’s societal position. Just give an easy example, rule by tyranny would be an unjust society, because doubtless no one would agree a proiri to governance by tyrant if he were not one himself.

I’ve never accepted this argument. First of all, I just don’t believe people are exchangeable in this fashion, because of hereditarian considerations; the exchanging of places before hand would not, in many cases, would not lead to a significant “shake-up” of society, if meritocracy is truly operating — so considering things with a veil seems needless. Secondly, using the veil to argue for distributive justice and egalitarianism, as Rawls does, in my opinion seems to presume that moral virtue is orthogonal to societal position, so that it is only “fair” that we “start off on the same foot”; I don’t agree with that either, because I think the poor, at least in America, are somewhat less virtuous than middle America or the rich, and that a moral accounting behind this veil would in any case send these lacking to the same positions they occupy.

But, alas, I’m a naif in philosophy, having never studied it seriously. So I have two questions: Are there any prominent attacks on Rawls’ position along these lines, and secondly, if so, have any liberal philosophers updated Rawls’ argument to deal with positions from hereditariainism and so on?

Answer accepted (score 13)

The classic answers to Rawls’s work come from his fellow Harvard professor, Robert Nozick. In particular, Nozick’s seminal work entitled Anarchy, State, and Utopia (1974). It’s written as an almost direct critique of Rawls’s Theory of Justice, published a few years prior in 1971. In it, Nozick adopts a libertarian approach to justice to challenge Rawls’s Second Principle of Justice.

Among other things, Nozick’s most easily understandable argument boils down to the point that property rights must be included within Rawls’s notion of individual rights; that is, the individualist right of and to self-ownership. But once we include that right, we arrive at a subtle contradiction. In order for Rawls’s theory to make sense, he must reject the conception of absolute property rights; but at the same time, at least in Nozick’s view, the absolute right to property is one of the individual rights that must be protected.

But personally, I’d say the best attacks against Rawls are those that fundamentally question the notion of social justice at its core, i.e., F. A. Hayek. As a member of the Austrian School, Hayek is probably most famous for his work on economics. But mixed in with the economics is a lot of fascinating treatment of social and institutional justice. His aptly-named book, The Mirage of Social Justice, is probably the best place to start researching such a critique. A few gems (emphasis added):

Though we are in this case less ready to admit it, our complaints about the outcome of the market as unjust do not really assert that somebody has been unjust; and there is no answer to the question of who has been unjust. Society has simply become the new deity to which we complain and clamour for redress if it does not fulfil [sic] the expectations it has created. There is no individual and no cooperating group of people against which the sufferer would have a just complaint, and there are no conceivable rules of just individual conduct which would at the same time secure a functioning order and prevent such disappointments.

The only blame implicit in those complaints is that we tolerate a system in which each is allowed to choose his occupation and therefore nobody can have the power and the duty to see that the results correspond to our wishes. For in such a system in which each is allowed to use his knowledge for his own purposes the concept of ‘social justice’ is necessarily empty and meaningless, because in it nobody’s will can determine the relative incomes of the different people, or prevent that they be partly dependent on accident. ‘Social justice’ can be given a meaning only in a directed or ‘command’ economy (such as an army) in which the individuals are ordered what to do; and any particular conception of ‘social justice’ could be realized only in such a centrally directed system. It presupposes that people are guided by specific directions and not by rules of just individual conduct. Indeed, no system of rules of just individual conduct, and therefore no free action of the individuals, could produce results satisfying any principle of distributive justice.

We are of course not wrong in perceiving that the effects of the processes of a free society on the fates of the different individuals are not distributed according to some recognizable principle of justice. Where we go wrong is in concluding from this that they are unjust and that somebody is to be blamed for this. In a free society in which the position of the different individuals and groups is not the result of anybody’s design—or could, within such a society, be altered in accordance with a generally applicable principle—the differences in reward simply cannot meaningfully be described as just or unjust. There are, no doubt many kinds of individual action which are aimed at affecting particular remunerations and which might be called just or unjust. But there are no principles of individual conduct which would produce a pattern of distribution which as such could be called just, and therefore also no possibility for the individual to know what he would have to do to secure a just remuneration of his fellows.

[…]

The conduct of the individuals in that process may well be just or unjust; but since their wholly just actions will have consequences for others which were neither intended nor foreseen, these effects do not thereby become just or unjust. (p. 69–70)

And that’s only a small tip of the iceberg; it’s really great stuff. Of course, he’s writing from the perspective of an economist, discussing the market system and its external effects, but that’s still applicable to Rawlsian theory on a number of levels.

Additionally, he sharply criticizes the notion of distributive justice on the basis of reallocation. He denounces any attempt by government to redistribute capital or income on the basis of individual need as an unacceptable intrusion upon individual freedom (bringing in shades of Nozick’s critique, which accuses distributive justice of being in contradiction with Rawls’s own expansive theory of individual rights).

If you’re not much of the book type, here’s a YouTube video that I just turned up in a Google search, showing James Buchanan and Hayek discussing where Rawls went wrong in his conception of social justice. But I must warn: There are probably better videos, and I don’t have sound where I am, so I can’t screen it.

Finally, if critical theory is your bent, you can find some good material from feminist authors to use as a critique of Rawls. Martha Nussbaum and Iris Marion Young (one of my personal favorites) are probably the most well-known here.


As far as a good contemporary of Rawls, you might look no further than Rawls himself! He has written several books following ATOJ  that aim to respond to some of his critics’ writing in the interim (Nozick in particular).

And I would strongly suggest reading the works of Thomas Nagel. He actually argues that Rawls’s theory of justice doesn’t go nearly far enough, as it merely seeks to redress the inequalities, rather than remove them altogether. He laments that a Rawlsian state would still permit intolerable inequalities and that we need to adopt an even more ambitious view of equality. For more on this, check out Equality and Partiality.

Answer 2 (score 4)

First of all, I just don’t believe people are exchangeable in this fashion, because of hereditarian considerations; the exchanging of places before hand would not, in many cases, would not lead to a significant “shake-up” of society, if meritocracy is truly operating — so considering things with a veil seems needless.

With respect, I think that this suggests a slight misunderstanding of what Rawls is arguing. He is well aware that people are not created equal. However, what he does believe is that every individual should be taken to have equal moral status i.e. our considerations of justice shouldn’t start from the starting point of preferential treatment towards some. (I would imagine - or hope! - that very few would disagree with this as a fundamental part of the definition of ‘justice’.)

So, we’re trying to work out fair principles that treat everyone as morally equally important, but these principles are to govern over a situation where people are not equal in strength, mental ability, inherited wealth, social connections, and so on. Now, if we actual people were to try to design these principles then it seems likely that, say, on the whole the weakest or poorest might try to design principles that put their interests above all others, whereas the wealthiest and most powerful might try to design principles that maintain their status. The naturally physically strongest might try to design principles that link power to physical aptitude. And so on - and this doesn’t seem fair, or workable.

So, how can we avoid this situation? Rawls thinks that we can avoid it by undertaking a thought experiment: if none of us actually knew anything about our social status, strengths/weaknesses, race, gender, etc., but knew that we were about to enter into a society that we were going to have to be happy in, what principles would we choose?

What is actually going on here is that the method, in the thought experiment, of depriving the deliberating parties of information is a way of building in fairness and impartiality into the deliberation. The parties can’t possibly be unfair to one another in their choice of principles because they wouldn’t know how, and wouldn’t know whether their choices would actually disadvantage themselves. As such, whatever principles these imaginary parties would choose will be fair and impartial.

Now, we could argue about exactly what principles the parties behind the veil would actually choose, but, at any rate, the above is the method and whatever else we might say one can understand the thinking behind it and appreciate the philosophical elegance. I think it would be a mistake to suggest that it relies on the idea that people could be ‘exchanged’; firstly, it is just a thought experiment designed to generate certain kinds of conclusions in the right way, and so doesn’t really have a lot to do with actual people, and secondly, its aim is to arrive at principles that can ensure the just social co-existence of people who, indeed, aren’t interchangeable.

Secondly, using the veil to argue for distributive justice and egalitarianism, as Rawls does, in my opinion seems to presume that moral virtue is orthogonal to societal position, so that it is only “fair” that we “start off on the same foot”; I don’t agree with that either, because I think the poor, at least in America, are somewhat less virtuous than middle America or the rich, and that a moral accounting behind this veil would in any case send these lacking to the same positions they occupy.

I think I read above that this isn’t a forum for opinion so I’ll move swiftly on from that one (!…) but I think again Rawls’s answer would centre around the idea of the equal moral status of persons (at least at birth). Clearly, many would argue that during life people through their agency makes choices that mean that they ‘deserve’ or ‘don’t deserve’ certain things, but Rawls thinks that in the eyes of justice every person is still equal; no matter how ‘good’ or ‘bad’, people don’t earn preferential treatment from justice (we wouldn’t say that someone who gives to charity should get away with murder, or that people who are mean to their friends should be stripped of their wealth). Rawls isn’t really interested in what people ‘deserve’ through their deeds (for that you want Robert Nozick) or through some idea of their innate virtue, but rather in having a social system that isn’t predestined to militate against the life chances of particular people and groups.

But, alas, I’m a naif in philosophy, having never studied it seriously. So I have two questions: Are there any prominent attacks on Rawls’ position along these lines, and secondly, if so, have any liberal philosophers updated Rawls’ argument to deal with positions from hereditariainism and so on?

On your first complaint, that people are different and not exchangeable, there is a well-known critique of Rawls - and perhaps of liberalism and the social contract more generally - that it assumes that all people are essentially equal and the same, when in fact they are not, as is proved by the ubiquitous fact of need and dependence in society. It is unclear that, say, the mentally handicapped or the very old and frail, or young children, can participate in the (hypothetical) social contract that Rawls envisages, and so - the critique goes - Rawls cannot deal with difference and dependence and need. This argument is particularly associated with feminist critics like Martha Nussbaum or Eva Kittay. I’ve not explained it particularly well but it is easy to look up and is often called the ‘dependence critique’ of Rawls. Handily for your second question, both Nussbaum and Kittay are still essentially within the liberal tradition and aim to adapt rather than to overhaul Rawlsian liberal egalitarianism.

On your second complaint, that the idea of ‘starting off on the same foot’ is misguided because virtue tends to increase up the income distribution (at least in the US), it sounds like Robert Nozick would be about the closest to what you have in mind. I doubt that he would express it in terms of the ‘virtue’ of different social groups, but he too doesn’t like the idea of starting off on the same foot because he is interested in property and what it means to hold property justly, and for him as long as property was acquired justly in the first place and has been passed on fairly - such as through a family - then it is still held justly. As a result, his conclusions are essentially very right-wing in advocating almost no redistribution or interference in the market (although not quite as right-wing as suggesting that the poor are less virtuous than the middle class and wealthy and even given the chance would still go sliding back down to a lowly and un-virtuous position…)

Answer 3 (score 4)

First of all, I just don’t believe people are exchangeable in this fashion, because of hereditarian considerations; the exchanging of places before hand would not, in many cases, would not lead to a significant “shake-up” of society, if meritocracy is truly operating — so considering things with a veil seems needless.

With respect, I think that this suggests a slight misunderstanding of what Rawls is arguing. He is well aware that people are not created equal. However, what he does believe is that every individual should be taken to have equal moral status i.e. our considerations of justice shouldn’t start from the starting point of preferential treatment towards some. (I would imagine - or hope! - that very few would disagree with this as a fundamental part of the definition of ‘justice’.)

So, we’re trying to work out fair principles that treat everyone as morally equally important, but these principles are to govern over a situation where people are not equal in strength, mental ability, inherited wealth, social connections, and so on. Now, if we actual people were to try to design these principles then it seems likely that, say, on the whole the weakest or poorest might try to design principles that put their interests above all others, whereas the wealthiest and most powerful might try to design principles that maintain their status. The naturally physically strongest might try to design principles that link power to physical aptitude. And so on - and this doesn’t seem fair, or workable.

So, how can we avoid this situation? Rawls thinks that we can avoid it by undertaking a thought experiment: if none of us actually knew anything about our social status, strengths/weaknesses, race, gender, etc., but knew that we were about to enter into a society that we were going to have to be happy in, what principles would we choose?

What is actually going on here is that the method, in the thought experiment, of depriving the deliberating parties of information is a way of building in fairness and impartiality into the deliberation. The parties can’t possibly be unfair to one another in their choice of principles because they wouldn’t know how, and wouldn’t know whether their choices would actually disadvantage themselves. As such, whatever principles these imaginary parties would choose will be fair and impartial.

Now, we could argue about exactly what principles the parties behind the veil would actually choose, but, at any rate, the above is the method and whatever else we might say one can understand the thinking behind it and appreciate the philosophical elegance. I think it would be a mistake to suggest that it relies on the idea that people could be ‘exchanged’; firstly, it is just a thought experiment designed to generate certain kinds of conclusions in the right way, and so doesn’t really have a lot to do with actual people, and secondly, its aim is to arrive at principles that can ensure the just social co-existence of people who, indeed, aren’t interchangeable.

Secondly, using the veil to argue for distributive justice and egalitarianism, as Rawls does, in my opinion seems to presume that moral virtue is orthogonal to societal position, so that it is only “fair” that we “start off on the same foot”; I don’t agree with that either, because I think the poor, at least in America, are somewhat less virtuous than middle America or the rich, and that a moral accounting behind this veil would in any case send these lacking to the same positions they occupy.

I think I read above that this isn’t a forum for opinion so I’ll move swiftly on from that one (!…) but I think again Rawls’s answer would centre around the idea of the equal moral status of persons (at least at birth). Clearly, many would argue that during life people through their agency makes choices that mean that they ‘deserve’ or ‘don’t deserve’ certain things, but Rawls thinks that in the eyes of justice every person is still equal; no matter how ‘good’ or ‘bad’, people don’t earn preferential treatment from justice (we wouldn’t say that someone who gives to charity should get away with murder, or that people who are mean to their friends should be stripped of their wealth). Rawls isn’t really interested in what people ‘deserve’ through their deeds (for that you want Robert Nozick) or through some idea of their innate virtue, but rather in having a social system that isn’t predestined to militate against the life chances of particular people and groups.

But, alas, I’m a naif in philosophy, having never studied it seriously. So I have two questions: Are there any prominent attacks on Rawls’ position along these lines, and secondly, if so, have any liberal philosophers updated Rawls’ argument to deal with positions from hereditariainism and so on?

On your first complaint, that people are different and not exchangeable, there is a well-known critique of Rawls - and perhaps of liberalism and the social contract more generally - that it assumes that all people are essentially equal and the same, when in fact they are not, as is proved by the ubiquitous fact of need and dependence in society. It is unclear that, say, the mentally handicapped or the very old and frail, or young children, can participate in the (hypothetical) social contract that Rawls envisages, and so - the critique goes - Rawls cannot deal with difference and dependence and need. This argument is particularly associated with feminist critics like Martha Nussbaum or Eva Kittay. I’ve not explained it particularly well but it is easy to look up and is often called the ‘dependence critique’ of Rawls. Handily for your second question, both Nussbaum and Kittay are still essentially within the liberal tradition and aim to adapt rather than to overhaul Rawlsian liberal egalitarianism.

On your second complaint, that the idea of ‘starting off on the same foot’ is misguided because virtue tends to increase up the income distribution (at least in the US), it sounds like Robert Nozick would be about the closest to what you have in mind. I doubt that he would express it in terms of the ‘virtue’ of different social groups, but he too doesn’t like the idea of starting off on the same foot because he is interested in property and what it means to hold property justly, and for him as long as property was acquired justly in the first place and has been passed on fairly - such as through a family - then it is still held justly. As a result, his conclusions are essentially very right-wing in advocating almost no redistribution or interference in the market (although not quite as right-wing as suggesting that the poor are less virtuous than the middle class and wealthy and even given the chance would still go sliding back down to a lowly and un-virtuous position…)

25: Was Socrates a fictional character invented by Plato? (score 36595 in 2012)

Question

I have read a lot of websites that suggest Socrates was a fictional character created by Plato (albeit without the citation of any corroborating evidence), but I have also read the opposite (and by “opposite” here I don’t mean that Plato was created by Socrates but rather that Socrates was a living, breathing person).

Is there any truth to this claim?

Answer accepted (score 40)

It’s essentially impossible to offer definitive proof on the matter, but it’s extremely unlikely that Socrates was merely a figment of Plato’s imagination.

The primary evidence in this regard is the fact that multiple independent sources make reference to him in various ways. For example, the philosopher Xenophon of Athens was a student and admirer of Socrates, who dedicated himself to the preservation of Socrates’s wisdom.

Specifically, in the Anabasis, Xenophon writes of asking Socrates for advice regarding his entrance into the service of Cyrus when he was a young man. Socrates is reported to have advised him to consult the oracle of Delphi, and later chastising Xenophon for the question he ultimately decided to ask (one that betrayed his mind had already been made up to go).

Additionally, the Memorabilia—itself a collection of Socratic dialogues—is notable for containing Xenophon’s extended defense of his mentor. He argues that Socrates was innocent of the charges levied against him, and describes how Socrates benefitted not only his friends, but all Athenians.

It has even been argued that Xenophon’s later exile from Athens was motivated (at least in part; his support for Athens’s rival Sparta at Coronea unquestionably had something to do with it as well) by his support for Socrates.

Of course, some of Xenophon’s writings have come under scrutiny for their historical reliability, much as you’ve noted that Plato’s writings have. And ultimately, this debate is probably unresolvable. But it does seem quite unlikely that both Plato and Xenophon would make up the same figure and agree about many of the details of his life.

Beyond the realm of philosophy, the playwright Aristophanes claimed to have known Socrates. His comedy, the Clouds, features Socrates as a character. But it goes without saying that plays, and especially comedies, are an unreliable source of historical information. The Clouds has come under particularly heavy criticism by scholars because it appears that its “Socrates” character is actually a bricolage of many different fifth-century intellectuals. (For more on this view, see in particular, the discussion in the introduction of Kenneth Dover’s 1968 translation of Clouds.)

Answer 2 (score 8)

Socrates was a living person, and was really sentenced to death. He wasn’t the only one being sentenced to death for contrived reasons during that time in Athens. People fleeing Athens for fear of persecution sometimes referred to Socrates death for justification. I see this as some form of convention, because the accusation “not holding the gods in honor” often just masked reasons more closely related to xenophobia, and referring to Socrates allowed the victims to avoid giving the real reasons why they feared persecution.

So when Aristotle explained “I will not allow the Athenians to sin twice against philosophy”, he really feared persecution, albeit for reasons unrelated to philosophy (or “not holding the gods in honor”).


Edit I just tried to get access to the tertiary literature again from where I took the information in this answer, in order to be able to cite the sources they reference. This wasn’t successful, but I found that most balanced accounts of the Socrates trial also contain the information to which I referred in this answer. Hopefully the following quote clarifies the significance of the “contrived reasons”:

A general amnesty issued in 403 meant that Socrates could not be prosecuted for any of his actions during or before the reign of the Thirty Tyrants. […]

Important support for Stone’s conclusion comes from the earliest surviving reference to the trial of Socrates that does not come from one of his disciples. In 345 B.C.E., the famous orator Aechines told a jury: “Men of Athens, you executed Socrates, the sophist, because he was clearly responsible for the education of Critias, one of the thirty anti-democratic leaders.”

I intentionally wrote “Socrates was a living person” instead of “it’s extremely unlikely that Socrates was merely a figment of Plato’s imagination”, because I’m convinced that the available evidence for this statement is not weaker than the available evidence for other commonly accepted statements.

Answer 3 (score 3)

I’ve always ascribed to the theory that Plato’s early dialogs portray his actual teacher, Socrates, with reasonable accuracy; but that his later dialogs more and more use the figure of Socrates as a mouthpiece for Plato’s own philosophies and views. In that sense, the “Socrates” that we know through Plato would be a highly fictionalized version of a real person.

26: Is there anything that is totally random? (score 35900 in 2012)

Question

When I say totally random, I mean absolutely random, not pseudorandom.

If I want to say “totally random” numbers such as 1,26,17,4,1 and 27, although I see them to be totally random, they aren’t. These are numbers that I think are influenced by my childhood, ideology and everything that I’ve seen.

So, what do you think? Is there anything completely random?

Answer accepted (score 5)

Quantum mechanical processes, such as a circularly polarized photon striking a surface that is linearly polarized (whereupon it, to anthropomorphize the situation, has to choose whether it was really polarized perpendicular to or parallel to the polarization axis), do not appear to be (locally) deterministic. For all practical purposes, therefore, it makes sense to consider them as “completely” random. It is formally possible that there are hidden state variables distributed throughout the universe away from the photon itself that nonetheless govern its behavior, but since no-one has figured out a way to test those, living in such a universe and living in a genuinely nondeterministic one may be indistinguishable.

(See “Bell’s inequality” for more details.)

Answer 2 (score 6)

Although this question is very closely related to the one which I commented about, I will still give an answer because the question is asking for a different answer.

Before I give you an answer, I want to clear one thing up.

I don’t quite understand what you are trying to convey when you state

Is there anything completely random in the universe?

This question seems quite incorrect as numbers are a theoretical concept invented/discovered (either which are open to debate: the question with the highest votes on this site) by human beings and are (as far as our knowledge concerns) exclusive to humans and the creations of humans (e.g. computers).

Therefore, I don’t understand what you mean by “the universe”.

First of all, it can be debated whether humans can/cannot generate random numbers. However, for the sake of simplicity, let’s pretend that humans, cannot under any circumstances generate random numbers.

That does not mean that it cannot be generated at all:

Computers are a prime example of something that can generate the closest thing to a truly random number. They are senseless, unbiased, and are not conscious. Thus, past experiences don’t affect them at all (unless otherwise programmed). They are linear machines that perform only the tasks they are instructed to, no more, no less.

EDIT: In response to Stoicfury’s comment, I am adding more to my answer.

Apparently, eventually, even computer hardware that can generate numbers that are very close to truly random, after a while, a rough pattern can be detected.

In that case, the closest thing you will ever get to creating a truly number generator is on random.org where the determined number is based on the slightest deviations in sound in the atmosphere. In this case, however, this randomness is no longer computer generated but rather generated by the environment.

It depends what you define as random, really.

If you define it to be a number that is generated unbiased and at the same time independent and uninfluenced by any physical factors, then it is impossible due the law of cause and effect.

Answer 3 (score 5)

Although this question is very closely related to the one which I commented about, I will still give an answer because the question is asking for a different answer.

Before I give you an answer, I want to clear one thing up.

I don’t quite understand what you are trying to convey when you state

Is there anything completely random in the universe?

This question seems quite incorrect as numbers are a theoretical concept invented/discovered (either which are open to debate: the question with the highest votes on this site) by human beings and are (as far as our knowledge concerns) exclusive to humans and the creations of humans (e.g. computers).

Therefore, I don’t understand what you mean by “the universe”.

First of all, it can be debated whether humans can/cannot generate random numbers. However, for the sake of simplicity, let’s pretend that humans, cannot under any circumstances generate random numbers.

That does not mean that it cannot be generated at all:

Computers are a prime example of something that can generate the closest thing to a truly random number. They are senseless, unbiased, and are not conscious. Thus, past experiences don’t affect them at all (unless otherwise programmed). They are linear machines that perform only the tasks they are instructed to, no more, no less.

EDIT: In response to Stoicfury’s comment, I am adding more to my answer.

Apparently, eventually, even computer hardware that can generate numbers that are very close to truly random, after a while, a rough pattern can be detected.

In that case, the closest thing you will ever get to creating a truly number generator is on random.org where the determined number is based on the slightest deviations in sound in the atmosphere. In this case, however, this randomness is no longer computer generated but rather generated by the environment.

It depends what you define as random, really.

If you define it to be a number that is generated unbiased and at the same time independent and uninfluenced by any physical factors, then it is impossible due the law of cause and effect.

27: What would Kant do when two categorical imperatives conflict? Could he ever justify lying? (score 35350 in 2011)

Question

Suppose a German SS officer knocked on my door, asking me whether I had any Jews. And suppose further that I had two Jews in a secret compartment in the attic that he’d never be able to find. Everybody will agree that I must lie and say I haven’t any Jews in my house.

But I’d have to disobey Kant’s categorical imperative “do not lie”, because I felt obliged to “not betray innocent people leading to their death”.

Both imperatives can be supported if I ask myself the question “what if everyone did it?”. If everyone lied whenever they felt like it, society and civilization would collapse. Therefore I must never lie. But society would be a grim, inhumane place if everyone cooperated in killing the innocent. Therefore I must never betray those under my protection.

How can this be solved? I remember hearing two “solutions”:

  1. I must heed both imperatives and choose a third option, thereby violating neither: “I refuse to answer you”. This would result in my death, but I would have upheld both imperatives.

    I find this absurd; I do not think Kant would have wanted this. And yet, when challenged by Constant with a similar situation, he said that one should challenge the murderer or refuse to answer. In this case, that would lead to my house being burned down, the Jews dying an even more horrible death.
  2. The “do not lie” imperative is too broadly formulated: it should be “do not lie, except when your lie would save another’s life”. I find this very weak: the power of a categorical imperative would be that it is a broad statement. It should be a universal law.

    The fundamental problem would seem to be that each rule saying “I wouldn’t want everyone to do this” has a reasonable exception, and that it should not be left to each single person to determine what the exceptions are depending on a particular situation. I believe Schopenhauer offered similar criticism.

How did Kant solve this problem while keeping his categorical imperatives intact? Or, if he didn’t, how could it be solved hypothetically?

Answer accepted (score 30)

Kant himself (and many other deontologists that follow in his footsteps) would never ask the question: “what if everyone did it?” In fact, it’s quite irrelevant to their moral calculi. He famously decrees that:

The greatest violation of a human being’s duty to himself regarded merely as a moral being (the humanity in his own person) is the contrary of truthfulness, lying. In the doctrine of right an intentional untruth is called a lie only if it violates another’s right; but in ethics, where no authorization is derived from harmlessness, it is clear of itself that no intentional untruth in the expression of one’s thoughts can refuse this harsh name. [ … ] By an external lie a human being makes himself an object of contempt in the eyes of others; by an internal lie he does what is still worse: he makes himself contemptible in his own eyes and violates the dignity of humanity in his own person. […] By a lie a human being throws away and, as it were, annihilates his dignity as a human being.
(Kant, The Metaphysics of Morals, 1996, p 182)

As you observe, from a strictly Kantian point of view, there is absolutely no way that a lie can ever be justified. It is a complete violation of human agency, insofar as it manipulates someone (effectively treating them as a means rather than an end in and of themselves) into believing something for whatever reason. While it could be argued that the overall goal of the lie is a moral good, Kant would reject it a priori for its means, which he would consider sharply and incontestably immoral. That immorality outweighs and precludes any morality that might come later from the results of the action.

Thus, it makes absolutely no difference with respect to the morality of the original decision (i.e., whether or not to lie to the soldier) whether the eventual outcome of your decision is utter social chaos or collapse. That’s an ends-based calculation, one that Kant says you can’t possibly know at the time and shouldn’t possibly matter when assessing the morality of one individual action.

At least one of the justifications that Kant would (and did) provide for this particular moral choice is that you are not responsible for the death of the two Jews, even if your telling the truth leads to their brutal murder by the Nazi soldiers. The consequence may be undeniably horrible, but it wasn’t your crime and you weren’t directly responsible for its execution. Essentially, he’s saying, “that’s on them, not you.”

However, if you lie, you now take ownership and responsibility in the atrocity. Or, in other words, you become morally culpable for the immoral actions of the Nazi soldiers because you are essentially manipulating the soldiers yourself through your decision to lie and distort the truth. And once you’ve made the choice to distort the truth, you’ve established control over the situation and you should be held responsible for whatever immoral outcome may result. This is the infamous causing versus allowing distinction.

All of that to say that this is really not a contradiction or dilemma for a strict Kantian. Of course, that hardly means that it fails to be one for most of us, who take a far less extremist view of morality.


It’s also worth considering that a neo-Kantian would take issue with your example on technical level, arguing that it’s not a good example because the Holocaust is an exceptional situation, a special case, if you will.

More specifically, the German state at the time formed an illegitimate government, and the SS officers were acting as agents of this illegitimate government, this state that is on face unjust. Given such case, one could argue, his entire corpus of ethics does not apply, as it was only intended to apply to situations involving a just government. (For more on this, see Kant’s “Doctrine of Right” and Metaphysics of Morals 6:264, etc.)


Finally, if a Kantian discussion of this exact question interests you, you might find this recently-published article to be a worthwhile read.

Answer 2 (score 5)

While the question is an interesting one, there are several issues at hand to consider. First, the question seems premised on the belief that there can be multiple categorical imperatives. This is not Kant’s view of the matter. For Kant, there can be multiple hypothetical imperatives but only one categorical imperative (not all contemporary Kantians agree). Hypothetical imperatives derive from either the goal of happiness (to be happy you need to eat) or the desire to accomplish some task (To make bread, you need yeast). But the Categorical Imperative follows from the dictates of reason.

I next want to look at this paragraph in your question:

Both imperatives can be supported if I ask myself the question “what if everyone did it?”. If everyone lied whenever they felt like it, society and civilization would collapse. Therefore I must never lie. But society would be a grim, inhumane place if everyone cooperated in killing the innocent. Therefore I must never betray those under my protection.

This paragraph has several interesting features. First, there’s a misunderstanding of how the categorical imperative works. Second, there is an important slippage in the language between the two universalizations.

On the first point, I will limit my remarks to the universalization model of the Categorical Imperative. The question is not “what if everyone did it?” but rather what if it was always to happen or in a possibly more loose way “what if it became a law of nature?” Moreover, the problem that is supposed to raise is not necessarily about societal collapse but about contradictions in implementation (I will always tell lies is a system that no longer works when every rational creature is always telling lies – regardless of social consequences it eliminates the meaning of its own criterion.)

Second, the characterization of telling the man at the door about the people you are hiding is problematic. Specifically, when it is described as “cooperat[ing] in killing the innocent.” This cannot possibly be an expression of the categorical imperative. But this is also not what Kant expects of the agent. Instead, Kant expects the agent to cooperate with the request of a seemingly rational agent (this distinction is crucial to Korsgaard’s modification of Kant’s answer). On Kant’s account, we must assume the requests we receive from other humans no matter how unreasonable are founded on reason. In other words, despite the blood on the axe, we have to assume they are asking – not to engage in the immoral act of killing the innocent but for some other reason.

Thus, for Kant, there’s no conflict between these two ideas. In some ways, this makes things worse – because it undermines one of the better outs we have on the picture where we cannot lie. To some extent, Kant gives us a positive duty to tell the truth to other rational beings – not merely a negative duty not to lie. If we look in the Metaphysics of Morals, Kant does seem to have little leeway in suggesting we needn’t purposelessly express the truth.

Answer 3 (score 3)

I agree with what Chad says in that there are other options and that, to answer your question, civil disobedience (refusing to answer) suffices a solution to this particular problem. Consequentialism (your house getting burned down) is not a sustainable justification for disavowing a categorical imperative. This is because categorical imperatives deal in absolutes; as a result, an consequential analysis must have an absolute result, otherwise it fails.

At base however is whether any two categorical imperatives can conflict. (Assuming categorical imperatives can exist,) if two categorical imperatives (assuming that even one could exist) were to “conflict” would that mean that one was wrong, or perhaps that there was some temporal incompatibility that has yet to be resolved which upon resolution will show the two to be compatible?

The point being, assuming that two imperatives conflict, and we are dealing in some absolutist terms, and any solutions are mutually exclusive in meeting the needs of both; it is likelier that one categorical imperative is wrong or misunderstood than that no solution is useful.

28: What is the difference between a statement and a proposition? (score 35273 in 2014)

Question

I’m doing a MOOC on mathematical philosophy and the lecturer drew a distinction between a proposition and a statement. This is very puzzling to me. My background is in math and I regard those two words as synonymous. I looked on Wikipedia and it says:

Often propositions are related to closed sentences to distinguish them from what is expressed by an open sentence. In this sense, propositions are “statements” that are truth bearers. This conception of a proposition was supported by the philosophical school of logical positivism.

http://en.wikipedia.org/wiki/Proposition

This also went right over my head. I (naively) regard both a proposition and a statement to be well-formed formulas that, once a suitable interpretation is chosen, have the ability to be either true or false. For example 2 + 2 = 4 is a proposition or statement because once I assume the Peano axioms along with the usual interpretations of the symbols ‘2’, ‘4’, ‘+’, and ‘=’, this statement is capable of being determined to be true or false.

Can anyone shed some light?

Answer accepted (score 19)

Leitgeb distinguishes between statements, which are declarative sentences (he calls them ‘descriptive sentences’), from propositions, which, unlike statements, are not linguistic objects. Propositions are the sort of objects that can have truth-values. E.g., [that snow is white] is a true proposition (Lecture 2-1).

Once the distinction is made, the key idea is this: statements express propositions, which are then said to be true or false. E.g. “snow is white” is a statement that itself doesn’t have a truth-value, but instead expresses the proposition that snow is white, which happens to be true. That’s pretty much it.

As regards your “2 + 2 = 4” example, Leitgeb could say this: “2 + 2 = 4” and “two plus two equals four” are two different sentences that express the same proposition. If you call them both ‘proposition’, then since the two sentences are syntactically distinct, you’ll be committed to the claim that “2 + 2 = 4” and “two plus two equals four” are different propositions (this might be okay with you, but I think something is wrong with that). You might find the following analogy between algorithms and programs useful: given a single algorithm (~proposition), there are often multiple programs (~sentences) that implement it.


Leitgeb, Hartmann (2014 Spring) Introduction to Mathematical Philosophy (Coursera).

Answer 2 (score 2)

In philosophy of language (and metaphysics), statements are linguistic objects, like sentences of a natural language. Propositions are (traditionally understood as) the meanings of sentences (of a language) (in a context of utterance).

To illustrate:

The German statement “Schnee ist Weiss.” expresses the same proposition as the English statement “Snow is white.”

The distinction is arguably not immediately relevant for model-theoretic semantics of formal languages. Very few (if any) take the well-formed formulas of a formal language of mathematics to express propositions, although the connection between the semantics of formal languages and the semantics of natural languages is a hotbed of linguistic and philosophical issues of active research since (at least) Montague.

Answer 3 (score 0)

I hope the following statements help you see the difference.

A proposition is a type of (logical) statement.
A statement does not have to be a proposition (logical).

29: Could ‘cogito ergo sum’ possibly be false? (score 34700 in 2013)

Question

I’ve heard it postulated by some people that “we can’t truly know anything”. While that does seem to apply to the vast majority of things, I can’t see how ‘cogito ergo sum’ can possibly be false.

No matter what I am, no matter in what way I’m being tricked, no matter how I may be deluded, I must exist in order to - in some way - be considering this right now (or be being tricked right now).

Is ‘cogito ergo sum’ necessarily correct, or have I missed something?

Answer accepted (score 71)

The objection to “I think, therefore I am” is that it presupposes the existence of an “I” doing the thinking. Possibly, “there are thoughts” is the true minimum statement that can be reached using Descartes’ method; this does not presuppose the existence of some sort of unified consciousness having the thoughts.

Bernard Williams and Søren Kierkegaard were early objectors to presupposing “I”.

That said, cogito ergo sum certainly has a stronger basis than the rest of what Descartes built on it with the Cartesian Circle.

Answer 2 (score 30)

This is exactly what Descartes was trying to prove. He wanted to build philosophy from the ground up based on a series of rational observations. I think he must’ve been quite dismayed when he realized that it is possible to take a skeptical position on almost everything.

His conclusion in his Meditations I & II was that this is the only truth, and the only thing we can know for absolute certainty. Whether this is true or not depends on how convinced you are by his Meditations.

The phrase is not without its critics, but I think it’s a matter of semantics. Let me explain:

The essence of his argument is that you can doubt almost everything about the world, but you can’t doubt that you’re doubting. Because if you doubt that you’re doubting, you’re still doubting. This is why the phrase is sometimes seen as ‘I doubt, therefore I am.’

Most of the contention comes from trying to place meaning to the words ‘I’ and ‘exist’. Perhaps it is not me who is doubting, but there is still doubt. I tend to prefer the more constrained version: ‘There is doubt.’ This statement avoids a lot of the semantic ambiguity, but connotes a weaker argument.

Answer 3 (score 12)

Yes it may be false

I think therefore I am is a logical conclusion, aimed at showing that something exists. It assumes that logic works and exists. How can you assume that before you even know if you or anything else exists?

Well, you can’t, of course. And famously logic is in itself impossible to prove. Therefore we can not say that Cogito is proven beyond all doubt, and hence it still may be false.

It is however an untenable position to claim that you or the world around you do not exist, but that is another question.

30: Why is Ayn Rand’s Objectivism philosophy dismissed by academics? (score 33727 in 2017)

Question

This is a question in response to this other one that I asked. I didn’t really get a satisfactory answer, mostly because it seems like Rand’s work is largely ignored by academics. The highest voted answer starts with “you’ll be hard-pressed to find a serious philosopher who takes Ayn Rand seriously”. Why is this the case? To a layperson like myself, her ideas seem pretty well thought out, self consistent, and thought provoking, if a bit black-and-white. Is her work lacking rigor? Is the problem that her work takes the form of fiction rather than non-fiction? Are the ideas presented flawed in a way that is not obvious to me?

Answer accepted (score 30)

“you’ll be hard-pressed to find a serious philosopher who takes Ayn Rand seriously”. Why is this the case?

This is the case because Rand offered very few philosophical arguments. I actually do not know of any particular arguments she did offer, but I will assume that she at least attempted to offer some.

Is her work lacking rigor?

In the analytic tradition, philosophical arguments consist of clearly stated premises and logically sound conclusions. Some (already noted) attempts to locate in Rand’s work such specific premises and conclusions show that her positions on several important points (e.g. causation and free will) are self-contradictory and inconsistent. This makes her conclusions irrelevant to the philosophical discourse in which she appears to have been attempting to participate. So in this way, the answer to your above question is a resounding “YES!”; a lack of argumentation entails a lack of rigor in argumentation.

Upon reflection, I realize that one could respond here by saying that Rand was engaged in a more Continental approach to philosophy. Consider her tendency to employ fiction to “make her point” – a strategy (as previously mentioned in another answer) employed by Sartre and Camus, among other Continentals. Reading her this way, Rand’s work could be seen as having a place within some larger historical philosophical discourse. However, her well-documented ideological struggle against Marxism undermines her own argument again here; the Marxist dialectic underpins the Continental approach to philosophy. If we are to take Rand’s conclusions seriously, which is to say that if we take her particular anti-Marxism to be the point she is making, then she is using the Marxist dialectic to completely disavow Marxism, thus ending the dialectic. There are two problems with this. First, her work would be little more than her bid to be the “the champion of all philosophical discourse”, and second, there are still WAY too many questions left unanswered for us to stop doing philosophy.

So frankly, she has written nothing particularly philosophically interesting or compelling.

To a layperson like myself, her ideas seem pretty well thought out, self consistent, and thought provoking, if a bit black-and-white.

This highlights another frustration some academic philosophers might have regarding Ayn Rand. She relies on absolute self-certainty where she should be relying on well-reasoned arguments. One problem here is doing this often resembles well-thought-out, consistent philosophy, but in terms of actual philosophy, it amounts to nothing more than polemics and screeds. From her writings, all that can be logically deduced are her opinions, yet I have no doubt that Rand dislikes Marxism, nor do I doubt her love of Capitalism. Another problem is that assuming the certainty of your conclusions is simply poorly-executed philosophy – every philosopher must be willing to accept that they could be wrong about their conclusions, or else they are not doing philosophy.

Again, her work is thus philosophically uninteresting. But if your thoughts are provoked by her writing, that’s a good thing. I recommend taking some time to sketch out her arguments and see for yourself if her conclusions actually follow from her premises.

Are the ideas presented flawed in a way that is not obvious to me?

I hope this is answered above.

Answer 2 (score 30)

“you’ll be hard-pressed to find a serious philosopher who takes Ayn Rand seriously”. Why is this the case?

This is the case because Rand offered very few philosophical arguments. I actually do not know of any particular arguments she did offer, but I will assume that she at least attempted to offer some.

Is her work lacking rigor?

In the analytic tradition, philosophical arguments consist of clearly stated premises and logically sound conclusions. Some (already noted) attempts to locate in Rand’s work such specific premises and conclusions show that her positions on several important points (e.g. causation and free will) are self-contradictory and inconsistent. This makes her conclusions irrelevant to the philosophical discourse in which she appears to have been attempting to participate. So in this way, the answer to your above question is a resounding “YES!”; a lack of argumentation entails a lack of rigor in argumentation.

Upon reflection, I realize that one could respond here by saying that Rand was engaged in a more Continental approach to philosophy. Consider her tendency to employ fiction to “make her point” – a strategy (as previously mentioned in another answer) employed by Sartre and Camus, among other Continentals. Reading her this way, Rand’s work could be seen as having a place within some larger historical philosophical discourse. However, her well-documented ideological struggle against Marxism undermines her own argument again here; the Marxist dialectic underpins the Continental approach to philosophy. If we are to take Rand’s conclusions seriously, which is to say that if we take her particular anti-Marxism to be the point she is making, then she is using the Marxist dialectic to completely disavow Marxism, thus ending the dialectic. There are two problems with this. First, her work would be little more than her bid to be the “the champion of all philosophical discourse”, and second, there are still WAY too many questions left unanswered for us to stop doing philosophy.

So frankly, she has written nothing particularly philosophically interesting or compelling.

To a layperson like myself, her ideas seem pretty well thought out, self consistent, and thought provoking, if a bit black-and-white.

This highlights another frustration some academic philosophers might have regarding Ayn Rand. She relies on absolute self-certainty where she should be relying on well-reasoned arguments. One problem here is doing this often resembles well-thought-out, consistent philosophy, but in terms of actual philosophy, it amounts to nothing more than polemics and screeds. From her writings, all that can be logically deduced are her opinions, yet I have no doubt that Rand dislikes Marxism, nor do I doubt her love of Capitalism. Another problem is that assuming the certainty of your conclusions is simply poorly-executed philosophy – every philosopher must be willing to accept that they could be wrong about their conclusions, or else they are not doing philosophy.

Again, her work is thus philosophically uninteresting. But if your thoughts are provoked by her writing, that’s a good thing. I recommend taking some time to sketch out her arguments and see for yourself if her conclusions actually follow from her premises.

Are the ideas presented flawed in a way that is not obvious to me?

I hope this is answered above.

Answer 3 (score 20)

There are a number of interlocking reasons.

First, as you point out, she wrote fiction, not philosophical papers.

Second, she did not appear to engage in any substantive way with the prior philosophical work done on the subjects that interested her. Her only connection to the philosophical tradition (writ large) is what we can charitably call a highly idiosyncratic reading of Aristotle.

Third, her arguments (once extracted from the fiction) appear to most philosophers to be lacking subtlety and depth. They may seem like deep thoughts to casual readers, but they are not at a level of sophistication anywhere near that of mainstream philosophers. In fact, her arguments are generally polemical in nature.

31: What does Foucault mean by discursive practices or discursive constitutions in definition of discourse? (score 33436 in 2014)

Question

Foucault uses the concept of discursive practices or discursive constitutions in the definition of discourse.

What does he mean by this concept? How are discursive practices different from discourses?

Answer accepted (score 4)

A discursive practice in foucauldian terms is “the process through which [dominant] reality comes into being”. This is a very nebulous process, of course, and Foucault focuses on questions of power. His notions of ‘governmentality’ and ‘biopower’, from his later work, are helpful to understand this. Foucault does not only focus on formal and semi-formal institutions like the state, the law, schools, clinics, prisons, the family, race, gender, and sexuality, or not just on what the critical theorists and neo-marxists call the ‘Culture Industry’ (like the media); he notoriously concerned with how power is inscribed on the body, at the level of people’s movement and perception of themselves. How does Power produce ‘docile bodies’? is another way in which he poses the question. Biopower in this sense refers to the capillary living network (like veins or hairs) of how Power is propagated and inscribed on docile bodies.

Answer 2 (score 1)

By discursive practices Foucault means power relations in the society;how is a relationship created by asserting power through the use of language.Language of the powerful is shown through a variety of techniques such as speech styles which include vocabulary,syntax intonation,proverbs,naming strategies etc.Power and resistance are inter linked.The discursive practices denote the social status of the speakers thus bringing social stratification.

Answer 3 (score 1)

By discursive practices Foucault means power relations in the society;how is a relationship created by asserting power through the use of language.Language of the powerful is shown through a variety of techniques such as speech styles which include vocabulary,syntax intonation,proverbs,naming strategies etc.Power and resistance are inter linked.The discursive practices denote the social status of the speakers thus bringing social stratification.

32: What would happen if suddenly, 1+1=2 is disproved? (score 32175 in 2018)

Question

Would the universe be thrown into chaos when the most fundamental equation is proved to be wrong?

Answer accepted (score 109)

Is the world in chaos now? Because one plus one is not equal to two, at least not all the time.

Take one liter of water and one liter of sand. Add them together. What do you get? Wet sand, but certainly not two liters of it.

Take one rabbit and add one rabbit. Add them together. You have a reasonable chance of ending up with quite a bit more than two rabbits, if you wait a sufficient amount of time.

Even in the realm of pure mathematics one plus one is not necessarily equal to two. If you’re working with modulo two arithmetic, 1 + 1 = 0. If you’re dealing with modulo two arithmetic and 1 + 1 = 2, you’ve done something very wrong. – Also, it’s not like modulo two arithmetic is an obscure side-note - your computer is using it right now in the form of “bitwise xor”, and modern computers could not function without it. (Though admittedly, modulo two arithmetic is rather simple in its properties, so there’s not a lot of mathematicians that bother to study it.)

Mathematics is based on axioms - assumptions about the properties of a system - and the implications that follow logically from those systems. If one of those implications is found to be “counter-factual”, then either the logic was invalid, or one of the axioms was incorrect for that system. - For that system is an important bit. Just because something is counter-factual for one set of axioms doesn’t mean that it’s counter-factual for a different set of axioms.

Take Euclid’s parallel axiom. Include those with the rest of Euclid’s axioms, and you get Euclidian geometry. This is the “standard” geometry which you and I are familiar with, and with which a substantial fraction of mathematicians operate. However, you can set up different geometries where this doesn’t hold. In fact, modern physics tells us that we’re actually living in a non-Euclidean geometry - advanced physics would not function in a true Euclidean geometry where the parallel axiom holds.

Now does that mean that Euclidean geometries and the parallel axiom are wrong? No. It’s a perfectly valid mathematical construct which hundreds of thousands of mathematicians and engineers - and physicists - use daily. The fact that Euclidean geometry has axioms which produce results inconsistent with the observed world doesn’t mean Euclidean geometry is invalid, it just means that those axioms don’t apply to the system you’re observing. It doesn’t mean that they won’t apply - or even that they aren’t the best ones to use - in some other situation.

So 1+1=2 is a very convenient observation, and holds in many cases. But not all. Sometimes 1+1=0, or some other number. Just because the axioms of standard, natural number arithmetic don’t hold for a particular system doesn’t mean they’re invalid, it just means they’re not applicable to that system, and you have to come up with another set and another arithmetic system.

Or, you could redefine your system such that the axioms do hold. (That’s what the people frantically typing “But if you …” comments below are doing. “If you keep them in separate containers, if they’re both female, if we ignore modulo arithmetic …” If you redefine things such that the axioms hold, the logical consequences of those axioms logically follow.)

Answer 2 (score 102)

As any mathematician will tell you, 1 + 1 = 2 follows trivially from definitions, and is not a theorem. Your question makes no sense.

It is as though you declared:

I define 1 fluid zounce to be exactly 30 millilitres.

But what if it turns out I’m wrong?

It is your definition. It cannot be wrong because fluid zounces, prior to your definition, simply did not exist.

Answer 3 (score 19)

most fundamental equation

Your assumption is flawed. 1 + 1 = 2 is not an axiom of mathematics, but (as Sputnik points out) a consequence of the Peano axioms applied to base 10 representations of numbers.

One can easily change from decimal (base 10) to unary (base 1) and say:

1 + 1 = 11.

Or, change to binary (base 2, what your computer actually uses), and say:

1 + 1 = 10.

And for the sake of it, I can go into roman numerals:

I + I = II.

So, there are representations in which 1 + 1 is not 2 (and even systems where you don’t have the glyph 1), but the universe hasn’t imploded yet because of that.


Now, what if your question was more like…

What if the Peano axioms contradicts observations from the natural world?

In that case, my answer would be two-fold:

  • Mathematics based off on the Peano axioms would still be useful
  • Mathematicians would come up with another set of axioms that would fit the natural world, along with mathematics based off on those new axioms

To understand this, take for example Newtonian physics: they are a big ruleset of mathematics built on top of some axioms that nicely fit the observations from the natural world.

But then Einstein noticed that some of the axioms didn’t really fit (in particular when things go the speed of light), and came up with relativistic physics, which pretty much invalidate all of newtonian physics.

Even we know newtonian physics are wrong (because they are based on a model too simple), they are a tool valid for a lot of problems.

Same with Peano-based arithmetics: even if they don’t fit some observation in the natural world, they would still be good tools. And as a consequence of the unfitness, another set of mathematics could be derived from that.

33: Can something come out of nothing or not? Why? (score 32089 in 2014)

Question

In our current state of affairs it is safe and reasonable to assume something exists - be it a universe, pure conciousness, illusion or other designations. If some readers nevertheless claim something does not exist right now, then this question effectively becomes meaningless to them but for us “cogito ergo sum” should suffice.

So, let us (justifiably) assume right now something exists.

Therefore, when this something (as a whole) cannot come from nothing, then something must have always existed and cannot have a beginning. Is that entity the Universe or the Creator, is a different topic and a different question.

However, when this something can come from nothing, then this something (the whole of reality) might not have always existed and thus can have a beginning. Is that entity the Universe or something else, is also a different topic and a different question.

And here lies the apparent contradiction: between the widely-accepted axiom that something cannot come from nothing and between the present scientific view that whatever there is, it must have had some kind of an absolute beginning.

Why is it a contradiction? Well, when something cannot come from nothing, then where did our reality come from? If it can’t come from nothing, then either (the fundamental) reality itself is eternal, or it emerged from something eternal. The only way for our present reality to have an ultimate beginning is when something can in fact come from nothing. Otherwise everything requires something else prior to it, thus mandating that something must have always existed.

So, which way is it? Can something come out of nothing or not?

Answer accepted (score 7)

One argument is that time itself has a beginning. And thus the universe can be eternal, in the sense of being existant at all times. One could also argue that time must have a beginning, for how can an infinite amount of time elapse for it to be now (this is one half of a pair of arguments by Kant - his antinomies - with which he argues that a certain concept is beyond human reason to establish).

This still leaves begging the question what ‘happened’ before time began. Although naively this question looks nonsensical since we no longer have time - for then what can before mean - it still has sense in a speculative & imaginative sense. The only rational sense it seems that one can pose such questions.

In fact, certain speculative cosmologies of the Big Bang implicitly allow something to be exist before the big bang. For example, the universe began as a quantum fluctuation; one must ask in what sense physical laws exist before there is a space & time as traditionally understood. For the assertion to make sense at least this much must be true.

The argument that something cannot come out of nothing is a metaphysical one that goes back to at least Parmenides, if not earlier. In fact in the phenomenal world things always have beginnings and endings. For example, I have my hand open & then I close it: a fist has appeared and an open palm has disappeared, but of course what has remained constant between this, is my hand.

If something comes out of nothing then by what agency has it happened? from whence did it come from? If we postulate some fundamental physical law that allows something to come out of nothing, then nothing+physical laws, is not in fact nothing.

Answer 2 (score 4)

There is a scientific axiom that says ‘proof lies in the assertion’. You are asking to prove a negation. Your question is asking why cannot - your asking for a proof of the negation, not an assertion. The question should be ‘How can something come out of nothing’ not ‘Why cannot something come out of nothing’. Stephen Hawkings has recently argued as to how the universe can come out of nothing, but to my mind his argument is rather circular and it’s not provable.

The Hindu scriptures say that the universe is eternal; there never was a time when it was not, nor will there be a time when it will not be. Rather they say that there are ‘cycles’ - the universe kind of ebbs and flows like the tides so to speak. The scriptures say there is a periods of expansion and periods of contraction, one following the other. At the end of a cycle, the universe almost completely contracts into Brahman where it rests in potentiality before expanding again. (Brahman which is by definition neither existence nor nonexistence). The current scientific theories as to a big bang, point to a beginning of the universe as we perceive it now, most people in the West get the scientific big bang theory confused with their Judeo-Christian beliefs that was taught them when they were young and lingers in all their analysis. They confuse ‘beginning’ with ‘creation’. There is an assumption that before there was the big bang, there wasn’t anything, that the universe thus came out of nothing - thus a creation. The big bang theory doesn’t address what happened before; laymen assume there was nothing. Cosmologists don’t know and we can never know by scientific means what came before. There are cosmologists that are now addressing that there are many universes; that we can only perceive our own. We are one verse in the mulitverse. In the Hindu scriptures it is said that our universe is like a small bubble on the ocean of Brahman, and there are many bubbles. Joseph Campbell does an excellent summary of this in the first chapter (chapter titled Eternity and Time) of the book “Myths and Symbols in Indian Art and Civilization” by Heinrich Zimmer, edited by Joseph Campbell.

For some ‘thing’ to come out of no ‘thing’ is not logical.

Answer 3 (score 1)

I believe the current theory of Quantum Mechanics is that some particles pop into and out of existence all of the time. I think they call that a “quantum fluctuation”.

I have heard it postulated that the Big Bang was a sorta helluva quantum fluctuation. Very improbable to happen, but if you can wait around for eternity, I guess anything can happen.

Skeptic magazine founder Michael Shermer was asked about this and he said something sorta intriguing: “Perhaps something is more stable than nothing.”

If nothing is a state, then all possible states that this can transition to is either the same nothing (which might be virtually 100% likely, but not exactly 100%) or many zillion possible states of something. But once we’ve transitioned from nothing to something (despite the unlikelihood, but eventually even the unlikely will happen as long as it is possible) then, when the state is something the likelihood to transition back to a state of nothing (amidst the zillion of other something states) is also tremendously unlikely.

Nothing is a state sorta like perfectly balancing a pencil on its tip. Theoretically, if you get it to balance perfectly and if there are no disturbing forces, the pencil should stay balanced on its tip. But if, for whatever reason, including randomness, it were to tilt slightly in any direction, that unstable state of balanced on its tip will transition to a far more stable state of lying on its side in some a priori unknown direction. I think this is sorta what Shermer means when he says that the union of a zillion different states of something is far more stable than the singular state of nothing.

34: How does one know one is not dreaming? (score 31452 in 2015)

Question

How does one know one is not dreaming? How could one logically demonstrate to a skeptic that one is “really” there, awake and not just dreaming the entire situation/world around him?

Specifically what I’m asking is: which if any philosophers have addressed this problem of how one knows one is or is not dreaming?

Which if any philosophies have attempted to evaluate the sense of claims like “I am not dreaming”?

Answer accepted (score 35)

This is actually an easier question than it seems, largely because it operates on assumptions that are easily conceded to or missed.

The first assumption is that reality is absolutely compartmentalized; it is not.

  • Where does Greek end and Latin begin? That’s a harder question than it looks like if you pay attention to language and there’s a whole book dedicated to studying the indistinction between languages. It’s called Echolalias.
  • Is Beckett’s The Unnameable a book in the same way that Joyce’s Ulysses and Nietzsche’s Will to Power are books? Is an oeuvre everything a philosopher published or do notes and fragments count to? Are these easy questions? Nope, and these are just some of the unitary ideas Foucault pulls apart in The Archaeology of Knowledge. You could say that these details don’t matter if this is but a dream, but then apparently at least your dream would itself call for these distinctions in mode of being, so being in a dream right now wouldn’t matter.

The second assumption is that perception must be observably identical in dream and reality; they are not. For every instance I ask you what you sense or remember in a dream, there are observable regularities that can be distinguished from the regularities of sense in waking life, lucid dreaming only exacerbating that distinction. Even if you’re in a dream right now, then as far as your dream goes, there are two distinct realities which negate the need to even remark on the possibility that this is all a dream.

The third assumption is that absolute necessity exist; it does not. Hume dispelled that idea in his Essay concerning Human Understanding and we haven’t successfully refuted him since.

I hope I’m being helpful.

Answer 2 (score 27)

We simply can’t. We can’t even prove that the Universe was created yesterday along with all memories of the past. We can’t prove the Universe isn’t just a run of a simulation (see the Simulation Hypothesis). If you put it this way, nothing can actually be proven.

Answer 3 (score 18)

Two interesting arguments from recent decades relevant to this are Wittgenstein’s Private Language Argument and the doctrine of Semantic Externalism.

Wittgenstein argues in Philosophical Investigations that it is impossible for there to be a language which only referrs to private, inner sensations. Very roughly, the idea is that there is nothing which could count as misapplying a word used to refer only to an internal mental state. Correct and incorrect depend essentially upon external frames of reference as reflected in the responses of others. The argument is targetted at empiricism, but it is clearly applicable to the skeptic who claims we are dreaming, for it would be impossible, if Wittgenstein is right, to ever refer to one’s own dream experience if dreaming was all one ever knew.

Semantic externalism is a doctrine associated with Davidson, Putnam, Burge and to some extent, Kripke. This is the doctrine that it is an essential component of language that it is not an internal psychological state, that meanings must be grounded in a shared, external world. When you speak words I must take you as referring to something common to both our worlds, or else there would be no basis for successful communication. A shared, outer world is a precondition of communication. The idea is similar to Wittgenstein’s.

I have not presented either argument in any detail, but really have just sketched them in order to answer the question. They are anti-skeptical arguments which apply equally to the claims that we are dreaming, decieved by a demon, or are a brain in a vat. They are widely accepted as effective in proving that any language user cannot have always been dreaming, always deceived about the reference of his words, for language must have been learned in a shared environment. But the arguments cannot really show that I am not dreaming or hallucinating right now. They do however try to show that deception cannot be the norm.

35: Is there a correct or better order to read Aristotle, Plato, and Socrates? (score 30253 in 2011)

Question

I would like to read the text written by Aristotle, Plato, Socrates and so on. Is there any reference of the best order to read them? Is the chronological order the best or correct one? Or is there a best or correct order?

Answer accepted (score 6)

Most philosophers are writing in a given historical background, either consciously within it or against it (Kant was reacting to Hume), so a chronological reading will tend to maintain context.

But each author has written quite a lot, so I’d advise against trying to read the totality of one author before moving on to the next one.

I’d suggest some kind of mix-up that generally follows time and influence, but putting off minor things til later.

That is, start with one or two dialogs of Plato then a chapter or two of Aristotle before trying a pre-Socratic. (note that Socrates is really only presented by Plato)

Another suggestion, if you’re just starting out, is to read an historical commentary along the way. For example, Bertrand Russell’s Intro to Western Philosophy is very readable and gives scope and relation between the main players (sort of a playbook to see were the trends are).

But don’t feel like it is a big slog that you have to get through from start to finish. You don’t have to read Descartes before Spinoza; they all have a tendency to stand on their own. If you like what people have suggested about Nietzsche, then go ahead and read something by him, you don’t have to consume Aquinas first.

Answer 2 (score 3)

With many philosophers, the idea of reading their major works in chronological order is a good one; unfortunately, there are no reliable records as to the order of composition of the works of Plato and Aristotle (and Socrates, of course, left no writings), so you’re completely out of luck in that regard.

I’d suggest you begin with a topic that interests you, and work from there. And, if you have not read them before, I’d definitely suggest using a fair bit of secondary literature to help elucidate; there are many things at play in the works of Plato and Aristotle that may not be obvious on a casual reading.

In addition, if you are new to this, I’d recommend starting with some shorter works, as the texts vary in length significantly.

Answer 3 (score 3)

With many philosophers, the idea of reading their major works in chronological order is a good one; unfortunately, there are no reliable records as to the order of composition of the works of Plato and Aristotle (and Socrates, of course, left no writings), so you’re completely out of luck in that regard.

I’d suggest you begin with a topic that interests you, and work from there. And, if you have not read them before, I’d definitely suggest using a fair bit of secondary literature to help elucidate; there are many things at play in the works of Plato and Aristotle that may not be obvious on a casual reading.

In addition, if you are new to this, I’d recommend starting with some shorter works, as the texts vary in length significantly.

36: Was mathematics invented or discovered? (score 29457 in 2011)

Question

What would it mean to say that mathematics was invented and how would this be different from saying mathematics was discovered?

Is this even a serious philosophical question or just a meaningless/tautological linguistic ambiguity?

Answer accepted (score 127)

Intuitionists” believe that mathematics is just a creation of the human mind. In that sense you can argue that mathematics is invented by humans. Any mathematical object exists only in our mind and doesn’t as such have an existence.

Platonists”, on the other hand, argue that any mathematical object exists and we can only “see” them through our mind. Hence in some sense Platonists would vote that mathematics was discovered.

Answer 2 (score 83)

My personal point of view is that mathematicians invented the axioms and the rules of operation, the rest are discovered. Mathematicians invented the notations for writing down the concepts which are discovered within the universe of an axiom.

The concept of numbers exists, but we invent the notation that the glyph ‘1’ and the sound /wʌn/ refers to the concept of singular object that we discovered. We invented the rules of matrix multiplication, but the consequences of the way we do matrix multiplications are discovered.

Most of the time, we deliberately invent a set of axioms that will lead us to discover a set of facts we want to be true. This is certainly true with imaginary numbers, we invented them so that we can discover the solutions to problems we previously were unable or difficult to solve.

Answer 3 (score 73)

There are things that are discovered, and things that are invented. The boundary is put at different places by different people. I put myself on the list and I believe that my position is objectively justifiable, and others are not.

Definitely discovered: finite stuff

By probablistic considerations, I am sure that nobody in the history of the Earth has ever done the following multiplication:

9306781264114085423 x 39204667242145673 = ?

Then if I compute it, am I inventing it’s value, or discovering the value? The meaning of the word “invent” and “discover” are a little unclear, but usually one says discover when there are certain properties: does the value have independent unique qualities that we know ahead of time (like being odd)? Is it possible to get two different answers and consider both correct? etc.

In this case, everyone would agree the value is discovered, since we actually can do the computation— and not a single (sane) person thinks that the answer is made up nonsense, or that it wouldn’t be the number of boxes in the rectangle with appropriate sides, etc.

There are many unsolved problems in this finite category, so it isn’t trivial:

  • Is chess won for white, won for black, or a draw, in perfect play?
  • What are the longest possible Piraha sentences with no proper names?
  • What is the length of the shortest proof in ZF of the Prime Number Theorem? Approximately?
  • What is the list of 50 crossing knots?

You can go on forever, as most interesting mathematical problems are interesting in the finite domain too.

Discovered: asymptotic computation

Consider now an arbitrary computer program, and whether it halts or does not halt. This is the problem of what are called “Pi-0-1 arithmetic sentences” in first order logic, but I prefer the entirely equivalent formulation in terms of halting computer programs, as logic jargon is less accessible than programming jargon.

Given a definite computer program P written in C (or some other Turing complete language) suitably modified to allow arbitrarily large memory. Does this program return an answer in finite time, or run forever? This includes a hefty chunk of the most famous mathematical conjectures, I list a few:

  • The Riemann hypothesis (in suitable formulation)
  • The Goldbach conjecture.
  • The Odd perfect number conjecture
  • Diophantine equations (like Fermat’s last theorem)
  • consistency of ZF (or any other first order set of axioms)
  • Knesser-Poulson conjecture on sphere-rearrangement

You can believe one of the two

  • “Does P halt” is absolutely meaningful, so that one can know that it is true or false without knowing which.
  • “Does P halt” only becomes meaningful upon the halting of P, or a proof that it doesn’t halt in a suitable formal system, so that it is useful to introduce a category of “unknown” for this question, and the “unknown” category might not eventually become empty, as it does in the finite problem case.

Here is where the intuitionists stop. The famous name here is

  • L.E.J. Brouwer

The intuitionistic logic is developed to deal with cases where there are questions whose answer is not determined true or false, so that one cannot decide the law of excluded middle. This position leaves open the possibility that some computer programs that don’t halt are just too hard to prove halt, and there is no mechanism for doing so.

While intuitionism is useful for situations of imperfect knowledge (like us, always), this is not the place where most mathematicians stop. There is a firm belief that the questions at this level are either true or false, we just don’t know which. I agree with this position, but I don’t think it is trivial to argue against the intutionist perspective.

Most believe discovered: Arithmetic heirarchy

There are questions in mathematics which cannot be phrases as the non-halting of a computer program, at least not without modification of the concept of “program”. These include

  • The twin prime conjecture
  • The transcedence of e+pi.

To check these questions, you need to run through cases, where at each point you have to check where a computer program halts. This means you need to know infinitely many programs halt. For example, to know there are infinitely many twin primes, you need to show that the program that looks for twin primes starting at each found pair will halt on the next found pair. For the transcendence question, you have to run through all polynomials, calculate the roots, and show that eventually they are different from e+pi.

These questions are at the next level of the arithmetic heirarchy. Their computational formulation is again more intuitive— they correspond to the halting problem for a computer which has access to the solution of the ordinary halting problem.

You can go up the arithmetic hierarchy, and the sentences which express the conjectures on the arithmetic hierarchy at any finite level are those of Peano Arithmetic.

There are those who believe that Peano Arithemtic is the proper foundations, and these arithemtically minded people will stop at the end of the arithemtic hierarchy. I suppose one could place Kronecker here:

  • Leopold Kronecker: “God created the natural numbers, all else is the work of man.”

To assume that the sentences on the arithmetic hierarchy are absolute, but no others, is a possible position. If you include axioms of induction on these statements, you get the theory of Peano Arithmetic, which has an ordinal complexity which is completely understood since Gentzen, and it is described by the ordinal epsilon-naught. Epsilon-naught is very concrete, but I have seen recent arguments that it might not be well founded! This is completely ridiculous to anyone who knows epsilon-naught, and the idea might strike future generations as equally silly as the idea that the number of sand grains in a sphere the size of Earth’s orbit is infinite— an idea explicitly refuted in “The Sand Reckoner” by Archimedes.

Most believe discovered: Hyperarithmetic heirarchy

The hyperarithmetic hierarchy is often phrased in terms of second order arithmetic, but I prefer to state it computationally.

Suppose I give you all the solution to the halting problem at all the levels of the arithmetic hierarchy, and you concatenate them into one infinite CD-ROM which contains the solution to all of these simultaneously. Than the halting problem with this CD-ROM (the complete arithmetic-hierarchy halting oracle) defines a new halting problem— the omega-th jump of 0 in recursion theory jargon, or just the omega-oracle.

You can iterate the oracles up the ordinal list, and produce ever more complex halting problems. You might believe this is meaningful for any ordinals which produce a tape.

There are various stopping points along the hyperarithmetic hierarchy, which are usually labelled by their second-order arithemtic version (which I don’t know how to translate). These positions are not natural stopping points for anybody.

Church Kleene ordinal

I am here. Everything less than this, I accept, everything beyond this, I consider objectively invented. The reason is that the Church-Kleene ordinal is the limit of all countable computable ordinals. This is the position of the computational foundations, and it was essentially the position of the Soviet school. People I would put here include

  • Yuri Manin
  • Paul Cohen

In the case of Paul Cohen, I am not sure. The ordinals below Church Kleene are all those that we can definitely represent on a computer, and work with, and any higher conception is suspect.

First uncountable ordinal

If you make an axiomatic set theory with power set, you can define the union of all countable ordinals, and this is the first uncountable ordinal. Some people stop here, rejecting uncountable sets, like the set of real numbers, as inventions.

This is a very similar position to mine, held by people at the turn of the 20th century, who accepted countable infinity, but not uncountable infinity. Those who were here include many famous mathematicians

  • Thorvald Skolem

Skolem’s theorem was an attempt to convince mathematicians that mathematics was countable.

I should point out that the Church Kleene ordinal was not defined until the 1940s, so this was the closest position to the computational one available in the early half of the 20th century.

Continuum

Most practically minded mathematicians stop here. They become wary of constructions like the set of all functions on the real line, since these spaces are too large for intuition to comfortably handle. There is no formal foundation school that stops at the continuum, it is just a place where people stop being comfortable in absoluteness of mathematical truth.

The continuum has questions which are known to be undecidable by methods which are persuasive that it is a vagueness in the set concept at this point, not in the axiom system.

First Inaccessible Cardinal

This place is where most Platonists stop. Everything below this is described by ZFC. I think the most famous person here is:

  • Saharon Shelah

I assume this is his platonic universe, since he say so explicitly in an intro to one of his more famous early papers. He might have changed his mind since.

Infinitely many Woodin Cardinals

This is the place where people who like projective determinacy stop.

It is likely that determinacy advocates believe in the consistency of determinacy, and this gives them evidence for consistency of Woodin Cardinals (although their argument is somewhat theological sounding without the proper computational justification in terms of an impossibly sophisticated countable computable ordinal which serves as the proof theory for this)

This includes

  • Hugh Woodin
Possibly invented: Rank-into-Rank axioms

I copied this from the Wikipedia page, these are the largest large cardinals mathematicians have considered to date. This is probably where most logicians stop, but they are wary of possible contradiction.

These axioms are reflection axioms, they make the set-theoretic model self-simialar in complicated ways at large places. The structure of the models is enormously rich, and I have no intuition at all, as I barely know the definition (I just read it on Wiki).

Invented: Reinhard Cardinal

This is the limit of nearly all practicing mathematicians, since these have been shown to be inconsistent, at least using the axiom of choice. Since most of the structure of set theory is made very elegant with choice, and the anti-choice arguments are not usually related to the Godel-style large-cardinal assumptions, people assume Reinhardt Cardinals are inconsistent.

I assume that nearly all working mathematicians consider Reinhardt Cardinals as imaginary entities, that they are invention, and an inconsistent invention at that.

Definitely invented: Set of all sets

This level is the highest of all, in the traditional ordering, and this is where people started at the end of the 19th century. The intuitive set

  • The set of all sets
  • The ordinal limit of all ordinals

These ideas were shown to be inconsistent by Cantor, using a simple argument (consider the ordinal limit plus one, or the power set of the set of all sets). The paradoxes were popularized and sharpened by Russell, then resolved by Whitehead and Russell, Hilbert, Godel, and Zermelo, using axiomatic approaches that denied this object.

Everyone agrees this stuff is invented.

37: What are the differences between sentience, consciousness and awareness? (score 29344 in 2016)

Question

Dictionary definitions such as this one often seem to use the terms sentience, awareness, and consciousness as if they are synonymous with each other. Is this really the case? If not, how do they differ? Some books suggest that sentience and awareness are simply facets of the larger concept of consciousness.

Answer accepted (score 13)

These are all terms that one frequently reads in texts on Cognitive Science. I will try to find some exemplary definitions:

  1. Consciousness: Many philosophers have argued that consciousness is a unitary concept that is understood intuitively by the majority of people in spite of the difficulty in defining it. Others, though, have argued that the level of disagreement about the meaning of the word indicates that it either means different things to different people, or else is an umbrella term encompassing a variety of distinct meanings with no simple element in common (Wikipedia). You should check the SEP article for there is a lot more to say about consciousness than one of us could actually summarise here.

  2. Sentience is the ability to feel, perceive, or be conscious, or to have subjective experiences. Eighteenth century philosophers used the concept to distinguish the ability to think (“reason”) from the ability to feel (“sentience”). In modern western philosophy, sentience is the ability to have sensations or experiences (described by some thinkers as “qualia”). Sentience is a minimalistic way of defining “consciousness”, which is otherwise commonly used to collectively describe sentience plus other characteristics of the mind. (Wikipedia)

  3. Awareness is the state or ability to perceive, to feel, or to be conscious of events, objects, or sensory patterns. In this level of consciousness, sense data can be confirmed by an observer without necessarily implying understanding. More broadly, it is the state or quality of being aware of something. In biological psychology, awareness is defined as a human’s or an animal’s perception and cognitive reaction to a condition or event. (Wikipedia)

If I’m not mistaken it’s halfway safe to say that awareness and sentience are levels or subclasses of the consciousness. Awareness is mainly the physical act of perceiving, while sentience is a subjective way of actually being affected. Consciousness then is used in many different meanings, but often as a umbrella term for several faculties. Hope I could help, check the links for more information.

Answer 2 (score 2)

I have been recently reviewing the question of consciousness vs. awareness and it seems to me that consciousness includes an element of knowing something that is in awareness while awareness can be sub-conscious as demonstrated by selectively becoming conscious of specific stimuli as they arise which would not be possible without sensory awareness. Based on this consciousness requires awareness but awareness does not require consciousness. Most of what we are aware of is not conscious and usually never will be and some of it cannot be conscious, at least normally, such as sensory processing. Consciousness is a known model of some of what is in awareness which itself is a representation of the output of various forms of unconscious processing. For reference see Psychology: The Study of Human Behaviour by Robert Ornstein an Laura Carstensen (1991). Sentience implies the capability of consciousness or highly responsive awareness.

Answer 3 (score 1)

Drawing on existing contributions, I wonder if introducing the concept of interoperability might lend clarity and a useful organizing stability to the discussion of these interrelated terms? To wit: Consider human development (both of an individual subject and of the specie in toto) and the utility of the underlying characteristics embodied by each of these three terms [probably a separate discussion in its own right].

SENTIENCE seems to carry and connote characteristics which satisfy conditions of minimal interoperability between the individual and a necessary set of others. Developmentally, for example, an otherwise physically nourished individual possess and makes individually beneficial use of what is sensed in the absence of a formal vocabulary and extensive societal protocal for information interchange. One acquires somatisensory input, such as it is and assigns highly personalized, idiosyncratic and subjective meaning to the input.

The need to express or the potential utility derived from sharing and exchanging of such assigned meaning drives the process of abstraction yielding vocabulary of both physical utterance and action that demands organized AWARENESS within the individual’s own mentation and that of other individuals in the relavent social environment. That is to say, a broader plane of interoperability.

Similarly, faced with a more expansive [read, “multicultural”] sphere of potential interchange, abstraction of a more complex nature is required. Introspection and imaginative extrospection are both requied to achieve high degrees of interoperability across boundaries of sophistocated, highly evolved and possibly ridgid systems of social interchange. Not an impossible objective (witness the UN, the OAS, the EEU) but one which demands an individually optimized abstraction of the full range of human experience integrated and shared among a spectum of multiple and distinct social systems. Such a state, IMHO, would qualify as a “CONSCIENCE”. A brightly robust, internally stable CONSCIOUS awareness of one’s own sense of values & assigned meaning as derived through abstract thought processes that are capable of adapting to and integrating with functionally similar processes in a wide range of inhabitants (i.e., across multiple species of living participants) in the relevant ecosystem.

The threshhold CAPACITY of the individual for achieving so developed a state of interoperability, predicated as it is on both sentience and awaress is, I submit, reasonably termed CONSCIOUSNESS.

38: What does Schopenhauer mean by ‘A man can do what he wants, but not want what he wants’? (score 28857 in )

Question

I first encountered the bolded quote on p 80, Philosophy: A Complete Introduction (2012) by Prof Sharon Kaye (MA PhD in Philosophy, U. Toronto), part of Chapter 6 on Thomas Hobbes, Rousseau, free will, determinism, and compatbilism.

[ Reddit :] I do not at all believe in human freedom in the philosophical sense. Everybody acts not only under external compulsion but also in accordance with inner necessity. Schopenhauer’s saying, “A man can do what he [wants]1, but not [want]2 what he [wants]3,” has been a very real inspiration to me since my youth; it has been a continual consolation in the face of life’s hardships, my own and others’, and an unfailing well-spring of tolerance.

I conjecture the 3 wants above mean different desires: 1 and 2 mean primitive superficial urges or John Stuart Mill’s phrase ‘lower pleasures’ (eg:chocolate, guilty pleasures, etc…); but 3 means John Stuart Mill’s phrase ‘higher pleasures’ or second-order virtuous aretes. Did I neglect anything?

Answer accepted (score 9)

This is just an old paradox in discussions of free will.

You are free to do whatever you desire. But you are not free to choose your desires. Similarly, Marx said, “man” makes his own history, but not under the historical conditions of his choosing. And Mill attempted to secularize the paradox by observing that we are slaves to habit, but can step back and form those habits. We can, in some measure, both rely on causes and effects and intervene between them.

The idea, which arises in many forms, is that “freedom” is indeed inevitably paradoxical. There is no such thing as “absolute” freedom nor “absolute” constraint. There are only indeterminacies and determinations on different levels, of which one may or may not be aware.

Answer 2 (score 1)

I think it is quite simple. A man can do what he wants. This, of course, means he is free to choose to do what he likes. A man however, cannot want what he wants. This, to me, means that I cannot have a choice in my desires. If I desire or want a red car, can I feel the same “wanting” for a blue one? Who knows why I want what I want? Can I want something which I never liked? Can I choose my desires? A man cannot want what he wants.

Answer 3 (score 1)

I think it is quite simple. A man can do what he wants. This, of course, means he is free to choose to do what he likes. A man however, cannot want what he wants. This, to me, means that I cannot have a choice in my desires. If I desire or want a red car, can I feel the same “wanting” for a blue one? Who knows why I want what I want? Can I want something which I never liked? Can I choose my desires? A man cannot want what he wants.

39: What is objective morality? (score 28773 in 2018)

Question

It has been argued by some that God is a necessary and sufficient condition for objective morality. I can’t accept this, because I don’t even know what the analysis of moral objectivity is. It seems an ill defined concept. Could someone explain what it would mean for morality to have an objective existence?

Answer accepted (score 12)

A proposition is objective if its truth value is independent of the person uttering it. A fact is objective in the same way.

For morality to be objective, moral propositions such as “Killing is bad”,“Stealing is bad”, etc… need to be true independently of the person who is stating them.

Moral statements are basically statements of value. Some value statements are clearly subjective: “Tabasco flavored ice cream tastes good” can be true for me, but false for you.

The challenge for finding an objective morality (independent of God) is to somehow demonstrate that a moral statement like “Helping others is good” or “Raping is bad” is true independently of the observer or subject of the statement.

There are ways to do this without resorting to God. One example is Kant, with his famous categorical imperatives. His approach to making morality objective rests on idea of whether a course of action can be applied in any situation whatsoever. If so then, it is true in all cases, and it is (objectively) moral.

Another approach is utilitarianism, where instead of trying to analyze the act itself, you measure the outcome: Which course of action leads to the greatest good for the greatest number of people? The objectivity for utilitarians comes from the statistical nature of the way goodness is assessed. Although the outcome of the act is subjective for each person, the overall truth of the statement depends on summing the value of the outcome for all people, and becomes objective that way.

In practice, both Kantian and utilitarian ethics have their challenges, but they are good starting points for an objective morality independent of God (or Gods).

Answer 2 (score 1)

‘Objective’ is open to many interpretations. For an objective morality I should mark out four positions (which isn’t to say there aren’t others).

Moral Realism

This is the view that moral judgements can be true or false; that some are true; and that some are known to be true. The ontology behind this requires really existing moral properties or facts. A moral epistemology is needed to explain how we can become aware of the moral properties or facts by virtue of which we know that some moral judgements are true.

Ethics of divine commands

There is no single, uncontestable statement of this position but it can be roughly stated as follows : divine commands have moral force in and of themselves - in which case at least some (perhaps all) moral obligations derive from divine commands.

It is possible to see an ethics of divine commands as a form of moral realism but moral realism can be held without commitment to an ethics of divine commands. This is why I have separated them.

Moral rationality

Finally, at least in this brief survey, morality can be seen as a form of rationality. This is clear in Kant, whose view is that it is irrational to act on maxims, principles of conduct, if it is logically impossible for everyone to act on such maxims or principles. (Kant’s position is more complex but this is at its heart.)

Kant and truth

Kant’s moral philosophy makes no use whatever of the concept of truth. The objectivity of morality rests on the sole basis that rational agents cannot consistently reject moral requirements. Such requirements derive their full and exclusive force from their rationality. (Brian K. Powell, ’Kant and Kantians on “The Normative Question”, Ethical Theory and Moral Practice, Vol. 9, No. 5 (Nov., 2006), pp. 535-544 : 535.) More specifically, rationality requires me to act on maxims or principles on which it is logically possible for everyone to act.

For instance, suppose I have been invited to a ‘bring a bottle party’ and I decide to take cheap and drink expensive. My maxim is (Kantianly construed) : ‘Whenever one is invited to a bring a bottle party, take cheap wine and drink the expensive wine that others have brought’. It is logically impossible for this maxim to be universalised : if everyone took cheap wine in order to drink the expensive wine brought by others, there would be no expensive wine for anyone to drink.

These are the lines along which Kant’s ethical theory runs. They have nothing to do with truth, everything to do with consistent universalisability.

Utilitarianism

This embodies a form of instrumental rationality. An intrinsic good or range of intrinsic goods, or a a set of intrinsic valuings, is identified. These have in themselves no moral character. Morality comes into play when as a social institution it is given the task of maximising the occurrence of these goods or valuings through the consequences of actions. An action is right if, for instance, it maximises through its consequences the occurrence of such intrinsic goods or valuings as pleasure, health, knowledge or whatever the list of intrinsic goods or valuings comprises. Utilitarian moral judgements are not true, or taken by utilitarians to be true; their moral commendation is their instrumental rationality in the maximising of intrinsic goods or valuings.


References

G. Sayre-McCord, Essays on Moral Realism (Cornell Paperbacks), SBN 10: 0801495415 / ISBN 13: 9780801495410 Published by Cornell University Press, 1988.

Michael J. Harris, Divine Command Ethics, ISBN 10: 1138869767 / ISBN 13: 9781138869769 Published by Routledge, 2015.

D. A. Rees, ‘The Ethics of Divine Commands’, Proceedings of the Aristotelian Society, New Series, Vol. 57 (1956 - 1957), pp. 83- 106.

Onora O’Neill, Constructions of Reason: Explorations of Kant’s Practical Philosophy, ISBN 10: 0521388163 / ISBN 13: 9780521388160 Published by Cambridge University Press, 2016.

Otfried Hoffe, Immanuel Kant (S U N Y Series in Ethical Theory), ISBN 10: 0791420949 / ISBN 13: 9780791420942 Published by State University of New York Press, 1994.

Brian K. Powell, ’Kant and Kantians on “The Normative Question”, Ethical Theory and Moral Practice, Vol. 9, No. 5 (Nov., 2006), pp. 535-544.

Geoffrey Thomas, In Introduction to Ethics, ISBN 10: 0715624318 / ISBN 13: 9780715624319 Published by Gerald Duckworth & Co Ltd, 1997.

J. J. C. Smart, Bernard Williams, Utilitarianism: For and Against, ISBN 10: 0521202973 / ISBN 13: 9780521202978 Published by Cambridge University Press, 1973.

Answer 3 (score 1)

‘Objective’ is open to many interpretations. For an objective morality I should mark out four positions (which isn’t to say there aren’t others).

Moral Realism

This is the view that moral judgements can be true or false; that some are true; and that some are known to be true. The ontology behind this requires really existing moral properties or facts. A moral epistemology is needed to explain how we can become aware of the moral properties or facts by virtue of which we know that some moral judgements are true.

Ethics of divine commands

There is no single, uncontestable statement of this position but it can be roughly stated as follows : divine commands have moral force in and of themselves - in which case at least some (perhaps all) moral obligations derive from divine commands.

It is possible to see an ethics of divine commands as a form of moral realism but moral realism can be held without commitment to an ethics of divine commands. This is why I have separated them.

Moral rationality

Finally, at least in this brief survey, morality can be seen as a form of rationality. This is clear in Kant, whose view is that it is irrational to act on maxims, principles of conduct, if it is logically impossible for everyone to act on such maxims or principles. (Kant’s position is more complex but this is at its heart.)

Kant and truth

Kant’s moral philosophy makes no use whatever of the concept of truth. The objectivity of morality rests on the sole basis that rational agents cannot consistently reject moral requirements. Such requirements derive their full and exclusive force from their rationality. (Brian K. Powell, ’Kant and Kantians on “The Normative Question”, Ethical Theory and Moral Practice, Vol. 9, No. 5 (Nov., 2006), pp. 535-544 : 535.) More specifically, rationality requires me to act on maxims or principles on which it is logically possible for everyone to act.

For instance, suppose I have been invited to a ‘bring a bottle party’ and I decide to take cheap and drink expensive. My maxim is (Kantianly construed) : ‘Whenever one is invited to a bring a bottle party, take cheap wine and drink the expensive wine that others have brought’. It is logically impossible for this maxim to be universalised : if everyone took cheap wine in order to drink the expensive wine brought by others, there would be no expensive wine for anyone to drink.

These are the lines along which Kant’s ethical theory runs. They have nothing to do with truth, everything to do with consistent universalisability.

Utilitarianism

This embodies a form of instrumental rationality. An intrinsic good or range of intrinsic goods, or a a set of intrinsic valuings, is identified. These have in themselves no moral character. Morality comes into play when as a social institution it is given the task of maximising the occurrence of these goods or valuings through the consequences of actions. An action is right if, for instance, it maximises through its consequences the occurrence of such intrinsic goods or valuings as pleasure, health, knowledge or whatever the list of intrinsic goods or valuings comprises. Utilitarian moral judgements are not true, or taken by utilitarians to be true; their moral commendation is their instrumental rationality in the maximising of intrinsic goods or valuings.


References

G. Sayre-McCord, Essays on Moral Realism (Cornell Paperbacks), SBN 10: 0801495415 / ISBN 13: 9780801495410 Published by Cornell University Press, 1988.

Michael J. Harris, Divine Command Ethics, ISBN 10: 1138869767 / ISBN 13: 9781138869769 Published by Routledge, 2015.

D. A. Rees, ‘The Ethics of Divine Commands’, Proceedings of the Aristotelian Society, New Series, Vol. 57 (1956 - 1957), pp. 83- 106.

Onora O’Neill, Constructions of Reason: Explorations of Kant’s Practical Philosophy, ISBN 10: 0521388163 / ISBN 13: 9780521388160 Published by Cambridge University Press, 2016.

Otfried Hoffe, Immanuel Kant (S U N Y Series in Ethical Theory), ISBN 10: 0791420949 / ISBN 13: 9780791420942 Published by State University of New York Press, 1994.

Brian K. Powell, ’Kant and Kantians on “The Normative Question”, Ethical Theory and Moral Practice, Vol. 9, No. 5 (Nov., 2006), pp. 535-544.

Geoffrey Thomas, In Introduction to Ethics, ISBN 10: 0715624318 / ISBN 13: 9780715624319 Published by Gerald Duckworth & Co Ltd, 1997.

J. J. C. Smart, Bernard Williams, Utilitarianism: For and Against, ISBN 10: 0521202973 / ISBN 13: 9780521202978 Published by Cambridge University Press, 1973.

40: How can experts disagree despite having access to the same facts? (score 28220 in 2016)

Question

How is it possible that experts in a certain field can disagree despite given access to the same facts?

For example in science, why is there disagreement when given access to the exact same information? Shouldn’t the scientific method correct the information so that it is at least similar? Thank you for your time

Edit: This is kind of a secondary question (very basic philosophy knowledge layman), but why are there different interpretations of different scientific facts? What causes these differences within people’s interpretations and decision to accept certain theories?

Examples: Scientists disagreeing over string theory, biologists disagreeing over a scientific paper, or scientists disagreeing over global warming. Art critics disagreeing over the quality of a painting, classical musicians disagreeing over an interpretation of a song.

Answer accepted (score 6)

Do not think of the scientific method as a process of deduction.

Nor should it be thought of as only used on typical science topics.

Take any complex system and you can apply the scientific method. For this example we can use an automobile.

Let’s say an automobile has a problem - it is getting poor gas mileage and has low power. Many problems can cause this symptom, and you can develop theories on what the problem(s) are. For instance, a clogged exhaust, exhaust leak, leaky air intake, bad air intake sensor, poor spark plugs, etc. are all possible causes. Once you add data, it may begin to narrow down these theories. Different theories will explain data in different ways (for this application, intake and exhaust theories will have good discussions on fuel trim data).

The same is applicable to science. Geocentric models could explain orbits for a long time - but heliocentric models made the explanation much simpler. Different models of the atom existed, and some models could predict some real world behaviour - until new data from a gold foil experiment left a single theory as the most plausible.

You will see these disagreements occur mainly over incomplete data. A single fact of data can be explained by different theories in different ways. Its not a deductive process. To learn more, I recommend researching the historic development of scientific theories.

Answer 2 (score 3)

As stated, most of your questions don’t strike me as being philosophical ones. How is it possible that experts, say scientists, disagree? Well, presumably this is explained by psychological facts about the scientists in question–for example, although they’re experts, they’re still human beings, so fallible and influenced by irrational biases and prejudices–and facts about the complexity of the domain in question–physics is hard; it’s not always straightforward what theories the experimental data support, for instance.

There are some philosophical questions in the neighborhood of the questions you’ve asked, however. Could two disagreeing experts both be rational? Epistemologists have been interested in this question for quite awhile. If you think that the total evidence uniquely determines which beliefs are rational, you’d probably be inclined to answer the question in the negative. But if you think that evidence can be indeterminate in some way, then you might answer in the affirmative.

ADDENDUM: Let me sketch for you a model of rational scientific disagreement that is actually prevalent in the philosophy of science, epistemology, and decision theory literature, since most of the current responses seem rather far off the mark to me. The model I’ll sketch is sometimes called Bayesianism.

According to Bayesianism, a rational inquirer begins inquiry with a probability distribution, called a prior, over the hypotheses of interest to her. The prior represents the inquirer’s degrees of belief in the hypotheses before collecting any evidence. Orthodox Bayesianism (DeFinetti, Savage, etc.) is extremely subjective in the sense that rationality only requires that these prior distributions actually be probability measures (as opposed to arbitrary set functions). As inquirers collect evidence, they update their priors by a process called conditionalization. Roughly, their updated probability distributions are equal to their prior conditional distributions given the evidence. This is a highly simplified exposition, but will suffice to answer your question.

Now, suppose this model is right, and let us ask again how two rational scientists might disagree on this model. The answer is simple. If Scientist 1 and Scientist 2 have different priors, then, even if they update by conditionalizing on exactly the same evidence, in general their updated probability distributions will be different. On this model, rationality demands only that priors be probabilistically coherent and that they be updated by conditionalization. Since for a given (non-trivial) space of hypotheses there are infinitely many coherent probability measures on that space, it’s easy to find examples of rational disagreement.

Answer 3 (score 2)

Why do scientists disagree over the same given fact?

  1. Scientists might also disagree over whether to count what they see as the “same” one. That is, they might argue whether their different observations equal one same fact.

    • It seems you have in mind a picture like this: a fact is given as an ingredient. There is a recipe to cook the fact. If we want the best (the truth about nature) out of it, why don’t we just follow the best recipe available in the same way?
    • But notice: there are even many ways to characterize a fact. For example, it might be just one of many ways to express phenomena that you “see” a “stone” “falling”. Is the stone “falling”? What does that even mean?
  2. Even if they set aside the problem of defining the “same fact”, scientists may have different “agendas”, i.e., what they want to achieve with their theories.
  3. We can assume: Scientists want their theory to (a) predict future events well, (b) show how things become so, and (c) give us an intelligible explanation for why they are so. And scientists often disagree which theory is the best in terms of (a), (b), and (c).

    • That is, scientists have to choose a theory to explain the given fact. scientists disagree which theory best accommodates the given fact. They see many ways to fit the fact into different theories.
  4. Who knows? Maybe scientists disagree just because they don’t like each other. This may sound ridiculous but it happens very often. (This is one of many explanations – which are not mutually exclusive – of “why” scientists often disagree.)

41: What are some good introductions to philosophy? (score 27521 in 2017)

Question

I’ve been searching the questions posed here, and don’t seem to find one that gives the answers I am looking for. I’ve proposed to start a list on meta but no one proposed anything, so I feel I must ask this question: What are some books generally regarded as good for a general introduction on philosophy? By this I mean good books that can introduce you to philosophy in general and throughout the times, not to some philosophy or period in particular.

This question almost answers mine, except for the fact that most things proposed there are not books, thus not giving me much options to weigh upon. This one regards contemporary philosophy in particular. And this one has the same problem as the first one, plus the fact that it asks for layman’s terms, which is not what I am looking for.

I’ve been tempted to buy Will Durant’s The Story of Philosophy, but am not sure if it is the best book to get started on philosophy (plus, the book has some mixed reviews). I also checked The Oxford Companion to Philosophy, which seems to be unanimously good, but has the format of dictionary/encyclopaedia, which is not exactly what I am looking for (at least as a main source). The Philosopher’s Toolkit also seems to be a good introduction regarding how to do philosophy, rather than to its history and ideals (this also interests me, but I’d rather learn about the philosophers and their theories first).

It is also important to point out that I have little notions (or none at all) on philosophy. I would like to read something that covers from classical to contemporary philosophy generally (although I do not know if there is such a book), so that from there I could pick some author, period or branch of philosophy in particular to read further on. So, to sum up, I would like to ‘hear’ your opinions on what are some books generally regarded as good for a general introduction on philosophy.

Answer accepted (score 18)

I’d go with Simon Blackburn’s Think: A Compelling Introduction to Philosophy (Cambridge University Press 1999).

Here’s the blurb:

This is a book about the big questions in life: knowledge, consciousness, fate, God, truth, goodness, justice. It is for anyone who believes there are big questions out there, but does not know how to approach them. Think sets out to explain what they are and why they are important. Simon Blackburn begins by putting forward a convincing case for the study of philosophy and goes on to give the reader a sense of how the great historical figures such as Descartes, Hume, Kant, and Wittgenstein have approached its central themes. Each chapter explains a major issue, and gives the reader a self-contained guide through the problems that philosophers have studied. The large scope of topics covered range from scepticism, the self, mind and body, and freedom to ethics and the arguments surrounding the existence of God. Lively and approachable, this book is ideal for all those who want to learn how the basic techniques of thinking shape our existence.

It’s not really an introduction to the history of philosophy; it uses a problem-oriented approach, yet it covers most historical classics.

Last, but not least: The book is enjoyable to read.

Answer 2 (score 9)

Sophie’s World by Jostein Gaarder polarises opinion as it is a history of philosophy wrapped up in a children’s story. As long as you feel OK with this mixture (and for me at 40 something it was not a problem) then this offers a gentle introduction. What I found especially good was that the explanations of the various philosophers’ thoughts were put into a historical context.

Answer 3 (score 8)

History of Western Philosophy by Bertrand Russell is usually recommended as a good place to start, haven’t read it myself, but it gives you a thorough overview of philosophy and how it has changed over the times. It’s usually recommended to read an overview book like this before jumping into a specific philosopher’s work.

On a side note: there is a sort of reference guide by Stanford here that you can also browse through to get an overview of some philosophical concepts.

42: What is the difference between “necessary” and “sufficient”? (score 27032 in 2011)

Question

What is the logical difference between something being necessary in order for something else to be true; as opposed to something being sufficient to make something else true. i.e.

Fuel is sufficient to make an internal combustion engine run.

vs

Fuel is necessary to make an internal combustion engine run.

and what are some subtle examples of how the difference between these two things can greatly impact the meaning of a sentence, discussion, or conclusion.

Answer accepted (score 24)

The difference between “necessary” and “sufficient” is the direction of the logical arrow.

If you have A is sufficient for B it means that every time you have A you will have B, without exception:

A ⇒ B

If you have A is necessary for B it means that every time you have B you will have A, without exception

A ⇐ B

So as an example of A being sufficient for B, it is correct to say that every time you (successfully) kill someone, they will be dead, and the assertion that “Person X being killed is sufficient for Person X being dead” would be true. By contrast, it is not correct to say that every time someone is dead, it is because they have been killed. They could have died of natural causes, or there could have been some sort of accident. So the assertion “Person X being killed is necessary for Person X being dead” would be false.

This page has an excellent example of how the difference between these two concepts can change your conclusion. In his answer to the famous Seven Bridges of Königsberg problem, Euler demonstrated that in order to walk across each bridge exactly once it is necessary that the number of places with an odd number of bridges is either 0 or 2. Or put another way, it is necessary that a graph have either 0 or 2 nodes with an odd number of edges for you to be able to draw it without lifting your pen from the paper. However, this is not sufficient to ensure that such a walk or drawing is possible:

Either 0 or 2 places have an odd # of bridges ⇐ You can walk across each bridge once is true

but

Either 0 or 2 places have an odd # of bridges ⇒ You can walk across each bridge once is NOT!

So the difference between the two matters very much. Can you think why? Hint.

Answer 2 (score 4)

Just to add to the answers given, I would like to give a slightly different way to arrive at the same conclusion. My background is in mathematics, and so my answer might seem amateurish from a philosopher’s or logician’s perspective.

Sufficiency

I’ll start with sufficiency, since it is easier. This answer is the same as those given. If p is sufficient for q, then, p is enough to have q. That is, if we have p, we have q, or

p ⇒ q.

Necessity

The statement “p is necessary for q,” means, we must have p to get q. In other words, if we don’t have p, then we can’t have q. In symbols,

~p ⇒ ~q,

where I use ~ to denote negation. The logical equivalent, called the contrapositive, of this is

q ⇒ p.

Necessity and Sufficiency

Thus, p is necessary and sufficient for q means

p ⇔ q.

Another way this is commonly written is

p if and only if q.

It is sometimes abbreviated as

p iff q.

Example of confusing a claim with its converse

Consider the claim “All dogs go to heaven.” This means

you are a dog ⇒ you go to heaven.

That is, being a dog is sufficient for entrance to heaven. The necessity statement, its converse, is this:

you go to heaven ⇒ you are a dog.

It’s possible that you might be a cat in heaven, and so this is not true. The only way this could be true is if there is a law that requires all dogs and only dogs to enter heaven. That is, being a dog is necessary to enter heaven.

This is also sometimes called confusing cause and effect. There is more explanation and examples on this website:

http://www.nizkor.org/features/fallacies/confusing-cause-and-effect.html

Answer 3 (score 4)

Just to add to the answers given, I would like to give a slightly different way to arrive at the same conclusion. My background is in mathematics, and so my answer might seem amateurish from a philosopher’s or logician’s perspective.

Sufficiency

I’ll start with sufficiency, since it is easier. This answer is the same as those given. If p is sufficient for q, then, p is enough to have q. That is, if we have p, we have q, or

p ⇒ q.

Necessity

The statement “p is necessary for q,” means, we must have p to get q. In other words, if we don’t have p, then we can’t have q. In symbols,

~p ⇒ ~q,

where I use ~ to denote negation. The logical equivalent, called the contrapositive, of this is

q ⇒ p.

Necessity and Sufficiency

Thus, p is necessary and sufficient for q means

p ⇔ q.

Another way this is commonly written is

p if and only if q.

It is sometimes abbreviated as

p iff q.

Example of confusing a claim with its converse

Consider the claim “All dogs go to heaven.” This means

you are a dog ⇒ you go to heaven.

That is, being a dog is sufficient for entrance to heaven. The necessity statement, its converse, is this:

you go to heaven ⇒ you are a dog.

It’s possible that you might be a cat in heaven, and so this is not true. The only way this could be true is if there is a law that requires all dogs and only dogs to enter heaven. That is, being a dog is necessary to enter heaven.

This is also sometimes called confusing cause and effect. There is more explanation and examples on this website:

http://www.nizkor.org/features/fallacies/confusing-cause-and-effect.html

43: What was Socrates/Plato’s view on free-will? (score 26734 in )

Question

Socrates (or, rather Plato, through the voice of Socrates) leads us to the conclusion in the dialog Protagoras (see this for example) that our actions are entirely determined by our beliefs of what is the good, and our beliefs of what is the good is determined by our knowledge. In other words, we cannot act against our knowledge of how we should act, and so our actions our determined entirely by our knowledge.

Does this conclusion mean that Socrates (or at least the Socrates Plato presents us with) in general denies free-will? Or is there some other work in which Socrates acknowledges free will?

Answer accepted (score 5)

Socrates’ personality was in some ways closely connected to his philosophical outlook. He was remarkable for the absolute command he maintained over his emotions and his apparent indifference to physical hardships. Corresponding to these personal qualities was his commitment to the doctrine that reason, properly cultivated, can and ought to be the all-controlling factor in human life. Thus he has no fear of death, he says in Plato’s Apology, because he has no knowledge of what comes after it, and he holds that, if anyone does fear death, his fear can be based only on a pretense of knowledge. The assumption underlying this claim is that, once one has given sufficient thought to some matter, one’s emotions will follow suit. Fear will be dispelled by intellectual clarity. Similarly, according to Socrates, if one believes, upon reflection, that one should act in a particular way, then, necessarily, one’s feelings about the act in question will accommodate themselves to one’s belief—one will desire to act in that way. (Thus, Socrates denies the possibility of what has been called “weakness of will”—knowingly acting in a way one believes to be wrong.) It follows that, once one knows what virtue is, it is impossible not to act virtuously. Anyone who fails to act virtuously does so because he incorrectly identifies virtue with something it is not. This is what is meant by the thesis, attributed to Socrates by Aristotle, that virtue is a form of knowledge.

Socrates’ conception of virtue as a form of knowledge explains why he takes it to be of the greatest importance to seek answers to questions such as “What is courage?” and “What is piety?” If we could just discover the answers to these questions, we would have all we need to live our lives well. The fact that Socrates achieved a complete rational control of his emotions no doubt encouraged him to suppose that his own case was indicative of what human beings at their best can achieve.

So Socrates’ view on free will, believing that the unexamined life is not worth living, was the wisdom and will for self-control, which for him required reflection or a conscience, in other words, for socrates free will is impossible without self-control, for people without self control arent capable of free will because being slaves to their passions they lack the free-will required for self-control. His view in the republic, (in the dialogue with Thrasymachus) is in conflict with the libertarian view, where people who are “slaves” to their passions have the right to self-determination, regardless of what their idea of the best life is, and the reason for his disagreement with libertarian free will in the republic is that justice requires reflection, and that the tyrant is a slave to his passions, does not reflect on his actions and believes he has the right to self-determination.

I also find the Stoic view (borrowed from Socrates) on emotional self-control interesting, namely, that all emotion emerges from incorrect judgement and that the person with reason properly cultivated is able to control their emotions because they posses the wisdom to properly put the source of emotional conflict into the correct perspective or context, and therefore able to make the correct decision.


Answer 2 (score 1)

Woah, something I actually read in my one year of college for philosophy.

Now remember folks, this here is pure conjecture.

I would say that Socrates, or Socrates as shown through Plato, don’t put much stock into free will based off that Utopia they(?) came up with. I think they more or less viewed it as a thing people just irrationally do, which honestly is fairly accurate.

I mean, if you look at that class schema that they wanted to implement, it was pretty bad, in terms of what you were allowed and not allowed to do, in the light of free will that is. You would be bred for certain roles in society, and you were demoted, or promoted where necessary. If you started to show certain qualities, they would pick your job for you. Families would be abolished and replaced with the community. Socrates was probably one of the first advocates of Eugenics.

And of course the Philosophers would be the main decision makers.

I mean, his utopia works on a community level, but not on an individual level.

Socrates didn’t care much for the emotional side of people in general. I forget the exact wording, but he pretty much totally disregarded any conclusion one made with their physical senses, stating that they aren’t nearly as trustworthy as your own mind, or at least his mind.

So with all this conjecture, I would say he didn’t care for it much. Edit: So with this conjecture I would say he at least respected it, but thought it was more of a nuisance. Or he thought it wasn’t true free will if they weren’t exercising their logic and reason to decide what they wanted to do.

Also if I got anything wrong, please let me know, and I will change the error of my ways. I mean, it has been a few years since I read Plato’s writings in general.

Lastly, This is just one topic. I’m not totally riffing on Socrates here, only his view on utopia.

44: What is the difference between intension and intention? (score 26125 in 2014)

Question

What is the difference between intention and intension? If one intends to do something is this intent part of the concept of intension?

Answer accepted (score 10)

The “intension” of a concept is its meaning, whereas the “extension” of the concept is the set of the things that fall under that concept. The most helpful way to see the distinction is with a pair of concepts that have the same extension, but different intensions.

For instance “renate” (=“having kidneys”) and “cordate” (=“having a heart”). Everything that is the one is also the other, so the terms are coextensive. Yet there is still clearly a difference in meaning between them.

“Intentional” on the other hand in these contexts usually means “being about something”. (The idea of doing something “intentionally” i.e. on purpose is actually quite different.) Mental states are often intentional, that is to say, usually we are seeing that p, or believing that p, or doubting that p, or whatever. There is some content p to the mental state that it is about.

Answer 2 (score -3)

Intension is a property of an object while intention is a property of a mind.

45: Why do they say I am committing a fallacy when I am just insulting someone? (score 25877 in 2018)

Question

On the internet, one is often accused of committing the so-called ‘ad hominem’-fallacy, which, according to Sikipedia, is defined as

a fallacious argumentative strategy whereby genuine discussion of the topic at hand is avoided by instead attacking the character, motive, or other attribute of the person making the argument, or persons associated with the argument, rather than attacking the substance of the argument itself

I find this a bit strange. No, when I — say — call somebody “an idiot”, I am not necessarily doing it as part of some clever “argumentative strategy” with the ultimate end goal of changing the topic or otherwise diminishing the arguments offered with irrelevant distracting comments. No, I am calling them an idiot because that’s … just my bloody opinion. And opinions is what one shares (along with arguments) in a discussion.

So why do people accuse me of committing a logical fallacy?

In fact, are they not the ones committing a non-sequitur fallacy, as it does not follow from me making an insult that the insult was specifically made in a strategic attempt to discredit their argument?

Basically, my question is, why do people assume that any insult is an ad-hominem fallacy, when, according to the definition, only certain insults are ad-hominem fallacies. The two examples below should demonstrate this distinction. It is fairly obvious which one is a fallacy and which one is an opinion.

Discussion A

Person A: Smoking is healthy because my doctor says so.

Person B: You’re a bloody idiot.

Discussion B

Person A: Smoking is unhealthy because my doctor says so.

Person B: You’re a bloody idiot. Therefore, we can safely assume your argument to be wrong.

Clearly, only person B in discussion B is committing the ad-hominem fallacy. The other person B is just giving their opinion. And yet, you will find that most people on the internet will describe both responses as ad-hominems. Why is this? Is my definition of the fallacy wrong? What’s going on here?

Answer accepted (score 158)

Because they expect arguments, not inflammatory noise
No, I am calling them an idiot because that’s … just my bloody opinion.

Exactly that: just your opinion. An opinion is not an argument; it is not a syllogism, it is not reasoning, it is not fact, it is not evidence, it is not anything other than just that: your [expletive] opinion.

So why do people accuse me of committing a logical fallacy?

It is true that if you state it as such — “It is my opinion that you are [demeaning characterisation]” — then you have not committed a logical fallacy. You have perhaps gone against good form, or maybe broken a code of conduct, but you have not committed a logical fallacy.

So why did they assume that you were putting forth an argument?

It is because if you say it as “You are an idiot”, you have not expressed it as a subjective opinion but as a statement of objective fact. With that you implicitly argue that your comrade in the discourse is not to be taken seriously. The statement therefore becomes a logical fallacy.

It is a fallacy because you have not in any way argued why their arguments are wrong. You have not made a logical statement that refutes their arguments or stance, nor validates your own arguments and/or stance. All you have done is characterise your comrade, and characterisation does not make an argument.

You have directed your argument (“argumentum”) towards (“ad”) the (hu)man (“hominem”) instead of towards their arguments or their stance.

At this point you might want to object and say that you did not explicitly state that it was an argument. But they will make the assumption that this is what you meant anyway, no matter if you intended it to be or not. This is there because in a discourse you are expected to provide arguments pertaining to the discussion you were already having, as opposed to throwing in irrelevant opinions about how you perceive the character of your comrade. Irrelevant opinions are noise; insulting irrelevant opinions are inflammatory noise.

So if you insist on insulting people in a discourse, the alternative to getting called on committing a logical fallacy is to get called on polluting the discourse with inflammatory noise…

…that is to say: trolling/flaming.

I do not know about you, but I much rather prefer being accused of committing a logical error than being accused of being a troll or a flamer.

Answer 2 (score 76)

If an opinion isn’t intended as an argument, it is noise

Frankly, nobody wants to hear your opinion unless you have some point to make about it.

You can have the opinion that a person is an idiot, but if you’re putting in the effort to type it in then you obviously have some point to it.

I don’t state every opinion I have on every post I make on the internet. If I state one of my opinions, it’s because I believe it is relevant. And if you are trying to claim that this insult is relevant to the argument (which you implicitly do by sharing it), you commit ad hominem.

In addition, if you are led to the conclusion that somebody is an idiot, I presume you have a good reason—in particular, the invalidity of their argument. It is far more effective to explain what led you to this conclusion rather than to simply state the conclusion.

Stating a conclusion without explaining how you arrived at it makes it difficult to argue with, and any argument that attempts to purport itself as indisputably true seems to suggest fallacy to me.

Answer 3 (score 23)

Basically, my question is, why do people assume that any insult is an ad-hominem fallacy, when, according to the definition, only certain insults are ad-hominem fallacies.

The simplest answer here is that people do not all assume that any insult is an instance of an ad-hominem fallacy, and those that do are sometimes mistaken. Firstly, an ad-hominem fallacy is not necessarily an insult. Rather, it is an attempt to use some characteristic of the person making an argument to disprove the argument itself.

For example:

A: Islam is a religion of peace.

B: Well, of course a Muslim would say that.

It is not insulting to acknowledge that speaker A is Muslim, but using their Muslim identity as a means of dismissing their argument is an example of an ad-hominem fallacy.

Perhaps your confusion is stemming from the fact that “ad-hominem” has two common usages? One refers to an ad-hominem attack, the other refers to an ad-hominem fallacy. The first usage is one that I have commonly seen online and it is essentially used to call out a person for being excessively insulting or personal. The second would refer to the technical definition of the ad-hominem fallacy.

Regardless, I would not concern myself too much with the parsing of what exactly constitutes which fallacy lest you fall prey to the fallacy fallacy.

46: What kind of government did Aristotle consider the best? (score 25827 in 2013)

Question

If Aristotle was opposed to democracy as an ideal form of government, what kind of government did he consider the best?

Answer accepted (score 8)

Aristotle classified states according to two variables: who holds power? And: in whose interest is it exercised? There are three politically possible answers to the first question (one, some and all:the kingship, aristocracy, and politeia), and two politically possible answers to the second (the holder of power, and everyone). Aristotle treats kingship and aristocracy as an ideal constitution run by morally and practically virtuous people and aiming at the development and exercise of virtue.

The ideal is the state in which the best, who are inevitably few in number, exercise power in the interests of all. However, since that ideal is hard to achieve, and even harder to sustain, Aristotle advocated a form of mixed government, or “politeia”, in which all citizens “rule and are ruled by turn”, and power is monopolized by no particular class. Aristotle was a vigorous critic of democracy.

47: What does Sartre mean when he says people are “condemned to be free”? (score 25565 in 2012)

Question

What did Jean-Paul Sartre mean when he said that because there is no creator, humans are “Condemned to be Free”?

Answer accepted (score 5)

Human being is freedom.

The external world is filled with in-itself being. Consciousness is the only anomaly, and consciousness only manifests itself through human being, insofar as we are aware of it. So, the starting point for an account of human being is in the account of the being of consciousness. We know that the being of consciousness is the consciousness of being, and it will follow that the being of consciousness of humans is the consciousness of being human. This consciousness of being human entails a consciousness of consciousness, and the immediately apparent features of consciousness. Conscious is infinite which means that the options for manifestations of human consciousness are infinite. The feature of having an infinite amount of options for the manifestation of one’s consciousness is what Sartre calls freedom. So, we have determined that the human being is freedom and that humans must be aware that their being is freedom.

This does not mean that human beings have an essence of freedom. “Man does not exist first in order to be free subsequently; there is no difference between the being of man and his being-free” (BN: 60). Freedom is the reason that human beings do not have an essence. “Human freedom precedes essence in man and makes it possible; the essence of the human being is suspended in freedom” (BN: 60). The freedom of human being manifests itself as the limitless choice of human action. No matter what the situation is, a human being can always choose to act and his action will define his being. Even in extreme situations of coercion (such as being threatened with death), a human being still has the ability to choose his action and to choose the conscious attitude with which he apprehends the world. This ability to choose is actually an inability not to choose. Sartre tells us that “we are a freedom which chooses, but we do not choose to be free” (BN: 623). We are not free beings. We are freedom itself.

Citations refer to this translation of Being and Nothingness

Answer 2 (score 4)

I don’t want to simplify, but this almost seems to be a logical blatancy: As a human being without a moral superior, you are forced (“condemned”) to choose. Even not choosing is a choice. A moral superior provides a) rules you have to follow to achieve b) a certain objective. Following these rules and knowing what you’re doing it for often makes choice a lot easier. Which is why the atheist is endued with condemnation - because it makes his/her life so much more troublesome and complicated.

Answer 3 (score 0)

(Disclaimer: beginner’s view here. Proceed with caution.)

In the views of Sartre, the world’s nature is strict and rigorous, even on a metaphysic point of view. Even with a dualist if point of view and certain influence (“each thing is defined by what is not that thing”) he realizes that the true nature of every existence is bound by the rules of that nature, and there is no way in which that may change. If it changes, then it was not true essence.

As such, human seems to be different, because our awareness allows us to have free will and choose what our actions will be. If we can accept at this capability comes from a superior being that transcends our nature, then there’s a chance, by those divine ways, that we may transcend it as well. In the absence of such a chance, human nature is, in Sartre’s point of view, not different than any other nature, and our free will is nothing but an illusion of choice. (This concept would be enforced by behavioral psychology.)

In a way, we are forced to exist, and we are forced to be who we are. We may choose to change, but as so, we were meant to change, as it was in our nature all along. We are then, “condemned to be free” and forced to choose, because it is in our nature.

48: Why distrust our senses? (score 25426 in 2014)

Question

It seems self-evident that the phenomena we sense are accurate and correlate to the real world.

What sorts of philosophical arguments might cast doubt on this conviction in the veridicality of perception?

Answer accepted (score 11)

“Distrust your senses” is a very long tradition. Recall Plato’s “cave” analogy from the early dawn of philosophy, ~400 BC.

Plato postulated that there is a reality outside of what humans experience. He compared the human “experience through the senses” to the experience of a caveman looking at a shadow play on the cave wall: The caveman can only see the shadows on the wall, he/she has never experienced anything else, and believes that those shadows are all that there is of reality.

Plato says that there’s a reality outside of that: For the caveman, there’s someone or something outside the cave that’s projecting those shadows. There’s a sun (or something) that provides the light, there’s a whole world out there, but the caveman looking at flickering shadows on the wall may not even realize that the outside world exists. The caveman’s senses (and of course, by analogy, any human’s senses) can only tell that “this thing here is a shadow” and “that thing over there isn’t”. Plato’s argument is that it doesn’t matter how accurately our senses can detect shadows on the wall: The argument is that there is a reality outside of what our senses can perceive.

It’s not hard to make modern analogies:

Suppose someone were brought up from birth exclusively on Hollywood movies, and had never seen the outside world. Would that someone be aware of the existence of scripts, cameramen, film sets, directors, actors, people whose only job is to do makeup or lighting? Or would they assume that whatever Hollywood showed them was reality?

Suppose that I’m watching a movie, and my senses tell me that this guy that they call the Joker is a very bad guy, while this other guy called the Batman is a nice guy and is going to save us all. What exactly does that sensory input (and my interpretations of it) tell me about reality?

Suppose that we all live in the Matrix, and that all our sensory inputs are created by somebody else, for some unfathomable (to us) purpose. Is there any way that an “I” living inside the Matrix could tell that my sensory experiences differ from an “I” living outside the Matrix?

That’s the basic argument of Descartes: How can my senses tell me whether I even exist, given that there might be a god somewhere dreaming the entire world, and we’re all living inside the god’s dream?

Descartes famously answered “I think, therefore I am”: If I have more or less control over my own thoughts, nothing else matters, and I have an existence independent of whether it’s inside a dream. (I’m not arguing whether this is a good or bad argument, I’m just saying that historical philosophers have struggled with these questions. I can point out that Neo in The Matrix might very well ask himself the same. The external agent has changed, from God to hostile AI, but the question remains the same.)

Suppose that you people live in the real world, but I personally happen to have a very fond relationship to drugs and magic mushrooms, and every bit of my senses are telling me that there’s an extremely lifelike dragon attacking me right now. Should I trust my senses?

There are lots of reasons to distrust our senses in general, even primary sense experiences like color and lightness. Illusionists, optical illusions, and drugs, show that even immediate senses can be fooled. Physics shows that there’s a lot of the world that humans were hardly even aware of: Ultraviolet, X-rays, radio waves, radioactivity; none of which is directly perceptible by humans, but it’s still real. So our direct sensory impressions can’t be trusted too far either. That’s perhaps a more modern argument, and these days it’s not hard to find examples where our immediate sensory impressions are simply wrong.

Kant says that there’s the “world as it is”, which humans can never actually know, and there’s the “world as it appears to me”. We try the best we can to extrapolate our “here and now” limited sensory experience to rules applying to the world in general, but we’ll never be able to know the “real” reality, we’ll only know the simplified model we’ve built up inside our head based on our limited experience. And there is no amount of experience that will let us know everything.

The original philosophy argument, from Plato 2500 years ago, didn’t doubt sense impressions as such: The cave analogy assumes that the sense impressions of the caveman accurately reflected the shadow play on the cave wall. The philosophical objection is that there might be an entire world outside of what humans happen to perceive, for some reason that the observer simply doesn’t know about.

Answer 2 (score 7)

The basic response to this question (whether you listen to ancient philosophy, early/late modern philosophy, or pretty much anybody who’s thought much about it) is that “you could always be missing something”.

Carneades: It’s basically impossible to KNOW anything with certainty, because you can never know how much you don’t know. But you have to live life, so just go with what seems the most reasonable to you.

Descartes: No matter how accurate and valid your experiences seem to you, it doesn’t guarantee that you’re not getting something horribly wrong / being tricked / living in The Matrix, etc. That’s okay, I have a special argument about the existence of God that can solve this problem…

Hume: Just because you woke up today doesn’t mean you’ll wake up tomorrow. In fact, it’s impossible to even give a PROBABILITY that you’ll wake up tomorrow - because the past says nothing about the future.

Kant: We actually can’t know much of ANYTHING about true reality (things as they are) - but only subjective reality (things as we see them). That’s okay, because I have a special system on which to build a theory of the human mind, truly universal ethics, and even arguments about the existence of God…

… and so on and so forth.

Answer 3 (score 5)

They don’t; that is not all do; for example the natural philosophers; most, of whom are now called scientists, and in antiquity physilogoi took their sense on trust; if you are going to physics as Galileo or even as Einstein did - you’d better be able to trust your measurements.

It was Descarte that popularised the view that one shouldn’t trust one sense; but this is a travesty of his views; he was looking for a secure point from where to begin his theory of true and justified knowledge.

49: What’s the difference between a ‘duty’ and ‘obligation’? (score 25417 in )

Question

Background

I enrolled in a class, The Philosophy of Human Rights. The authors of the course readings never use ‘duty’ and ‘obligation’ interchangeably, so I suspect that the terms may have distinct meanings.

Question

Do the terms ‘duty’ and ‘obligation’ represent different concepts to philosophers? If so, what do the terms mean?

My attempt to answer the question

I thought that a duty might be an obligation that entails action, and that an obligation is a more general term that may or may not entail action.

However, one author, Henry Shue, mentions a ‘negative duty’, such as a duty to respect a person’s right to liberty: In essence, a duty to do nothing to someone. I don’t consider actions to include ‘doing nothing’ (that’s ‘inaction’), so I don’t believe that my definition of ‘duty’, as ‘an obligation that entails action’, is correct.

What’s the difference between ‘duty’ and ‘obligation’?

Thank you.

Answer accepted (score 3)

It would be interesting to have some passages to compare from these authors that you speak about. I too would say, intuitively, that they can’t mean exactly the same. So I put my head into some books (not really) to see what I can come up with. I do not find my finding interesting enough to earn a bounty, but nonetheless I prefer to share.

What I did find interesting was this distinction by Cicero from the Wiki on “duty”:

Cicero, an early philosopher who discusses duty in his work “On Duty”, suggests that duties can come from four different sources:[2]
as result of being human
as a result of one's particular place in life (one's family, one's country, one's job)
as a result of one's character
as a result of one's own moral expectations for oneself

It also states:

Various derivative uses of the word have sprung from the root idea of obligation, a concept involved in the notion of duty; thus it is used in the services performed by a minister of a church, by a soldier, or by any employee or servant.

The concepts thus seem to be interwoven. The Wiktionary on obligation and duty provided me with information on the etymology of the words.

Obligation: From Latin obligatio, from obligatum (past participle of obligare), from ob- to + ligare to bind, from Proto-Indo-European *leig- (“to bind”).

Duty: From Middle English duete, from Old French deu (“due”), past participle of devoir (“to owe”), from Latin debere (“to owe”), from de (“from”) + habere (“to have”).

But what’s really interesting is this Pdf on legal terminology I found. It states that even from a legal point of view, there is uncertainty about the use of these terms. I quote:

The terms ‘obligation’ and ‘duty’ are sometimes used as synonyms. They refer either to the entire contractual relationship between the parties or, more narrowly, to what is due by the obligor to the obligee.

Under American law and English law, as under French law, an informal consensus would appear to have emerged, to the effect that the terms ‘duty’ and ‘obligation’ are synonymous.

For instance, the term ‘obligation’ in the singular or ‘obligations’ in the plural is univocal when it refers to what one party has agreed to perform under the terms of an agreement. In this sense, the positive counterpart of the obligation is the right (‘rights and obligations’), that is to say what the creditor is entitled to receive from the debtor. This is a classical view of the term ‘obligation’ seen as ‘a tie which exists between at least two individual persons which enables one person to request something from the other’1. The obligation should therefore be perceived as including a legal tie, a legal tie between at least two persons and a coercitive power enabling the enforcement of the obligation.

The area covered by duties is wider than that covered by obligations. A duty may be owed to a person other than the other party to the contract. This distinction is in fact applied in English law, in order to define the duty of confidentiality.

It would be useless to quote the whole paper, though I really think that it might answer your question. That leaves me to say that, though it might have seemed to you that they’re not used synonymously, both the encyclopedia and the legal praxis claim they pretty much are. As you already figured out, your first thought, namely that “duty” might entail an action, is not right, because of the negative duties. Both concepts can refer to the human as human or to specific qualities. Both have, even etymologically, a reference to the object of the duty/obligation. No difference, even there. There might be a little legal difference, for it seems you can have duties that aren’t part of the contract’s obligations, but that should have no impact on philosophical jargon, I reckon.

Answer 2 (score 3)

It would be interesting to have some passages to compare from these authors that you speak about. I too would say, intuitively, that they can’t mean exactly the same. So I put my head into some books (not really) to see what I can come up with. I do not find my finding interesting enough to earn a bounty, but nonetheless I prefer to share.

What I did find interesting was this distinction by Cicero from the Wiki on “duty”:

Cicero, an early philosopher who discusses duty in his work “On Duty”, suggests that duties can come from four different sources:[2]
as result of being human
as a result of one's particular place in life (one's family, one's country, one's job)
as a result of one's character
as a result of one's own moral expectations for oneself

It also states:

Various derivative uses of the word have sprung from the root idea of obligation, a concept involved in the notion of duty; thus it is used in the services performed by a minister of a church, by a soldier, or by any employee or servant.

The concepts thus seem to be interwoven. The Wiktionary on obligation and duty provided me with information on the etymology of the words.

Obligation: From Latin obligatio, from obligatum (past participle of obligare), from ob- to + ligare to bind, from Proto-Indo-European *leig- (“to bind”).

Duty: From Middle English duete, from Old French deu (“due”), past participle of devoir (“to owe”), from Latin debere (“to owe”), from de (“from”) + habere (“to have”).

But what’s really interesting is this Pdf on legal terminology I found. It states that even from a legal point of view, there is uncertainty about the use of these terms. I quote:

The terms ‘obligation’ and ‘duty’ are sometimes used as synonyms. They refer either to the entire contractual relationship between the parties or, more narrowly, to what is due by the obligor to the obligee.

Under American law and English law, as under French law, an informal consensus would appear to have emerged, to the effect that the terms ‘duty’ and ‘obligation’ are synonymous.

For instance, the term ‘obligation’ in the singular or ‘obligations’ in the plural is univocal when it refers to what one party has agreed to perform under the terms of an agreement. In this sense, the positive counterpart of the obligation is the right (‘rights and obligations’), that is to say what the creditor is entitled to receive from the debtor. This is a classical view of the term ‘obligation’ seen as ‘a tie which exists between at least two individual persons which enables one person to request something from the other’1. The obligation should therefore be perceived as including a legal tie, a legal tie between at least two persons and a coercitive power enabling the enforcement of the obligation.

The area covered by duties is wider than that covered by obligations. A duty may be owed to a person other than the other party to the contract. This distinction is in fact applied in English law, in order to define the duty of confidentiality.

It would be useless to quote the whole paper, though I really think that it might answer your question. That leaves me to say that, though it might have seemed to you that they’re not used synonymously, both the encyclopedia and the legal praxis claim they pretty much are. As you already figured out, your first thought, namely that “duty” might entail an action, is not right, because of the negative duties. Both concepts can refer to the human as human or to specific qualities. Both have, even etymologically, a reference to the object of the duty/obligation. No difference, even there. There might be a little legal difference, for it seems you can have duties that aren’t part of the contract’s obligations, but that should have no impact on philosophical jargon, I reckon.

Answer 3 (score 2)

Well this has been subject to various philosophical debates for a very long time. If you look at the definition of the two terms, you will get what seems like a circular answer. One refers to the other and the other refers back, case in point:

Definition of duty (n)

  • obligation: something that somebody is obliged to do for moral, legal, or religious reasons
  • need to meet obligations: the urge to meet moral or religious obligations
  • allocated task: a task or service allocated to somebody, especially in the course of work
  • Synonyms: responsibility, obligation, onus, burden, calling, liability

Definition of obligation (n)

  • duty: something that must be done because of legal or moral duty
  • state of being obligated: the state of being under a moral or legal duty to do something
  • gratitude owed: something that somebody owes in return for something given, e.g. assistance or a favor
  • Synonyms: duty, responsibility, requirement, compulsion, commitment, onus

Now you can see that in a way they are being defined interchangeably. However it seems that what separates the two are a matter of perspective and understanding of its context. Ironically what brings more light to this difference are the synonyms.

Observe that while some of the synonyms are the same, given the way they are defined not a real surprise, there are key differences in there that if they were truly the same would not happen.

The primary differences are:

  • burden (duty)
  • calling (duty)
  • liability (duty)
  • requirement (obligation)
  • compulsion (obligation)
  • commitment (obligation)

The trend of the differences for duty seem to imply that some part of the social contract is responsible for it. Duty to vote can be a burden to some and liability to others (the disenfranchised vs. those seeking future political office). Calling to be a soldier, cop, doctor, journalist, humanitarian, activist, etc etc (suggesting a familial trend - my father, his father, and his father ^n was a soldier so its my calling to serve, etc). So we can say perhaps that DUTY is born out of ideology or social contract which motivates the action with a sense of implied greater purpose.

Now looking at the differences for obligation seem to imply imposition of some of kind of responsibility out of legal, moral, or religious rules/laws. Obligation to obey traffic rules, caring for your offsprings, going to church, academic honor to not plagiarize or cheat, do not steal, do not kill, etc, you get the idea. We can perhaps see that obligation tends to come out of legal responsibility, moral or religious beliefs. The differences may seem subtle but they do include nuances that may be significant to how the individual behavior is guided.

50: How empiricism and positivism is distinguished? What’s their differences? (score 25316 in )

Question

According to Wikipedia,

Empiricism is a theory that states that knowledge comes only or primarily from sensory experience. One of several views of epistemology, the study of human knowledge, along with rationalism and skepticism, empiricism emphasizes the role of experience and evidence, especially sensory experience, in the formation of ideas, over the notion of innate ideas or traditions; empiricists may argue however that traditions (or customs) arise due to relations of previous sense experiences.

Positivism is the philosophy of science that information derived from logical and mathematical treatments and reports of sensory experience is the exclusive source of all authoritative knowledge, and that there is valid knowledge (truth) only in this derived knowledge.

Both of them seem same to me. I want to know what’s the differences and field of applications with examples.

Answer accepted (score 5)

Empiricism is the thesis that knowledge comes from experience. Logical positivism is the thesis that the meaning of a sentence is the set of conditions under which that sentence could be verified to be true. (This implies that any sentence which cannot be verified empirically is either meaningless nonsense, or a tautology.) All logical positivists are empiricists, but not all empiricists are logical positivists. For instance, Aristotle would agree that all knowledge comes from sensation. (To my knowledge, he is actually the first to make this claim.) However, Aristotle might well reject that a sentence is meaningful even if there is no way for us to tell whether it’s true or not. (I can’t think of any examples of sentences that Aristotle would take to be like that, but I also can’t think of any reason he would have to deny this either.)

Answer 2 (score 6)

Empiricism is the thesis that knowledge comes from experience. Logical positivism is the thesis that the meaning of a sentence is the set of conditions under which that sentence could be verified to be true. (This implies that any sentence which cannot be verified empirically is either meaningless nonsense, or a tautology.) All logical positivists are empiricists, but not all empiricists are logical positivists. For instance, Aristotle would agree that all knowledge comes from sensation. (To my knowledge, he is actually the first to make this claim.) However, Aristotle might well reject that a sentence is meaningful even if there is no way for us to tell whether it’s true or not. (I can’t think of any examples of sentences that Aristotle would take to be like that, but I also can’t think of any reason he would have to deny this either.)

Answer 3 (score 5)

Empiricism is the thesis that knowledge comes from experience. Logical positivism is the thesis that the meaning of a sentence is the set of conditions under which that sentence could be verified to be true. (This implies that any sentence which cannot be verified empirically is either meaningless nonsense, or a tautology.) All logical positivists are empiricists, but not all empiricists are logical positivists. For instance, Aristotle would agree that all knowledge comes from sensation. (To my knowledge, he is actually the first to make this claim.) However, Aristotle might well reject that a sentence is meaningful even if there is no way for us to tell whether it’s true or not. (I can’t think of any examples of sentences that Aristotle would take to be like that, but I also can’t think of any reason he would have to deny this either.)

51: Who is the Big Other, and is there a little Other? (score 25021 in )

Question

Zizek often draws on Lacan term the Big Other; what is this and what does it mean?

One supposes that there may also then be a little other - is this right?

What would be sensible examples of either kind?

Answer accepted (score 1)

this wiki site is a good Lacan resource, at least - I have encountered it a few times.

http://nosubject.com/index.php?title=Other

The big Other designates radical alterity, an otherness which transcends the illusory otherness of the imaginary because it cannot be assimilated through identification. Lacan equates the big Other with language and the law, and hence the big Other is inscribed in the symbolic order.

So the little other

is inscribed in the imaginary order as both the counterpart and the specular image.

The imaginary is the realm of image and imagination, deception and lure

and so the other is what we imagine other people to be, whereas the Other is their symbolic existence - which is the Otherness that operates within the law.

The law is

the set of universal principles which make social existence possible

and so, the Other is the Other that we interact and are obliged to, etc..

It may help, if this just seems like STUFF, to read an article or two of Lacan’s and see how he uses these concepts. E.g. in the purloined letter.

The first is a glance that sees nothing: the King and the police. The second, a glance which sees that the first sees nothing and deludes itself as to the secrecy of what it hides: the Queen, then the Minister. The third sees that the first two glances leave what should be hidden exposed to whoever would seize it: the Minister, and finally Dupin.

I read the first glance to be the imaginary, the second glance the symbolic, and the third the real. But it was quite some time ago and I should add the caveat that I have been told I was wrong there, by a fan of Zzizek.

Answer 2 (score 2)

The Big Other is simply the symbolic order as it exists for an individual subject (language, law, culture, etc.)

And yes, there is a little other as well. The little others are essentially other individuals.

Lacan writes them “Other” and “other” (Autre/autre in french).

Answer 3 (score 1)

The Big Other is simply the symbolic order as it exists for an individual subject (language, law, culture, etc.)

And yes, there is a little other as well. The little others are essentially other individuals.

Lacan writes them “Other” and “other” (Autre/autre in french).

52: How does Nietzsche define and characterize “freedom” throughout his works? (score 24849 in 2011)

Question

My basic question concerns the meaning of freedom in Nietzsche’s work. Nietzsche suggests that, in reality, a will can never be absolutely “free” or “unfree”—rather, any particular will is going to be strong or weak to some actual degree, ruling indeed though ruled in turn. So (he claims) our “free will” is a “boorish simplicity, a long folly, owing to our extravagant pride”—from Beyond Good and Evil:

The desire for “freedom of will” in the superlative, metaphysical sense, such as still holds sway, unfortunately, in the minds of the half-educated, the desire to bear the entire and ultimate responsibility for one’s actions oneself, and to absolve God, the world, ancestors, chance, and society therefrom, involves nothing less than to be precisely this causa sui, and, with… daring, to pull oneself up into existence by the hair, out of the swamps of nothingness.

But I’d like to contrast this with another short bit from Twilight of the Idols:

Freedom is the will to be responsible for ourselves. It is to preserve the distance which separates us from other men. To grow more indifferent to hardship, to severity, to privation, and even to life itself.

How do those two quotations square with one another? In my (admittedly limited) understanding, the meaning of the will is about the meaning of the Earth, that it is, in its way, a matter for destiny to determine and we must accept its discipline stoically. So with all the irony and caprice of universal history, the future is entirely in our hands, despite the fact that we are not truly free—but in our unfreedom, critical new freedoms are heralded, albeit at the cost and the end of long and painful transformations.

The underlying point seems to be about the possibility of the emergence at long last of nobler spiritualities, an expanded image of thought, etc. Is this interpretation more or less correct? What is the real sense of “freedom” here, or in this oeuvre generally?

Answer accepted (score 15)

Nietzsche has a tenuous relationship with free will. His theories here are fairly difficult, conceptually, to grasp, and I certainly won’t claim that I have a thoroughly complete understanding of it. Also, it’s worth bearing in mind that it isn’t unheard of to find contradictions across Nietzsche’s various works. Even putting aside the very late stage in his life where he becomes maniacal, his books were written at different times, and you can see his thought developing and becoming more refined in the later books, which occasionally contradicts sweeping claims that he made in the earlier books. That’s in part why academic studies of Nietzschean philosophy generally center on a single text.

But as I understand it, the point he’s trying to make in Beyond Good and Evil  is that there is really no such thing as free will. No one can ever be a truly free agent (i.e., no one can truly be sufficiently free to be morally responsible) because such would require that one be causa sui  (or, the cause of oneself), and since we are not causa sui , we cannot be free agents.

There are a couple of different ways that he goes about proving this in BGE, but they’re fairly complex and not particularly relevant here. As a short summary, the first is that it’s logically impossible to be causa sui  (merely claiming that the very concept is “fundamentally absurd” and “the best self-contradiction that has been conceived so far”), and the second is that, based on his previously-developed notion of human agency, that we don’t have sufficient control over our actions in order to claim that we acted out of free will. Or, in other words, that human beings are not self-caused in a sense that is sufficient to underwrite ascriptions or claims of moral responsibility.

He argues that both our moral and religious traditions (particularly Christianity, of which he holds a particularly bitter resentment) conspire to prevent us from ever truly having free will. In fact, he says that what we possess are causally determined wills. One of the aphorisms he uses to prove this asks the question, “Does a Christian want to sin?” Nietzsche disagrees, arguing that a true Christian can never truly want to sin, thus concluding that the Christian never truly has free will, as he was never free in the first place to do whatever he wished.


However, in (primarily) Twilight of the Idols  (and Genealogy of Morals), Nietzsche beings to develop a more positive, productive sense of free will, and more generally, freedom itself. He says that free will is not, in fact, characterized by the ability to do whatever you want, whenever you want, because if it were, it would be a meaningless concept. One who indulges their every whimsy is really just one who is a slave to her own impulses. Suffice it to say that Nietzsche doesn’t think this is a particularly desirable state of affairs.

Instead, he asserts that true free will is more accurately characterized by ambition and achievement. Really, he says, it’s the ability to set a goal and act in such a way as to achieve it. This is also known in his works as the “Will to Power” (not to be confused with the corrupted form later espoused by his sister). Striving to reach the highest possible position in life is the ultimate goal of the will to power, and is in fact itself a manifestation of the will to power. In other terms, the will to power is just agency free will, as opposed to deserts-based free will. And he takes such agency free will to be a rare achievement, as opposed to the natural endowment that is deserts free will, the most commonly conceived form.

Examined again in this light, it becomes apparent that the quote you’ve taken from TI  is really just a restatement or reprisal of the will to power:

Freedom is the will to be responsible for ourselves. It is to preserve the distance which separates us from other men. To grow more indifferent to hardship, to severity, to privation, and even to life itself.

True freedom, from a Nietzschean perspective, is really the will to affirm and to be responsible for oneself. It requires struggle against hardship and an acceptance of life’s pain and suffering in a positive, life affirming way (amor fati ). But it explicitly does not  mean the denial of one’s impulses and instincts. Indeed, it’s the freedom from having to rely on them, but at the same time, the freedom from having to categorically reject them.

(In summary: I think that the two quotes are really talking about two completely different things, although Nietzsche labels them both “free will”. In fact, there are two entirely different notions of free will at play here, one of which Nietzsche harshly criticizes and the other, he exalts.)

Answer 2 (score 1)

The desire for “freedom of will” in the superlative, metaphysical sense, such as still holds sway, unfortunately, in the minds of the half-educated, the desire to bear the entire and ultimate responsibility for one’s actions oneself, and to absolve God, the world, ancestors, chance, and society therefrom, involves nothing less than to be precisely this causa sui, and, with… daring, to pull oneself up into existence by the hair, out of the swamps of nothingness.

In (so-called) primitive societies religion is pervasive, one owes thanks to ones ancestors, to gods or the God. Of this Nietszche approves. The ‘half-educated man’ is a certain figure of the Romantic who emancipates himself from this posture of submission and renouncing and dissolving all ties looks back into the abyss that has thus opened up ‘from above’ as Caspar David shows, in the mountains the wayfarer rests above the sea of fog.

But is this not also the position of the ‘spirit that moves upon the deep’ as in Genesis? Now, suddenly, we see the wayfarer is etherealised into Noble Spirit, the uncaused cause of himself, and the abyss, the deep, his to mold into a world.

Nietsche finds all this risible - for this is high hubris - as the Greeks & Judiacs knew all too well - Oedipus suffered from it: This New Type of Man having kindled a Enlightment using his bare hands hoists himself above the learning of Antiquity, into a new space pregnant with possibilities, he imagines himself as master (of himself) & creator (of a new world). But man is not metaphysical, but physical; he is substantial but he isn’t the uncaused substance, the causa sui itself

Or as Shakespeare put it (in Measure for Measure):

Man, proud man! Drest in a little brief authority

Most ignorant of what he’s most assured,

His glassy essence

Like an angry ape

Plays such fantastic tricks before high heaven

As makes the angels weep

But abjuring such impossible & fantastic flights ego-fantasy we return to the image of the wayfarer - weary and stoic - having climbed so far - he has endured hardship, silence & solitude; and having endured becomes indifferent. He is the figure of the ascetic enduring privation, and the figure of the mystic practising austerities and able to consider a calm contemplation the gap that separates himself from the world - the sublime abyss - and yet able to imagine himself into the world, and the world inside of himself. This is the proper sense of freedom; a freedom that knows its place. Or:

Freedom is the will to be responsible for ourselves. It is to preserve the distance which separates us from other men. To grow more indifferent to hardship, to severity, to privation, and even to life itself.

54: Why do humans desire power, and are we less independent than we think we are? (score 23788 in )

Question

Throughout history we have seen the general population wanting to look up to some leader, some god – wanting some kind of leadership and something to place their worries upon. Individuals who notice humans’ susceptibility to this “phenomenon” have managed to use it to their advantage, such as by using religion to control a mass of people, such as by managing to impose and portray themselves as an all powerful leader where everyone can look up to and place their worries upon. I have nothing against religious beliefs, but like almost anything else, it can be abused and throughout history, people have used it to control others.

Which leads me to three separate points:

  1. Why do us humans desire or in some sense, need a leader, a god or something to look up to and guide us. Can’t we be more independent, enjoy the days that we have on earth and seek peace and happiness? Why do we need anyone or a religion to guide us in our lives?

  2. Why do humans desire power, and control over others?

  3. As Bertrand Russell said,

From the beginning of civilization until the Industrial Revolution, a man could, as a rule, produce by hard work little more than was required for the subsistence of himself and his family, although his wife worked at least as hard as he did, and his children added their labor as soon as they were old enough to do so.

Whilst mentioning

One of the commonest things to do with savings is to lend them to some Government. In view of the fact that the bulk of the public expenditure of most civilized Governments consists in payment for past wars or preparation for future wars,

In bold, it relates to point II), which I wonder, why do humans desire power and control over others?

Whilst Bertrand Russell was talking below about what would happen if no one was compelled to work more than four hours a day, I wonder, why can’t us human beings just enjoy the simple pleasures of life and not crave for power, and hopefully:

Ordinary men and women, having the opportunity of a happy life, will become more kindly and less persecuting and less inclined to view others with suspicion. The taste for war will die out, partly for this reason, and partly because it will involve long and severe work for all. Good nature is, of all moral qualities, the one that the world needs most, and good nature is the result of ease and security, not of a life of arduous struggle.

Answer accepted (score 3)

We’re social primates. Dominance hierarchies provide an effective way to allow directed action by a whole group (because one person is in charge) among other advantages for the group; and there are clear rewards for the dominant individual so it makes sense to want to be that individual if one can. Things will be bad for you if you don’t recognize the authority of that individual, though, so you need to be emotionally prepared for accepting authority also.

Answer 2 (score 1)

Why do we need anyone or a religion to guide us in our lives?

Anthropological research has shown that there is a deep felt need for religion in all societies. When a society has self-consciously done away with god one has to unearth exactly what is their god. In this situation I am using ‘god’ for philosophy/theology/worldview.

It’s an easy mistake to think that we by our own strength and our own self can come up with a fulfilling and convincing theology. This is the work of many men and many generations and not of the individual thinking in a void. An easy parallel is that of language.

In this paradigm, Athiesm or Epicureanism is a religion as much as say Islam. One can wonder what will become the ‘canon’ for the New Athiesm, or what shape or form will condense as its own rituals and saints to genuflect at. As religion is more often referred to certain established forms of worship, its better perhaps to call them worldviews - but this doesn’t really capture the its force.

Can’t we be more independent, enjoy the days that we have on earth and seek peace and happiness?

This is the fruit or of a complex theology/philosophy. Perhaps better described as a distilled slogan. As one can use a computer without understanding at all the complex nature of how its put together - its physics, chemistry, computer science - one can be the beneficiary of an inherited philosophy/theology without being at all aware of all its intricacies or ramifications, or indeed that one has inherited it. (Although, one could say that the seed of all thought is already inside one and it is the conversation of our ancestors that has brought it to flower).

why do humans desire power and control over others

This is difficult.

Answer 3 (score 1)

Why do we need anyone or a religion to guide us in our lives?

Anthropological research has shown that there is a deep felt need for religion in all societies. When a society has self-consciously done away with god one has to unearth exactly what is their god. In this situation I am using ‘god’ for philosophy/theology/worldview.

It’s an easy mistake to think that we by our own strength and our own self can come up with a fulfilling and convincing theology. This is the work of many men and many generations and not of the individual thinking in a void. An easy parallel is that of language.

In this paradigm, Athiesm or Epicureanism is a religion as much as say Islam. One can wonder what will become the ‘canon’ for the New Athiesm, or what shape or form will condense as its own rituals and saints to genuflect at. As religion is more often referred to certain established forms of worship, its better perhaps to call them worldviews - but this doesn’t really capture the its force.

Can’t we be more independent, enjoy the days that we have on earth and seek peace and happiness?

This is the fruit or of a complex theology/philosophy. Perhaps better described as a distilled slogan. As one can use a computer without understanding at all the complex nature of how its put together - its physics, chemistry, computer science - one can be the beneficiary of an inherited philosophy/theology without being at all aware of all its intricacies or ramifications, or indeed that one has inherited it. (Although, one could say that the seed of all thought is already inside one and it is the conversation of our ancestors that has brought it to flower).

why do humans desire power and control over others

This is difficult.

55: What are some criticisms of Epicurus’ “death is nothing to us”? (score 23427 in 2016)

Question

Epicurus famously asserted that death should not be feared, with roughly the following argument:

  1. When we die, we no longer exist;
  2. Since we no longer exist, we can feel neither pain nor pleasure. Rather, we simple “are not”;
  3. Therefore, there is nothing to fear in death, as death literally is nothing from our perspective.

Is this argument logically sound, though? In its brevity it seems to be leaving out a plethora of other considerations that can easily make death a very fearsome thing. For example, one may fear leaving behind one’s family, being forgotten without a legacy, or one may fear “nothing” itself, as “not existing” is a fairly mysterious - and therefore possibly disturbing - notion itself. Or are the former not directly related to death, and is the latter illogical?

In response to one of the answers below, I thought it would be pertinent to clarify my main concern: assuming that in death there is no perception nor experience, what criticisms of Epicurus’ argument remain? I appreciate the answer from the dualist perspective, but I was also hoping for something more scrutinizing of Epicurus’ assertion that “if there is no experience in death, it should not be feared.”

Have any authors written about this?

Answer accepted (score 18)

Yes, many authors have written about this. Shelly Kagan of Yale comes to mind. His famous class on Death should prove to be informative. He also wrote a book. Kagan cites many philosophers (many of which I forget) throughout the series.

Back to Epicurus; his argument is logically sound, except that you misrepresent the corollary. Philosophers that think death is bad think so not because there may be suffering in the afterlife (I don’t know of any philosopher that argues this), but rather because life is precious. As a matter of fact, you could not believe in any sort of afterlife and still believe that death is bad. Losing something precious, is, after all, never a good thing. Whether bad things should be feared or not is another discussion.

FYI, Kagan disagrees. He thinks that death may be good (he provides several very clever arguments) and that even suicide is morally justifiable - a pretty controversial position.

A simple critique of the Epicurean position is the following:

Epicurus: When we die, we no longer exist;
Skeptic: What do you mean by no longer exist?
Epicurus: Our body ceases to function, blood stops flowing, our neurons stop firing, etc.
Skeptic: What about the soul?
Epicurus: It’s destroyed.
Skeptic: How/why/what mechanism destroys the soul?
Epicurus: Well the soul is corporeal. It dies with the body.
And now our Skeptic unveils himself
Descartes: The soul you describe is nothing but an extension of the body. The kind of soul I’m talking about exists even after bodily death and is non-corporeal therefore cannot die in the first place.

So now we’re at the mind-body problem. If you accept Epicurus’ premise (and read what he has to say about the soul) you can circumvent this whole debacle. It’s of note to say that most philosophers nowadays are not dualists and that Cartesian thought is a dying breed. Most, I think, would agree (at least in part) with Epicurus. Kagan certainly does.

Answer 2 (score 2)

Also there is a similar discussion in the Shia Islam’s resources which came in a dialogue between a religious leader, named Ja’far al-Sadiq, and an unbeliever of God and the Judging day. The religous leader asks the unbeliever something like this: “Why do you think people who believe in God and the judging day are dumb? If what you think is true and everything will finish up to nothingness with death then all of you the believers and non-believers would be the same, but if what they say about God and judging day is true then after death you would find yourself in a long journey never got ready for it before whereas they have got ready for it and can easily continue their way toward the uttermost joy ever after, and you will be regretful of the time you have lost here

With respect to the above quoted question I think fear from death would persist as the first assumption that “When we die, we no longer existhas just no guaranty!

Answer 3 (score 2)

Also there is a similar discussion in the Shia Islam’s resources which came in a dialogue between a religious leader, named Ja’far al-Sadiq, and an unbeliever of God and the Judging day. The religous leader asks the unbeliever something like this: “Why do you think people who believe in God and the judging day are dumb? If what you think is true and everything will finish up to nothingness with death then all of you the believers and non-believers would be the same, but if what they say about God and judging day is true then after death you would find yourself in a long journey never got ready for it before whereas they have got ready for it and can easily continue their way toward the uttermost joy ever after, and you will be regretful of the time you have lost here

With respect to the above quoted question I think fear from death would persist as the first assumption that “When we die, we no longer existhas just no guaranty!

56: What is the difference between Functionalism and Structural-Functionalism? (score 22767 in 2012)

Question

I don’t know if I should ask this here because the question is more like a sociology one. Anyway, I will give it a try, since most of the sociologists are also philosophers, so maybe I’ll be lucky finding an answer.

I am preparing for an exam, and I understand really well Lévi-Strauss’s structuralism and I also understand well functionalism(Malinowski).

My problem is that later on in the book I have a question that asks me to explain what Structural-Functionalism is.

I don’t know how to answer that:

-Is that the same as just functionalism or is it a new point of view?

-Could you define or explain it a little bit for me please?

Please don’t just give me a link to Wikipedia, because I’ve already been there and was not helpful to me.

Answer accepted (score 6)

If you find the Wikipedia article on Structural Functionalism too dense, I think the easiest links for you are going to be the Wikipedia articles on Radcliffe-Brown and Malinowski; both of these explain the relationships between (Malinowski’s) functionalism, (Lévi-Strauss’s) structuralism, and (Radcliffe Brown’s) structural functionalism quite concisely.

For example:

In contrast to Radcliffe-Brown’s structural functionalism, Malinowski argued that culture functioned to meet the needs of individuals rather than society as a whole. He reasoned that when the needs of individuals, who comprise society, are met, then the needs of society are met.

Answer 2 (score 1)

Malinowski argued that every institution has FUNCTION to fulfil individual physical and psychological needs like school for Education . In his View NEEDS AND CULTURE Act. As a functional Instrument. . But. Radcliffe- Brown Critic That Biological needs is not same they are changing so In his views Functionalism Merely Depend on Social structure rather than Needs . Both. Worked on how society changes and. Both made functionalism as basis to study. Society but. Has two opinion on is. Needs. And other is social structure.

Answer 3 (score 1)

Malinowski argued that every institution has FUNCTION to fulfil individual physical and psychological needs like school for Education . In his View NEEDS AND CULTURE Act. As a functional Instrument. . But. Radcliffe- Brown Critic That Biological needs is not same they are changing so In his views Functionalism Merely Depend on Social structure rather than Needs . Both. Worked on how society changes and. Both made functionalism as basis to study. Society but. Has two opinion on is. Needs. And other is social structure.

57: Asking a genie for more wishes (score 22671 in 2013)

Question

I just saw this SMBC comic. The second picture looked really promising, but as far as I see it, this attorney screwed up. With his second wish it doesn’t matter whether you say wish or splork, so effectively the genie was forbidding to splork for more splorks as well. Is there a way to ask the genie for infinite wishes in a very clear logical way, that leaves the genie no loophole? (and let’s pretend the genie really let’s you have only three wishes, because in the comic one cannot say how many wishes the genie actually (wanted to) allow for. It is just somewhat implied that it was three so let’s stick with that idea.)

Edit: I acknowledge and appreciate the easy answer of Gugg but if someone could give an answer with the now additional made up - and admittedly kinda dull - restriction, please answer as well: One cannot just tell the genie that what he says just doesn’t count any more at all. I don’t know now if the question is very well defined now, but maybe someone can still make something of it.

Answer accepted (score 2)

If you want to leave no loopholes, I would recommend wishing for a logical system to be true. E.g. you could wish that everything you show in naive logic would come true, and then use Curry’s Paradox to prove whatever you wanted.

Answer 2 (score 3)

This solution uses the term void, which is (also) used in law, thus keeping it sort of in line with the question.

Genie: Fine, fine! So let me guess – your first wish is to void all the wishing rules., right? I’ve been waiting 2,000 years for someone to figure that out.

Aladdin: Yup, and then infinite wishes.

Genie: Granted and… granted…

Aladdin: And I wish for you to prevent me from ever making a bad wish.

Source: CollegeHumor

I think that wish number 3 is actually quite smart, as many wishes are not quite thought through well, and usually backfire when granted (particularly in jokes). The meaning of “bad” is of course discussed elsewhere on Philosophy SE. Also note that these wishes might result in the (perceived) loss of (some) free will (if there was any, perceived or not). Which might (or might not) be “bad” in itself.

Answer 3 (score 2)

A man is granted three wishes by a genie. His first wish is for infinite wishes. The Genie replies: “Sorry, but that is a wish about wishing, a meta wish. You’d need a meta-genie for that.”

58: What did Nietzsche and Marx think of each other? (score 22344 in 2016)

Question

Nietzsche (1844-1900) and Marx (1818-1883) weren’t quite contemporaries, but both were prominent and influential German thinkers, and one might expect that they have at least heard of each other. Marx might have missed Nietzsche’s most active period (1880-1889), but it is certain that Nietzsche had known of Marx’s ideas.

Both were prominent materialists and anti-religious thinkers: “God is dead” vs “Religion is the opiate of the people” and both also subscribed to historicist views on the evolution of human thought. But mostly they seem to be in dialectic opposition to each other.

Marx’s view on the value of community and “From each according to his ability, to each according to his needs” seems antithetical to Nietzsche’s views on master vs slave morality and his ideal ubermensch.

  • Is there a chance that Marx had heard of Nietzsche, or did Nietzsche only gain prominence after Marx’s death?
  • Assuming Nietzsche read Marx, what did he make of his ideas?
  • Is my reading of their ideas as being antithetical correct, or is it superficial, and they can actually be reconciled?

Answer accepted (score 12)

Nietzsche mocked German idealists at length, but I think calling him a materialist is a bridge too far, same as for all his anti-Christianity it is not clear that he was an atheist. He inherited his metaphysics from Schopenhauer, transforming his World Will into will to power, who can be seen as irrationalizing Hegel’s Absolute Geist with a side of that “intellectual intuition” that Kant kept rejecting but couldn’t let go of. Nietzsche’s is a highly personalized and individualistic philosophy focused on human condition and action, like existentialism, barely a realism but hardly materialism, and with panpsychic overtones perhaps.

He explicitly rejected and mocked the dominant version of materialism of his day, atomism. As Nietzsche writes in Beyond Good and Evil:“As regards materialistic atomism, it is one of the best-refuted theories that have been advanced, and in Europe there is now perhaps no one in the learned world so unscholarly as to attach serious signification to it, except for convenient everyday use (as an abbreviation of the means of expression)”. Moreover, he joined Hume in deflating the categories of substance and causality, along with the logical laws of identity and contradiction, all of which he saw as crutches of intellect clinging to the ephemeral stability of Being, and inadequate for capturing the Becoming of life, which only truly manifests itself in willings and urges:“Psychologists should bethink themselves before putting down the instinct of self-preservation as the cardinal instinct of an organic being. A living thing seeks above all to DISCHARGE its strength — life itself is WILL TO POWER”.

Marx passed away in 1883 and Nietzsche started writing about philosophy only in 1878. He probably did not gain enough prominence for Marx to notice in his waning years. As for the influence the other way, Clark and Leiter write in Nietzsche: Daybreak, “there is no evidence, however, that Nietzsche ever read Marx”. He was aware of socialists and Young Hegelians more broadly, like Strauss, Stirner and Feuerbach, and yes, his individualism and emphasis on the historical role of “exceptional individuals”, was antithetical to socialism and historical materialism, and his philosophy of life was antithetical to everything rationalism, especially Hegel. This said, he did embrace the Heraclitean aspects of Hegel, the becoming, the flux of life, irrationalized and vitalized by Schopenhauer. So there is one point of contact between Nietzsche, and especially early Marx of Young Hegelian days and alienation from Economic and Philosophic Manuscripts of 1844 (Stirner wrote in a similar spirit), — the Hegelian dialectic.

See also online discussions concerning Nietzsche on Marx, and Marx on Schopenhauer.

Answer 2 (score 4)

I doubt there is any crossover whatsoever.

Nietzsche’s breakdown was in 1889. Marx died in England in 1883, and was not at all the most prominent socialist of that time. Engels completed Capital in 1894 and “Marxism” only emerged in distinct contrast to other variants of socialism and anarchism around the turn of the century.

What they shared in their views of history, attacks on bourgeois culture, abhorrence of British mechanics, and inheritance of German romanticism is mainly zeitgeist and retrospective selection on the part of later thinkers. Whatever they shared as atheists, they were in wholly different denominations in respect to god’s replacement…Culture.

Answer 3 (score 1)

Nietzsche says in the Genealogy of Morals in dispersed sections in the book:

I have already let it out: in the contractual relationship between creditor and debtor, which is as old as the very conception of a ‘legal subject’ and itself refers back to the basic forms of buying, selling, bartering, trade and traffic.

 

Precisely here, promises are made; precisely here, the person making the promise has to have a memory made for him: precisely here, we may suppose, is a repository of hard, cruel, painful things. The debtor, in order to inspire confidence that the promise of repayment will be honoured, in order to give a guarantee of the solemnity and sanctity of his promise, and in order to etch the duty and obligation of repayment into his conscience, pawns something to the creditor by means of the contract in case he does not pay, something that he still ‘possesses’ and controls…

 

Punishment as a means of rendering harmless, of preventing further harm. Punishment as payment of a debt to the creditor in any form (even one of emotional compensation).

One could compare and contrast Nietzsche’s and Marx’s views on credit. (See especially section 5, second essay, in Genealogy of Morals).

Also, Nietzsche was niether a materialist nor an “immaterialist” because he despised all metaphysics. He saw Metaphysics as that very culprit of making the distinctiom between “appearance” and “reality” which ends up destroying both worlds of metaphysics because philosophers can’t decide if the transcendent (“reality”) world is real or if the phenomenal (appearances) world is real. To get a glimpse of how Nietzsche solves this problem you’re gonna have to read Thus Spoke Zarathustra and Ecce Homo. In Ecce Homo he says: “Zarathustra was the first to see in the struggle of good and evil the true wheel in the working of things — the translation of morality into the metaphysical, as force, first cause, end-in-itself, is his work.”

Nietzsche saw morality and metaphysics as interconnected, so he uses Zarathustra to go beyond morality and thus go beyond metaphysics. You’re gonna have to do a close reading on Thus Spoke Zarathustra on how he does go beyond metaphysics! (You also have to remember materialism and metaphysics are predominantly philosophy of science terms and Nietzsche certainly was not a philosopher of science, so its best not to read him through that lens…)

59: What are examples of analytic a posteriori knowledge? (score 22325 in )

Question

There is the analytic/synthetic distinction and the a priori/a posteriori distinction. These two distinctions form four types of knowledge:

  • analytic a priori
  • synthetic a priori
  • analytic a posteriori
  • synthetic a posteriori

Kant thought analytic a posteriori is self-contradictory. But, some philosophers (e.g. Stephen Palmquist) treat it as valid.

What are examples of analytic a posteriori knowledge? What are their refutations?

Answer accepted (score 11)

Kripke has some examples in his book Naming and Necessity. The proposition Hesperus is Phosphorus (the evening star is the morning star, both being what we call Venus) is one of them. Kripke finds this to be analytic a posteriori because there once was a time in which people thought of Hesperus and Phosphorus as two different stars, later on they found out that they we’re actually the same planet. In this way they necessarily point to the same object but this has been found out through the empirical evidence.

You can read the passage here: http://books.google.com/books?id=04CSCh06t0MC&lpg=PP1&pg=PA102#v=onepage&q=hesperus&f=false

Answer 2 (score 6)

Analytic a posteriori claims are generally considered something of a paradox. First, let’s recall that an analytic proposition’s truth is entirely a function of its meaning – “all widows were once married” is a simple example; certain claims about mathematical objects also fit here (“a pentagon has five sides.”)

So, an analytic proposition is ‘inherent’ in a way that isn’t the case for a synthetic proposition. Consider Kant’s own example of a synthetic proposition: “all bodies are heavy.” The reason this isn’t analytic is that the predicate (“heavy”) isn’t ‘contained’ in the subject (“body”), as it would be for the claim, e.g., that a square is four-sided. There is an inherent ‘ease’ with analytic claims, since all one needs to do to know it is “extract” the predicate from the subject.

Now, the a priori/a posteriori distinction is about whether we know something from experience. This may seem similar but it is distinct from the analytic-synthetic question (which again is about whether the subject contains the predicate or not); note that many a priori claims are also synthetic. Perhaps the clearest examples of a priori claims are mathematical expressions (2+3=5).

Finally, let’s consider the problematic hybrid you have asked after. A proposition that’s analytic a posteriori would contain the predicate within the subject (as ‘triangle’ contains ‘three sides’) but would only be justifiable based on experience. Kant thought this category was paradoxical, as he thinks you never need to resort to experience to justify analytic claims.

However, some modern critics like Stephen Palmquist have argued that in fact philosophy requires these aposterior analytic claims to function in its characteristic ‘hypothetical’ mode:

To begin with, the impossibility of analytic a posteriori knowledge is generally considered to be ‘quite evident’ [P5:182-3]: indeed, it is a nonsensical contradiction in terms for those who equate ‘analytic’ and ‘a priori’ [see Ap. IV]. Even though Kant argues against those who identify analyticity and apriority [e.g., in Kt1:1-10], he joins them in dismissing this class of knowledge with only a brief explanation: ‘it would be absurd to found an analytic judgment on experience. Since, in forming the judgment, I must not go outside my concept, there is no need to appeal to the testimony of experience in its support’ [Kt1:11; cf. Kt2:268 and Kt4:12]. There are, however, a few theorists who do regard the analytic a posteriori as providing the best description of certain types of knowledge.[20] Notwithstanding Kant’s lack of concern for this class of knowledge, I shall argue in IV.3 that certain aspects of his philosophy can best be understood by reinterpreting them in terms of the analytic a posteriori. At this point, though, it will suffice to say that we should expect such knowledge, if it is possible, to have its validity grounded in some way in experience (a posteriori), and yet also to proceed by making inferences solely on the (analytic) basis of an application of the laws of logic to the concepts or propositions involved.

You can read Palmquist’s whole book here. (This section appears in Chapter Four.)

Answer 3 (score 2)

While this question is concerned with explaining and exemplifying the notion of analytic a posteriori knowledge, Saul Kripke suggests his version of what should be necessary a posteriori knowledge and provides examples. Do note the difference between Kripke’s suggestion and what is asked for in the original question, as Necessity and Analyticity are not the same thing.

From the Wikipedia article “A priori and a posteriori”:

… Aprioricity, analyticity, and necessity have since been more clearly separated from each other … Kripke argued that there are necessary a posteriori truths, such as the proposition that water is H2O (if it is true). According to Kripke, this statement is necessarily true (since water and H2O are the same thing, they are identical in every possible world, and truths of identity are logically necessary) and a posteriori (since it is known only through empirical investigation). Following such considerations of Kripke and others (such as Hilary Putnam), philosophers tend to distinguish more clearly the notion of aprioricity from that of necessity and analyticity.

Now, this next paragraph (from the same article and consecutive to the one above) could suggest why this may be relevant to our question regarding analytic a posteriori:

Kripke’s definitions of these terms, however, diverge in subtle ways from those of Kant. Taking these differences into account, Kripke’s controversial analysis of naming as contingent and a priori would best fit into Kant’s epistemological framework by calling it “analytic a posteriori”.

And this is the footnote of the last paragraph:

Stephen Palmquist, “A Priori Knowledge in Perspective: (II) Naming, Necessity and the Analytic A Posteriori”, The Review of Metaphysics 41:2 (December 1987), pp.255-282. See also “A Priori Knowledge in Perspective: (I) Mathematics, Method and Pure Intuition”, The Review of Metaphysics 41:1 (September 1987), pp.3-22. In this pair of articles, Palmquist demonstrates that the context often determines how a particular proposition should be classified. A proposition that is synthetic a posteriori in one context might be analytic a priori in another.

60: Why does the universe obey scientific laws? (score 21715 in 2016)

Question

As far as anyone is aware, the universe consistently acts according to predictable laws (and scientific inquiry exists to determine those laws). Is there any metaphysical reason for this? Is such a question even answerable?

EDIT: I think my question was misunderstood, so I’ll try to clarify. I know about the mathematics question, but this question is, why is the universe consistent? It’s related to the problem of induction: just because all hitherto observed emeralds are green doesn’t necessarily mean that all emeralds are green. Yet, those who have hypothesized that emeralds are green have (thus far) been found to be correct. In other words, as far as anyone can tell, the universe is consistent to the point where much of its behavior is predictable using known laws and statistics. Is there any philosophical discussion regarding why that appears to be the case?

FURTHER EDIT: The question is more fundamental than the simple, ‘why are all emeralds green’, to which the answer is obviously, ‘because if it wasn’t green, we wouldn’t call it an emerald’, and once I formulate the question better, I think that the answer becomes obvious. Let’s use an actual law, F=ma. We’ve checked rocks, we’ve checked feathers; we’ve checked slow moving objects and fast moving objects, and yet, lo and behold, the law always seems to be true, and it’s stayed true for at least a few hundred years (but we can reasonably assume that it was just as true a millennium ago). Now, I ask the metaphysical (in the most literal sense) question: why is this law always true? Why does the universe behave so consistently?

Answer accepted (score 34)

I like immortal squish’s answer, but I’m going to take it a step further.

Physics (and other science) as we know it is a way to describe how the universe behaves. If gravity worked in reverse, but it was consistent about it, that would be the physics. It’s perfectly valid to say that the universe has a set of physics, for example. A different universe could have a different physics - at least in theory.*

However, that doesn’t say anything as to why the universe has a consistent set of physics in the first place. The answer to that is that we don’t know if it does. We only know what the physics in the area we’re able to explore. Light could travel in spirals before it gets within the “bubble” of physics as we know it, where it begins traveling in a straight line. However, there is no evidence for this, nor can there be, by definition - everything in our area behaves consistently, according to the one set of physics.

If physics weren’t consistent from moment to moment, at least in our local area, then it’s highly unlikely anything even remotely resembling intelligent life as we understand it would have been able to emerge. If gravity started varying in how quickly it falls off, or the nuclear force equations changed, or suddenly there were one fewer type of quark, then everything would fly apart, or crush together, or annihilate itself, or so on. That is not to say that there can’t be other physics - just that what we do have has to be consistent.

* Side note: This theory is a big part of how many science-fiction stories get around the apparent restriction on FTL travel - the traveler dips into other universes where the physics allows for it, or pulls some of it here, or otherwise goes “around” physics with other universes/dimensions/etc.


Just to edit in some of the things I’ve been saying in comments:

Is this just the anthropic principle? Sortof, in a much more general form than it’s usually used. We can conceive of intelligence in a different universe with radically different physics. That isn’t to say we can describe how consciousness could exist in a universe where gravity fell off twice as fast or one where atoms didn’t hold together into molecules, but we can conceive that that awareness could exist. (See Boltzmann brains, for example).

What was that about life evolving? I’ve corrected it to “emerge”. I didn’t intend to invoke evolution. Spontaneously generated intelligence (such as the aforementioned brains) would count as “emerging”, but definitely not “evolving”.

Why couldn’t we conceive of such an intelligence? Mostly, because of the lack of the ability to convey information. If fundamental particles (whether or not they’re the same fundamental particles we have) don’t behave in consistent manners, then there’s no way to know anything about them or any larger structures based on them. We see color because of the wavelengths associated with a photon. If that photon suddenly became a proton, it would no longer convey that information. And without the ability to receive information from the environment, there’s no way to observe (in the quantum sense) anything. And intelligent life that is unable to observe is incomprehensible to us.

Answer 2 (score 34)

I like immortal squish’s answer, but I’m going to take it a step further.

Physics (and other science) as we know it is a way to describe how the universe behaves. If gravity worked in reverse, but it was consistent about it, that would be the physics. It’s perfectly valid to say that the universe has a set of physics, for example. A different universe could have a different physics - at least in theory.*

However, that doesn’t say anything as to why the universe has a consistent set of physics in the first place. The answer to that is that we don’t know if it does. We only know what the physics in the area we’re able to explore. Light could travel in spirals before it gets within the “bubble” of physics as we know it, where it begins traveling in a straight line. However, there is no evidence for this, nor can there be, by definition - everything in our area behaves consistently, according to the one set of physics.

If physics weren’t consistent from moment to moment, at least in our local area, then it’s highly unlikely anything even remotely resembling intelligent life as we understand it would have been able to emerge. If gravity started varying in how quickly it falls off, or the nuclear force equations changed, or suddenly there were one fewer type of quark, then everything would fly apart, or crush together, or annihilate itself, or so on. That is not to say that there can’t be other physics - just that what we do have has to be consistent.

* Side note: This theory is a big part of how many science-fiction stories get around the apparent restriction on FTL travel - the traveler dips into other universes where the physics allows for it, or pulls some of it here, or otherwise goes “around” physics with other universes/dimensions/etc.


Just to edit in some of the things I’ve been saying in comments:

Is this just the anthropic principle? Sortof, in a much more general form than it’s usually used. We can conceive of intelligence in a different universe with radically different physics. That isn’t to say we can describe how consciousness could exist in a universe where gravity fell off twice as fast or one where atoms didn’t hold together into molecules, but we can conceive that that awareness could exist. (See Boltzmann brains, for example).

What was that about life evolving? I’ve corrected it to “emerge”. I didn’t intend to invoke evolution. Spontaneously generated intelligence (such as the aforementioned brains) would count as “emerging”, but definitely not “evolving”.

Why couldn’t we conceive of such an intelligence? Mostly, because of the lack of the ability to convey information. If fundamental particles (whether or not they’re the same fundamental particles we have) don’t behave in consistent manners, then there’s no way to know anything about them or any larger structures based on them. We see color because of the wavelengths associated with a photon. If that photon suddenly became a proton, it would no longer convey that information. And without the ability to receive information from the environment, there’s no way to observe (in the quantum sense) anything. And intelligent life that is unable to observe is incomprehensible to us.

Answer 3 (score 14)

The lawfulness of the Universe originated with the philosophy of the Stoics. Prior to the Stoics, there were many philosophies that were rooted in religion and religious ontology. In these, the Universe was lawful because the gods and cosmic scheme operated as they did. In contrast, the Stoics asserted that the Universe was lawful, regardless of the nature or source of those laws:

“Everything is subject to the laws of Fate, for the Universe acts according to its own nature, and the nature of the passive matter it governs.” Wikipedia

The lawfulness of the Universe is not derived from induction, but it does facilitate inductive methods. The lawfulness of the Universe exists prior to – ex ante – to any attempts to understand it, including inductive methods.

Creationists disagree on this. The Creationist view is, taken literally, the laws of the Universe can change at any time and change by any arbitrary degree to bring about the will (end goals) of the Creator. Emphasis: these changes are due to an exogenous Creator. This is not something you can anticipate or understand with the system of the Universe.

The lawfulness of the Universe is rooted in the mechanisms of causation at a physical level. If there is no meta-process of change operating on these mechanisms of causation, then those mechanisms won’t change. Once “Natural Philosophers” let go of religious and Creationist belief systems, there was no theory of meta-process for change. Furthermore, experimentalist who assumed stationary causal processes were not surprised. (Or, any “surprise” was later attributed to mistakes and errors.)

The Medieval philosopher Duns Scotus was perhaps the first to advocate inductive methods to investigate the lawfulness of the Universe. Paraphrasing: “We don’t have to sample every data point, but only a great many. What we find will represent the whole.” And: “We don’t have to keep sampling every possible data point.”

If the Universe is not lawful (i.e “lawful” = same laws operating at all times, in all settings, and independent of all observers), then experimental science would founder. No experiments would be repeatable. Now experimental science is not perfect and not all scientific experiments are easily reproducible. But for the most part experimental science is successful and viable. This doesn’t prove the lawfulness of the Universe, but it adds weight of evidence against the proposition that the Universe is unlawful.

It should be noted that the precursors of the idea of a lawful Universe go back much farther, to Plato’s Ideal Forms, to Pythagoras’ Transcendental Philosophy, and to the Pre-Socratics. What these early systems lacked was an explicit and formal process for specifying mechanisms. Starting with Galileo, Newton, and Kepler, the laws of the Universe could be specified in mathematics in a way that fully characterized the causal mechanisms.

61: Brahma, Brahman, Brahmin, and Atman? (score 21527 in )

Question

In Hinduism and Indian philosophy, what is the difference between Brahma, Brahman, Brahmin, and Atman?

Answer accepted (score 11)

In his commentary on the Brahma Sutras, the Sri-Bhasya, Ramanuja says:

The word Brahman is derived from the root ‘brh’ which denotes greatness, and is therefore applicable to all objects which have the quality of greatness, but more aptly to that object which by nature and qualities possesses this greatness to an infinite degree…

Brahman is not a name, but rather a way to refer to the Absolute, the Supreme Reality of the Vedanta philosophy.

The word Brahma can refer to two things depending upon where it is being used. First, Brahma is the first being created with every new cycle. All beings in a particular cycle are created by Brahma. A different soul becomes Brahma in every new cycle. Brahma is a god, but the gods are office holders so to speak and new souls take the stations in every new cycle. Second, Brahma is one of the three gods making up the Hindu trinity, called the Trimurti, which can be thought of as Brahman (Isvara). Brahma, Vishnu, and Siva make up the trinity and stand for the creative, sustainer, and destroyer aspects of the Godhead.

Brahmin refers to a person who belongs to the priest caste, the highest caste in Hindu society.

Atman is the soul of man. According to the Advaita Vedanta philosophy the soul of man is one with and the same as Brahman. When Brahman within an individual is referred to, it is referred to as the Atman. Thus Atman = Brahman. There are differences between what is meant by ‘soul’ in Hindu philosophy and Christian philosophy. Swami Vivekananda says (Complete Works, V6, p 85):

One of the chief distinctions between the Vedic and Christian religion is that the Christian religion teaches that each human soul has its beginning at its birth into this world; whereas the Vedic religion asserts that the spirit of man is an emanation of the Eternal Being and had no more a beginning than God Himself.

A person has an individual ‘soul’ (in Hinduism - the jiva) that survives death and is the ‘soul’ that passes from one birth to another. The innermost part of the soul is the atman. For a more detailed description see my answer here - https://hinduism.stackexchange.com/questions/6758/where-does-a-soul-attached-to-the-body

For a good description of the the Atman see the following link. There are 2 lectures under the topic Jnana Yoga, titled The Atman and The Atman:Its Bondage and Freedom - http://cwsv.belurmath.org/volume_2/vol_2_frame.htm

Answer 2 (score 5)

  • Brahma is one of the main Hindu deities.

  • Brahman is an abstract concept with a wide meaning. One of its meaning is a hypothetical principle of the universe.

  • A Brahmin is a member of the first of the four classes.

  • Atman again has a wide meaning. One meaning is similar to soul. The main statement of the Indian upanishads is the equation “atman = brahman”. That’s difficult to understand. But a good metaphor for it is Indra’s net, even when the latter is from Buddhism. See Gail Atkins figure and the five bullet points below at http://awakeningtoreality.blogspot.de/2009/04/net-of-indra.html

Answer 3 (score 2)

Brahmin

The Hindu caste system has four Varna :

  • Brahmins: priests, scholars and teachers
  • Kshatriyas: rulers, warriors and administrators
  • Vaishyas: agriculturalists and merchants
  • Shudras: laborers and service providers

The Brahmin caste are thus the caste that produced the clergy in traditional Hindu society.


Brahma

Hinduism has three primary Gods, known as the Trimurti. These Gods are references to the three primary forces in nature that hold the universe together :

  • Brahma (order), the source of creation, can be understood as an anthropomorphic representation of emergence
  • Shiva (chaos), the source of destruction, can be understood as an anthropomorphic representation of entropy
  • Vishnu (balance), the source of balance, can be understood as an anthropomorphic representation of the laws of thermodynamics

Brahma is thus the Hindu representation of the creative force in the universe.


Atman & Brahman

The Atman and Brahman are concepts from the Vedanta branch of Hinduism that represent respectively your individual consciousness and universal consciousness. The Brahman is thus a pantheistic perspective on the divine.

Advaita Vedanta, a school of thought within the Vedanta branch, stresses the notion that the Atman and Brahman are just different manifestations of the same divine essence, which is basically just simplified Animism.

See also my article The Atheistic approach to God… or how to bridge the gap between Atheists and Theists where I explain how Atheism, Pantheism, Hinduism, Animism and Shamanism are really just different perspectives on the same core concepts.

62: Have I found a paradox, or is the universe digital? Or am I just plain wrong? (score 21380 in 2017)

Question

If the universe is analog, there must exist an infinite number of positions. This raises an interesting question.

Let me boil it down to something familiar: a table and an ashtray. I’ll let the ashtray be square, just for convenience.

In an analog universe, the table alone contains an infinite number of locations on its surface. No matter which two points you select, you’ll always be able to point out a location between them. Like the way there are infinite rational numbers between any two rational numbers, unequal to each other.

We imagine the table is 50 x 50 cm, and we say the origin (0,0) is at the lower left. If I put the ashtray on the table, at a random position, what is the probability that it ends up with its lower left corner at the exact coordinate (10,10)?

According to other answers regarding infinity and probability I’ve found on the web, the chances of hitting a certain position out of a quantity containing an infinite number of positions, is zero.

The problem is that all positions on the table have the same probability. That means the probability for the ashtray to hit any position on the table is zero.

Nevertheless, obviously it is possible to place an ashtray on a table. So, in spite of a zero possiblity, one position is chosen anyway. This cannot be!

If the universe is digital (so a minimum distance exists no matter how small), there is no problem at all: the table contains a finite number of positions, all have the same probability greater than zero of being chosen at random, and one of them is selected when I set down the ashtray.

I remember Max Planck concludes something about the radiation of energy, because an object would radiate equal amounts of energy on all wavelengths, and since there are an infinite number of wavelengths, all objects would radiate infinite energy. This could not be true, so Planck concluded that the energy of atoms could only have discrete values.

So, does my example prove the universe to be digital? Or, is the universe analog, and this is a paradox? Or does my logic fail somewhere?

Answer accepted (score 4)

The problem is that all positions on the table have the same probability. That means the probability for the ashtray to hit any position on the table is zero.

Not quite. The probability for the ashtray to hit any given position on the table is zero. But as you have observed, you can most certainly lay the arrow on the table and get a new position.

Nevertheless, obviously it is possible to place an ashtray on a table. So, in spite of a zero possibility, one position is chosen anyway. This cannot be!

Since you are not targetting a given position, there is no paradox. There is zero probability that you will pick up the ashtray and put it back down at the exact same given location, but you can certainly put down the ashtray and get a new position, just like you did the first time.

Answer 2 (score 28)

We run into essentially the same problem almost any time we try to combine the real numbers as described by mathematics with probability theory.

When applying probability theory to something like a coin flip, a die, or a deck of cards, we use what’s known as a Probability Mass Function to assign a probability value to each possible result. What’s the probability rolling this fair six-sided die will give 3? 1/6.

But as you noticed, this type of probability model doesn’t work well when trying to work with real numbers, because there are just too many of them. Since there are more real numbers even within a min-to-max interval than there are natural numbers, if you try to assign a single non-negative probability to each number in the domain and require they add up to 1, almost all of them will need to be zero.

So instead, when dealing with a domain of real numbers, probability theory normally switches to using a Probability Density Function. This function assigns a real value to every possible outcome in the domain, but these values are not probabilities. Instead, to get a probability from a probability density function, you need to take the integral of the function from a start point to an end point, and this represents the probability the result will be between those points.

For a simple example, a process that generates a random real number x chosen out of the interval from 0 to 1 such that the likelihood is the same over the entire interval could be represented by the function p(x)=1. The probability of “x is between 0 and 1/4” is the integral from 0 to 1/4 of 1, which is 1/4. The probability of “x is exactly 1/4”, if meaningful at all, would need to be the integral from 1/4 to 1/4 of 1, which is 0. So even though the probability of any particular exact value is zero, you still have a probability of 1 that the value is somewhere in the interval, and meaningful probabilities for any smaller pieces of the interval you care to calculate.

Real numbers tend to be useful in physics because generally speaking you can divide ordinary quantities of distance, time, mass, and energy into arbitrary intervals (and sometimes non-algebraic numbers like pi and e pop up in the useful models). Of course as modern science ran into observations now attributed to atoms and photons, and developed the Standard Model of particle physics, we needed new theories that no longer assume mass and energy are indivisible on the smallest (non-ordinary) scales. Could the same happen for distance and time? Who knows?

By the way, probability density functions turned out to be rather useful in physics too: basic quantum mechanics describes position and momentum of an object in the form of complex-valued “wave functions” which obey Schrödinger’s Equation. The squares of the absolute values of these wave functions are actually probability density functions for the variable being described. These wave functions do still assume real-valued position, time, and momentum.

Finally, some of the promising looking ideas about quantum gravity (including string theory) suggest there may in fact be a minimum or fundamental unit of space-time, somewhere around the scale of the Planck Length. But as far as I’ve heard, none of these are quite on the level of an established useful theory yet.

Now, all this goes to show that there are ways around the paradox, and ways to still describe our observed universe fairly well using both real numbers and probability. But that doesn’t necessarily imply that’s the true fundamental nature of the universe, or anything like that. It’s also possible to construct working physical laws using just rational numbers, or numbers which are all multiples of some single chosen very small number. (But those systems may not be as easy to calculate with!) All any physical theory or law can claim is that it does well at explaining and predicting observations, and possibly covers more observations than other theories, and/or is simpler and easier to understand and use than other theories.

So when it comes to questions like: Does the universe truly work this way or that? or Does a “better” set of physical theories get us closer to understanding a true underlying nature of the universe? or Why can we predict that things will happen at all? … we’re into the interesting realm of metaphysics. But that’s another story (and one I’m not so qualified to comment on).

Answer 3 (score 14)

We run into essentially the same problem almost any time we try to combine the real numbers as described by mathematics with probability theory.

When applying probability theory to something like a coin flip, a die, or a deck of cards, we use what’s known as a Probability Mass Function to assign a probability value to each possible result. What’s the probability rolling this fair six-sided die will give 3? 1/6.

But as you noticed, this type of probability model doesn’t work well when trying to work with real numbers, because there are just too many of them. Since there are more real numbers even within a min-to-max interval than there are natural numbers, if you try to assign a single non-negative probability to each number in the domain and require they add up to 1, almost all of them will need to be zero.

So instead, when dealing with a domain of real numbers, probability theory normally switches to using a Probability Density Function. This function assigns a real value to every possible outcome in the domain, but these values are not probabilities. Instead, to get a probability from a probability density function, you need to take the integral of the function from a start point to an end point, and this represents the probability the result will be between those points.

For a simple example, a process that generates a random real number x chosen out of the interval from 0 to 1 such that the likelihood is the same over the entire interval could be represented by the function p(x)=1. The probability of “x is between 0 and 1/4” is the integral from 0 to 1/4 of 1, which is 1/4. The probability of “x is exactly 1/4”, if meaningful at all, would need to be the integral from 1/4 to 1/4 of 1, which is 0. So even though the probability of any particular exact value is zero, you still have a probability of 1 that the value is somewhere in the interval, and meaningful probabilities for any smaller pieces of the interval you care to calculate.

Real numbers tend to be useful in physics because generally speaking you can divide ordinary quantities of distance, time, mass, and energy into arbitrary intervals (and sometimes non-algebraic numbers like pi and e pop up in the useful models). Of course as modern science ran into observations now attributed to atoms and photons, and developed the Standard Model of particle physics, we needed new theories that no longer assume mass and energy are indivisible on the smallest (non-ordinary) scales. Could the same happen for distance and time? Who knows?

By the way, probability density functions turned out to be rather useful in physics too: basic quantum mechanics describes position and momentum of an object in the form of complex-valued “wave functions” which obey Schrödinger’s Equation. The squares of the absolute values of these wave functions are actually probability density functions for the variable being described. These wave functions do still assume real-valued position, time, and momentum.

Finally, some of the promising looking ideas about quantum gravity (including string theory) suggest there may in fact be a minimum or fundamental unit of space-time, somewhere around the scale of the Planck Length. But as far as I’ve heard, none of these are quite on the level of an established useful theory yet.

Now, all this goes to show that there are ways around the paradox, and ways to still describe our observed universe fairly well using both real numbers and probability. But that doesn’t necessarily imply that’s the true fundamental nature of the universe, or anything like that. It’s also possible to construct working physical laws using just rational numbers, or numbers which are all multiples of some single chosen very small number. (But those systems may not be as easy to calculate with!) All any physical theory or law can claim is that it does well at explaining and predicting observations, and possibly covers more observations than other theories, and/or is simpler and easier to understand and use than other theories.

So when it comes to questions like: Does the universe truly work this way or that? or Does a “better” set of physical theories get us closer to understanding a true underlying nature of the universe? or Why can we predict that things will happen at all? … we’re into the interesting realm of metaphysics. But that’s another story (and one I’m not so qualified to comment on).

63: What is the meaning of “predicate” in this definition? (score 21146 in 2013)

Question

I have trouble understanding the article on existence in this Philosophy dictionary.

Instantiation in reality, or actual being. Kant pointed out that existence is not a predicate.

What is the meaning of predicate in other words, put simply?

Answer accepted (score 5)

Predicate is a fact about some thing or person, e.g.:

I am tall

I am a man

I am going for a walk

tall, a man and going for a walk are predicates about me. But in

I exist

(according to Kant) exist isn’t a predicate, because if I didn’t exist there wouldn’t be me to apply the predicate to. Cf.:

I don’t exist

Here I isn’t someone at all, while in:

I am not a man

I is someone about whom it is told that she is not a man.

Answer 2 (score 5)

Predicate is a fact about some thing or person, e.g.:

I am tall

I am a man

I am going for a walk

tall, a man and going for a walk are predicates about me. But in

I exist

(according to Kant) exist isn’t a predicate, because if I didn’t exist there wouldn’t be me to apply the predicate to. Cf.:

I don’t exist

Here I isn’t someone at all, while in:

I am not a man

I is someone about whom it is told that she is not a man.

Answer 3 (score 3)

I thought to quote some parts from this helpful website that answers the question above. Beware that I do NOT quote all of the website, only the pertinent parts.

Source:

Predicates

According to Kant the confusion lies in the fact that existence is not a predicate. The predicate is that part of a sentence which is not the subject but which gives information about the subject. A predicate might be a single word like ‘John laughed’ where John is the subject and ‘laughed’ is the predicate. Or a string of words as in the sentence Clare went to school, ‘Clare’ is the subject and ‘went to school’ is the predicate. A predicate is a property that a thing can either possess or lack.

Predicates and the Existence of God

When people assert that God exists they are not saying that there is a God and he possesses the property of existence. If that were the case, then when people assert that God does not exist they would be saying that there is a God and he lacks the property of existence, i.e. they would be both affirming and denying God’s existence at the same time. Kant suggests that to say that something exists is to say that the concept of that thing is exemplified in the world. For Kant, existence is not a matter of a thing possessing a property i.e. existence. Existence is a concept corresponding to something in the world.

Kant’s objection to the ontological argument is that existence is not a property that can be attributed to beings like we can attribute other properties such as being blue, hard, or round. When we talk about entities existing, Kant contends that we do not mean to add existence as a property to their beings. In other words, the objection seems to be that one cannot go around adding existence as a property to God (or anything else for that matter) in order to define God (or anything else) into existence. Unfortunately, defining my bank account as such a place that contains millions of pounds would not mean that a careful understanding of that definition of ‘my bank account’ would really make it so. In order to see if that definition were true, we would have to go to an ATM and check the balance of my account and see if it is accurate. Similarly, a definition of God must be checked with reality to see if it is correct.

The following para discusses contemporary views, and departs from Kant’s use of ‘predicate’.
So I separated it from the above because it’s optional reading.

[…]

Contemporary Views of the Ontological Argument

Kant’s objection has been very influential in the ontological argument debate. Philosopher are still divided as to whether or not existence is a predicate. Some thinkers controversially believe that existence can be thought of as a unique property. A modern advocate of the ontological argument is Alvin Plantinga (b.1932) Professor of Philosophy at Notre Dame University, USA. He has forcefully argued that Kant’s objection does not conflict with anything in Anselm’s argument. For Anselm does not contingently add existence as a property to God and define him into existence. Naturally these objections are contentious, which adds to the intrigue of the ontological argument.

64: What is speculative philosophy? (score 20779 in 2013)

Question

What is speculative philosophy? What might be good or bad about it?

Answer accepted (score 3)

What is speculative philosophy?

Speculative philosophy" on Wikipedia redirects to “continental philosophy”. This makes sense:

The philosopher Charlie Broad distinguishes between “critical” and “speculative” philosophy. Critical philosophy is analysis of the basic concepts of ordinary life and of science, roughly in the tradition of Moore and Russell. The speculative attempt to arrive at an overall conception of the nature of the universe and the position of human beings. Speculative philosophy, is a form of theorizing that goes beyond verifiable observation; specifically, a philosophical approach informed by the impulse to construct a grand narrative of a worldview that encompasses the whole of reality. In its aggressive use of the systematic principle, geared to a unification of human experience, speculative philosophy aspires to a comprehensive understanding and explanation of the structural interrelations of the culture spheres of science, morality, art, and religion.

Analytic philosophers tend largely, though not exclusively, to be English-speaking academics whose writings are directed, on the whole, to other English-speaking philosophers. They are the intellectual heirs of Russell, Moore, and Wittgenstein. Wittgenstein, in the Tractatus Logico-Philosophicus, contended, though obliquely, that the structure of language reveals the structure of the world; every meaningful sentence is analyzable into atomic constituents that designate the finegrained constituents of reality. Analytic philosophy is, perhaps, an implicit respect for argument and clarity, an evolving though informal agreement as to what problems are and are not tractable, and a conviction that philosophy is in some sense continuous with science.

Continental philosophy is the gradually changing spectrum of philosophical views that in the twentieth century developed in Continental Europe and that are notably different from the various forms of analytic philosophy that during the same period flourished in the Anglo-American world: phenomenology, existentialism, Heidegger’s philosophy, structuralism, deconstruction, Frankfurt School, Habermas’s theory of communicative action.

Cambridge Dictionary of Philosophy.

What are its pros and cons of speculative philosophy?

I think we can interpret speculative or continental philosophy in contrast to analytic philosophy and that the best approach is to consider some famous direct debates between famous analytic and continental philosophers:

Reception of Heidegger’s philosophy by Analytic and Anglo-American philosophy
http://en.wikipedia.org/wiki/Martin_Heidegger#Reception_by_Analytic_and_Anglo-American_philosophy

Gilbert Ryle on Heidegger’s Being and Time
http://www.waggish.org/2012/gilbert-ryle-on-heideggers-being-and-time/

“The Elimination of Metaphysics Through Logical Analysis of Language” Rudolf Carnap
http://www.calstatela.edu/dept/phil/pdf/res/Carnap-Elimination-of-Metaphysics.pdf

The first recorded announcement of the analytic-continental divide in the twentieth century.
http://jhaponline.org/journals/jhap/article/view/1449/1380

1958 Royaumont encounter between British and French philosophers
http://www.tandfonline.com/doi/pdf/10.1080/09608788.2010.524764

Sokal affair
http://en.wikipedia.org/wiki/Sokal_affair

Searle-Derrida debate
http://en.wikipedia.org/wiki/John_Searle#Searle-Derrida_debate http://en.wikipedia.org/wiki/Jacques_Derrida#Dispute_with_John_Searle

Protests against Derrida’s honorary degree in Cambridge
http://en.wikipedia.org/wiki/Jacques_Derrida#Cambridge_Honorary_Doctorate

How rational can a polemic across the analytic-continental ‘divide’ be?
http://www.tau.ac.il/humanities/philos/dascal/papers/divide.html

The genuinely continental and original voices in continental field, in so far as any remain, strike me to be as obscure as ever. The debate remains easier among analytic philosophers than between analytic and continental philosophers.
What is analytic philosophy? Hans-Johann Glock

If we have to choose only one word to represent the view of analytic philosophers to continental philosophy I think is “unintelligible” or “obscure”. One frequent criticism of analytic philosophy is that it is a highly technical mode of thought, restricting itself to criticism of minute details of language, and has abandoned the great narrative of a worldview. A perennial criticism of analytic philosophy is that it fails to engage with our deepest and most basic human concerns, and has thereby rendered itself irrelevant to the larger culture.

Answer 2 (score 0)

Continental philosophy is generally philosophy of the European continent (ie excluding les Anglais) - most famously German or French philosophy. Its style is generally informed by literature and religion. For example Kierkegaard, Derrida, Hegel or Kant.

Its in contra-distinction to the Anglo-American tradition which is generally informed by mathematics/logic and science (but there is a tacit understanding that all science boils down to physics, and all physics to mathematics and then to logic).

The Anglo-American Analytic tradition pride themselves on their pragmatism, their amour fou of mathematics and thus of rigor. Their excesses are arid and instill rigor-mortis in the obstinate reader.

The continental tradition prides itself on its flights of poetic fancies, its speculative daring. Their excesses tend towards wind-baggery which is what Schopenhauer accused Hegel of peddling.

Continental Philosophy is not quite the real thing when the one doing the talking or writing is an Anglo-American philosopher or aspires to be one either secretly and furtively, or openly and brazenly.

Of course there can be rigor in speculation, and speculation in rigor.

And of course, one should take these characterisations with a broad pinch of salt as they are broad generalisations.

I have no idea what Speculative Philosophy is. Kant does say:

Hume “interrupted my dogmatic slumber and gave my investigation in the field of speculative philosophy quite a new direction”

So perhaps it can be identified entirely with continental thought.

65: Does Pascal’s Wager contain any logical flaws or fallacies? (score 20728 in 2011)

Question

Blaise Pascal’s famous wager was that even if the existence of God cannot be determined through reason, a rational person should wager as though God exists, because living life accordingly has everything to gain, and nothing to lose.

What logical flaws and/or fallacies (if any) are committed in making this argument?

Answer accepted (score 46)

  1. He assumed that if God exists then

    1a. humans are immortal (as a necessary condition for 1b)

    1b. God decides whether they are tortured eternally or happy eternally (one of them is enough to profit infinitely, though)

    1c. God bases his decision on whether they believe in him (if this is not the case, the wager does not work, because believing in God or believing in God because of the wager might cause infinite torture)

    1d. God bases his decision on whether they follow religious law (again, the wager does not work if God does not like people following religious law)

    1e. God is the Christian God and you have to believe in exactly the right doctrinal points and those are the points that Pascal was raised with (again, the wager does not work if God does not like people to follow a particular religious law)
  2. It is not true that you have nothing to lose if there is no afterlife and you have spent all your life following the instruction of the religion even though you would have preferred to do something else.

  3. He also assumed that he can rationally decide to believe.

Summary: The wager assumes that there is a positive probability for God rewarding a particular behaviour with eternal bliss and a zero probability for God punishing this very behaviour with denial of eternal bliss. There is no reason for this assumption.

If the probability for a reward is in fact zero, it does matter if you sacrifice all the life you have for a non-existing reward.

If the probability for God disliking the behavious is not zero, then you have to weigh more than one infinite expectance values.

Even if everything works fine, you would have to be able to believe or disbelieve just because you want to. (Like, say, an atheist who reads a study that believers have a better healing chance for a particular cancer and decides to become a believer as a therapy.)

It’s logically fine if someone believes all the assumptions and then says that it is a good idea to believe. But this person believes in the first place, they do not believe because of the assumptions, they are just happy with their belief because of what they believe.

Answer 2 (score 20)

There are a great many logical fallacies in how the Wager is applied. Often the Wager is suggested as some sort of proof or last-ditch argument for God. But since it was embedded in the Pensées, which was Pascal’s life project to defend Christian thought, it seems unlikely he intended for the Wager to stand alone. Whether anyone would be convinced by such an argument seems not to be the main thrust of Pascal’s formulation of the Wager.

The very introductory statement to his argument shows that Pascal concedes that God cannot be proven in the Aquinian sense:

If there is a God, He is infinitely incomprehensible, since, having neither parts nor limits, He has no affinity to us. We are then incapable of knowing either what He is or if He is. This being so, who will dare to undertake the decision of the question? Not we, who have no affinity to Him.

Who then will blame Christians for not being able to give a reason for their belief, since they profess a religion for which they cannot give a reason? They declare, in expounding it to the world, that it is a foolishness, stultitiam; and then you complain that they do not prove it! If they proved it, they would not keep their word; it is in lacking proofs, that they are not lacking in sense.

It makes more sense to view Pascal’s Wager as a precursor to Alvin Plantinga’s Warranted Christian Belief. In contradiction to Cartesian skepticism, we are free (even compelled) to act on beliefs that we cannot prove from first principles. Any reasonable reading of Pensées would conclude that Pascal is not trying to take the risk out of choosing the Christian faith. Rather, he was defending the faith from the charge that it is irrational.


The Wager takes on added weight when you consider that Pascal himself made a bet and took up an ascetic lifestyle near the end of his life. It was this time that the Wager was formulated and it was not published in his lifetime. It’s difficult to ignore the possibility that Pascal took the Wager quite seriously. He summarized the argument by making this precise point:

The end of this discourse.—Now, what harm will befall you in taking this side? You will be faithful, honest, humble, grateful, generous, a sincere friend, truthful. Certainly you will not have those poisonous pleasures, glory and luxury; but will you not have others? I will tell you that you will thereby gain in this life, and that, at each step you take on this road, you will see so great certainty of gain, so much nothingness in what you risk, that you will at last recognise that you have wagered for something certain and infinite, for which you have given nothing.

“Ah! This discourse transports me, charms me,” etc.

If this discourse pleases you and seems impressive, know that it is made by a man who has knelt, both before and after it, in prayer to that Being, infinite and without parts, before whom he lays all he has, for you also to lay before Him all you have for your own good and for His glory, that so strength may be given to lowliness.—Pensée 233

And I suppose that is the primary fallacy of Pascal’s Wager: it can mean little to nothing to people who have not thrown themselves into the pattern of thought that Pascal himself followed. It is unpersuasive since it builds on uncommonly held premises.


On a personal note, I find the Wager touching, even though or perhaps because, it is flawed. I love T. S. Eliot description of Pascal: “a man of the world among ascetics, and an ascetic among men of the world.” A normal Christian considering the sacrifice he made would be content to contemplate the words of Jesus in Mark 8:34-38 (ESV):

And calling the crowd to him with his disciples, he said to them, “If anyone would come after me, let him deny himself and take up his cross and follow me. For whoever would save his life will lose it, but whoever loses his life for my sake and the gospel’s will save it. For what does it profit a man to gain the whole world and forfeit his soul? For what can a man give in return for his soul? For whoever is ashamed of me and of my words in this adulterous and sinful generation, of him will the Son of Man also be ashamed when he comes in the glory of his Father with the holy angels.”

But not Blaise Pascal, mathematician. Instead, he converted Jesus’ idea into an example of the relatively new field of probability. And while the problem was setup in a way that it is easily rejected as an apologetic of Christianity, it is still valid and useful as a mathematical puzzle.

Answer 3 (score 14)

Here’s my problem with it:

  • Choose a group of self-described Christians and call them Group A.
    Group A believes that Jesus was the son of God, he died on the cross for our sins, and only through him can we go to heaven.

  • Choose another group of self-described Christians and call them Group B.
    Group B believes that Jesus was the son of God, he died on the cross for our sins, and only through him can we go to heaven.

Given: for every Group A, there is a Group B that firmly believes that Group A is going to hell.

As a result, the statement “a rational person should wager as though God exists” is meaningless, unless you first define to which group you’re referring.

66: Logic Question: Can a logically valid argument have a logically false conclusion? (score 20537 in 2015)

Question

Are there any examples of such an argument? What would its premises be like? How could such an argument be possible?

Where an argument is logically valid “if and only if it is not possible for all the premises to e true and the conclusion false.”

And logically false is defined as “if and only if it is not possible for the sentence to be true.” (e.g. ‘June will pass Chemistry 101 and she will not pass Chemistry 101’).

Answer accepted (score 3)

  1. Premise: all horses are brown
  2. Premise: X is a horse
  3. Conclusion: from premise 1 and 2 follows that X is brown

However, suppose that X in fact isn’t brown, but white (meaning that either X isn’t a horse - premise 2 is false -, or not all horses are brown - premise 1 is false).

The reasoning leading to the conclusion is logically valid, it’s a valid reasoning, but the conclusion is not true, because we started with false premises.

Answer 2 (score 1)

Any argument with necessarily false premises is valid, regardless of the conclusion. Therefore as long at least one of your premises is always false, you can have a false conclusion and still have a valid argument.

This is a counterintuitive fact. However, we need to remember that the only thing logical validity guarantees us is that we will NEVER have a situation where the premises are true and the conclusion is false. Outside of that single guarantee, validity tells us nothing.

This may not seem useful, but in fact, it is very useful because it preserves the quality of truth (where truth is as defined within the system of logic).

Answer 3 (score 0)

In a formal system, if you can prove a statement is valid, it is valid for that formal system (yay for tautologies!)

However, part of the formal system is the axioms one is assuming. If you disagree with the axioms, you can disagree with the conclusion.

Consider:

  • Assume: All cats are human is a human.
  • Assume: Pixel is a cat
  • Conclusion: Pixel is a human.

The logic is sound, but the first assumption is suspect.

67: Why is faith seen as a sign of weakness, instead of an unexplored land/opportunity? (score 20303 in 2018)

Question

Hope this is the right place to put this question!

I am a person of faith (more specific, a Christian) and most of the time people consider me somehow inferior for my belief. I am not antisocial, not sick, nor crazy, but rather rational (even if apparently this is in contradiction with “faith”). I feel that many times when discussing with some people. After reaching religion in our discussion, when I tell them that “I am a faithful person” the answer I get is something like: “Oh, faithful… I see…” (from non-believing persons).

In another situation, I was having a conversation with a person who seemed impressed by my thinking and education, and we were going along this way for some time. However, after mentioning about my faith, that attitude disappeared and a despise-superiority one came out. Why is this happening in our time, since in the past there were many great faithful personalities (Abraham Lincoln, Martin Luther King, George Washington, or even Mother Teresa)?

Somehow, people see me as some inferior kind of human, maybe even crazy - no matter of my other capabilities or social position. For many, believing what Christians believe is a sign of weakness. Now, my question is:

Why is faith seen in such a way in our times: as a sign of weakness instead of an unexplored land (considering the fact that we DO NOT KNOW an enormous quantity of things, fields, etc.)?

Why most of the people choose to despise, instead of accepting the possibility that we know too little about faith and its experiences?

Basically, why do folks close the door without checking what’s inside?

EDIT: 1. I do not need a “faith” definition, that’s not the idea.

  1. Giving me example or quotes from dead people that did not live in our time (in order to understand it), doesn’t really help much.

  2. I know I am not inferior, I just feel people treat me like I would be, and what I need to understand is why.

  3. If you have any experience from your community, about believers (of any kind) who are seen as I am, then you could write an answer. Not opinions, because someone’s opinion could be that my question is wrong, and such thing doesn’t exist, but I know what I am getting through and what is the reality.

  4. I would need an answer, not a debate… Please!

Answer accepted (score 72)

I will be writing this answer from the frank, blunt perspective of someone who is inclusive and accepting of all peoples and belief systems, but who tries to analyze my own personal pattern-matching.

I would not say that faith is a sign of weakness, nor do I think it is seen as one. Rather, it implies the existence of a ‘hole’ in someones rationalism that is impossible to mend. By ‘hole’ I mean to say that many religions will provide certain information and claim it as being irrefutable without evidence. Accepting this means that there are certain cognitive areas that cannot be explored. This ‘irrefutable information’ tends to branch out significantly, covering areas that often seem unrelated to non-religious people.

You could think of it from a different perspective- if someone tells you that they are a new-age “Alternative Medicine” proponent, there are certain connotations that go along with that. You could quite easily make assumptions about that person, generated via your brains subconscious pattern-matching.

These assumptions re-arrange the context of everything that person says or does from then on. Perspective is everything, and every piece of information you know about a person changes your perspective of them.

In this way, when you tell someone that you are religious, this comes with the full connotations of such a statement, and their brain will mentally re-analyze everything you’ve said and done, or will say and do, to the best of its ability in order to better prepare them for your behavior.

A vocal, politically active subset of religious people are unwilling to bend when it comes to their belief, and it causes them to deny or willingly avoid evidence if it points them in an uncomfortable direction that questions their beliefs. (Evolution? No, Creationism. Climate Change? God would not allow such a thing, would he? Etc…)

The typical rationalist perspective, then, isn’t that faith provides “opportunities”. Rather, faith is a cage which limits the questions you can ask. Instead, it gives unverifiable answers. Such a thing is the enemy of rationalism. It is, as I said earlier, a “hole” in someones rational mind that the person will refuse to fill with facts because it is held open by faith instead.

I feel the need to reiterate that I say this only to provide a potentially useful analysis of the world view of a rationalist. I have nothing against people of faith, and I firmly believe that religion can be an incredibly powerful tool for bringing people peace and happiness, though I do not choose it for myself.

Answer 2 (score 72)

I will be writing this answer from the frank, blunt perspective of someone who is inclusive and accepting of all peoples and belief systems, but who tries to analyze my own personal pattern-matching.

I would not say that faith is a sign of weakness, nor do I think it is seen as one. Rather, it implies the existence of a ‘hole’ in someones rationalism that is impossible to mend. By ‘hole’ I mean to say that many religions will provide certain information and claim it as being irrefutable without evidence. Accepting this means that there are certain cognitive areas that cannot be explored. This ‘irrefutable information’ tends to branch out significantly, covering areas that often seem unrelated to non-religious people.

You could think of it from a different perspective- if someone tells you that they are a new-age “Alternative Medicine” proponent, there are certain connotations that go along with that. You could quite easily make assumptions about that person, generated via your brains subconscious pattern-matching.

These assumptions re-arrange the context of everything that person says or does from then on. Perspective is everything, and every piece of information you know about a person changes your perspective of them.

In this way, when you tell someone that you are religious, this comes with the full connotations of such a statement, and their brain will mentally re-analyze everything you’ve said and done, or will say and do, to the best of its ability in order to better prepare them for your behavior.

A vocal, politically active subset of religious people are unwilling to bend when it comes to their belief, and it causes them to deny or willingly avoid evidence if it points them in an uncomfortable direction that questions their beliefs. (Evolution? No, Creationism. Climate Change? God would not allow such a thing, would he? Etc…)

The typical rationalist perspective, then, isn’t that faith provides “opportunities”. Rather, faith is a cage which limits the questions you can ask. Instead, it gives unverifiable answers. Such a thing is the enemy of rationalism. It is, as I said earlier, a “hole” in someones rational mind that the person will refuse to fill with facts because it is held open by faith instead.

I feel the need to reiterate that I say this only to provide a potentially useful analysis of the world view of a rationalist. I have nothing against people of faith, and I firmly believe that religion can be an incredibly powerful tool for bringing people peace and happiness, though I do not choose it for myself.

Answer 3 (score 35)

I should know better than to jump into this fray. But I can’t help myself since this is something I’ve wondered about myself. Here are the reasons I’ve collected over the years. Often times I don’t remember where I first learned of these reasons, I’m going to leave this un-cited.

  1. How do you distinguish faith from gullibility? Perhaps faith is sometimes seen as a kind of gullibility. Few people would criticize you if you have faith “the sun will come up tomorrow” or “I have faith that I’m skillful and smart enough to support my family for the foreseeable future.” It’s the faith in uncommon things that raises eyebrows. To use the over-used Sagan quote: “Extraordinary claims require extraordinary evidence.” It’s the willingness to believe extraordinary claims on inconclusive evidence that might strike some as gullible.

  2. How do you know what to have faith in and what not to have faith in? Why do people have faith in one religion but not another? Often it’s based on what they want to believe, or what they think ought to be true. (See https://www.psychologytoday.com/us/blog/ulterior-motives/201107/you-end-believing-what-you-want-believe.) Or they will believe things based on the culture they were brought up in, while discounting equally plausible claims from cultures they weren’t brought up in. Some say you know what to have faith in based on feeling. But that ignores the many who have feelings that are just as strong, but towards a different belief. Or it ignores the selection biases, conflicts of interest, and other influences that can subconsciously shape our feelings. Or it ignores the numerous times that feeling alone has proven to be unreliable. So to some, it may seem like faith is weakness if it appears that someone will selectively turn off their critical thinking skills for beliefs they have a vested interest in maintaining.

  3. They are inflexible. There a numerous examples of this. Consider the bumper sticker “The Bible says it. I believe it. That settles it.” Or this quote from Kurt Wise:

“I am a young-age creationist because that is my understanding of the Scripture. As I shared with my professors years ago when I was in college, if all the evidence in the universe turns against creationism, I would be the first to admit it, but I would still be a creationist because that is what the Word of God seems to indicate. Here I must stand.”

The title of this post mentions “unexplored land/opportunity.” Are you receiving criticism for exploring? Or for coming to conclusions when there is, admittedly, so much unknown? Faith may sometimes cause a person to appear close-minded: unable or unwilling to change beliefs based on new evidence or experiences. That may be a 3rd reason why faith might be seen as a weakness.

  1. Stereotyping. I’m sure every person of faith doesn’t match all 3 of these reasons. They are more of a caricature, and that might be a fourth reason: Those people who consider you weak because of your faith may be stereotyping you.

68: Why is a lion not evil? (score 20217 in 2018)

Question

One often hears the claim that animals who kill and such, are not in fact evil. The typical example is that of a lion or a tiger that kills a prey. The argument is more or less that the animal acts instinctively in order to survive and thus we cannot fault it for killing a prey. It must eat meat to live, therefore it is forced to do so. End of story.

This argument has two parts. There’s the instinctive part, i.e. that the animal just acts, without thinking too deeply about it, hence it is not evil. Then there’s the necessity part, i.e. it has to eat meat to survive.

Both parts sound suspect to me. On the instinctive part, … well, many humans who commit evil acts often do so instinctively, because it is part of their innate evil nature to do so. Surely just because something is instinctive, doesn’t mean one is absolved from responsibility. In fact, to be evil means precisely that one has evil instincts. Nobody becomes evil by thinking philosophically a posteriori about ethical paradigms and theories and coming up with the most evil one. People are evil instinctively. So this part of the argument seems of no use.

The second part is equally confusing. So just because you “have” to do something in order to survive, then all bets are off? You can do whatever you want, just because you “have” to? So if I took a person, put a bullet to their head, and said I’d kill them unless they pressed a button which would destroy all other life in the Universe, then if this person pressed that button, they would not be evil? They would be exempt from responsibility, just because they “had” to do it to survive? I’m sure some Objectivists ascribe to such a philosophy, but surely most well-functioning humans find this approach to ethics completely absurd.

To conclude, my question is, what actually is a good argument for why animals are not evil (because I, as most others, also intuitively think that animals are not “really” evil, so a better argument than the above must exist).

Answer accepted (score 39)

Interesting and difficult question. I’m inclined to deny that a lion can act evilly (in killing) on the following grounds :

  1. An action is morally right (in the sense of deserving praise) only if the agent is capable of recognizing or judging that it is the morally right thing to do and of doing it because it is the moral right thing to do.

Conversely :

  1. An action is morally wrong (in the sense of deserving blame) only if the agent is capable of recognizing or judging that it is the morally wrong thing to do and - motivation - of refraining from doing it because it recognizes or judges that it is the moral wrong thing to do.

  2. To do evil is to do the morally wrong thing and is deserving of blame.

  3. A lion cannot recognise or judge that anything is the morally wrong thing to do and refrain from doing it because it recognizes or judges that it is the morally wrong thing to do.

  4. Therefore a lion cannot recognize or judge that to do evil is the morally wrong thing to do and refrain from doing it because it recognizes or judges that it is the morally wrong thing to do.

Therefore:

  1. A lion is incapable of the recognition, judgement and motivation to do evil and is not deserving of blame.

[Note - this answer does not deny that non-human animals are morally significant; it does not deny that they have rights; it does not deny that at least some of them are capable of moral emotions (sympathy, attachment, for example). It just answers the question about the capacity of lions for evil.]


REFERENCES

S. F. Sapontzis, ‘Are Animals Moral Beings?’, American Philosophical Quarterly, Vol. 17, No. 1 (Jan., 1980), pp. 45-52.

Mark Rowlands, Can Animals Be Moral?, New York, NY: Oxford University Press, 2012.

Dan Hooley, ‘Can Animals Be Moral? by Mark Rowlands’, Journal of Animal Ethics, Vol. 4, No. 2 (Fall 2014), pp. 86-92.

Answer 2 (score 23)

The problem with questions like this is that there is no universally agreed upon definition of ‘evil’. You could try to reduce it to ‘harm’, but then you again run into troubles defining that, but at least it’s easier to get people to agree on a working definition.

The solution, when this actually matters, is not to get hung up on words like good and evil but to be explicit about what you actually mean:

Do you mean ‘evil’ as in ‘something that is harmful to humans’? In that case, wolves and lions are certainly ‘evil’ as hanging around them is likely to get you severely injured or even killed.

Do you mean ‘evil’ as in ‘something that is intentionally causing harm to humans’? Or ‘something that is capable of not causing harm to humans, but chooses to do so’? In that case you run into a bit of a pickle because then the discussion turns to the cognitive capabilities of lions compared to humans. That being said, lions possess long term memory, have social interactions and are capable of planning actions and then executing those plans. In addition, some lions are more likely to injure humans than others. So I think we can grant that they have both intent and the capability to choose for the purpose of this discussion; Which would make at least some lions evil. Of course you could equally deny this (it’s not uncommon for animals to be thought of as little more than machines that can’t act against their programming, a state which humans are usually thought to be exempt of).

Note that I have always specified ‘humans’ above - without qualification, the harm model kind of falls apart (at least when applied to individuals) because everything causes harm to something. For example, the example of lions implies that the OP thinks that at least killing humans or other animals could potentially be classified as evil; But plants are life forms as well, despite that killing them for food is rarely regarded as evil. This is because to classify as an evil act in the harm model, the subject of the harm generally has to have certain qualities (such as some level of cognition, being biologically human or possessing a ‘soul’ of some description).

This is all just on an individual level and with just a very direct idea of harm too, the situation gets even worse if you add things like society and concepts like responsibilities.

TL;DR the question is essentially unanswerable because evil by itself isn’t a definitive (but nevertheless emotionally charged) term, hence the confusion. In general language ‘evil’ just means ‘something I strongly disapprove of’ and moral theories are more or less just attempts to formalise the class of things to which this applies and describe it with a minimal set of rules. Because people start with different sets of things they disapprove of and the rules are generally just approximations of these sets rigorous applications of them frequently lead to seemingly absurd results.

Answer 3 (score 20)

Good and evil refer ultimately to a moral judgment: that you have an intent to do good or evil. It’s more difficult to ascribe intentionality to lower “orders” of creatures, and especially the cultural and religious sorts of intentionality associated with the ethical act.

Could a paramecium have a “consciousness of guilt”? What about a fly? Dogs certainly seem to have an internal life (at least appear to emote shame, guilt, etc.) Monkeys engage in what some characterize as war, suggesting something like the “sociocultural intentionality” (which I’m positing here as a kind of “floor” for the intelligibility of the good-evil distinction) might be present.


Maybe a bit more speculatively: consider Nietzsche’s role in the problem of moral relativism. For lions, the ‘law-tables’ of good and evil have to be reckoned differently (than for those of the goats and the apes.) The values of good and evil aren’t ultimate, but rather transform with the modes and means of existence.

69: Philosophical Similarities For Chinese Confucianism and Daoism (score 19947 in )

Question

I know there a lot more differences then similarities between Daoism and Confucianism. All I can find is that both have one goal and focus of self-improvement from being “individuals” by becoming a greater whole to contribute to society. Are there any more similarities?

Answer accepted (score 2)

Confucianism and Daoism are a classic pair of opposites in Chinese philosophy. They are also both terms that are notoriously different to pin down.

The Problem of Definitions

I know of at least three definitions of Confucianism: (a) the works attributed to Confucius and Mencius, (b) the works in (a) plus 24 centuries or so of commentaries on them, and (c) the political implementation of something vaguely related to (a) or (b) often in China but also in Joseon Korea and Tokugawa Japan. Also, there are different streams of interpretation in (b) with the most prominent being the neo-Confucians, such as Zhu Xi and the Chang brothers, and the New Confucians in the 20th Century including such thinkers as Kwongloi Shun, Chengyang Li, and Tu Weiming. (For a treatment of just the different forms this takes in China, see Xinzhong YAO, An introduction to Confucianism “Confucianism, Confucius, and Confucian Classics” Cambridge University Press, 2000).

For Daoism, there are also multiple definitions which I’m less competent to comment on, but we can see Zhuangzi and Laozi as two separate traditions and then we can also look at Sun Tzu’s Art of War as a Taoist text (at least Roger Ames does). We can ask what its relationship is to itself and what its relationship is to Buddhism (some forms of Taoism copied Buddhist practices). We can also see it as either the philosophy of protest against the “Confucian” state or an esoteric religion about finding immortality potions. Philosophical Taoism has been having a recent resurgence with prominent defenders.

The History of Dissimilarity

As rival views, the two are often presented as opposites when teaching Chinese philosophy. But this is a useful teaching device rather than proof that we should see the two as truly opposite. There are surely differences.

The Claim of Similarity

Many of the prominent defenders of Taoism in contemporary philosophy see Confucianism as having much in common with Taoism. I have heard Karyn Lai say things to that effect as well as Roger Ames.

Why do they make this claim?

First, Confucianism and Taoism as philosophical positions both are about Dao (道). Second, even though they are rival views about what Dao is, they share some of the same ideas as they are rivals in context. Comparing Descartes and Confucius is harder than comparing Confucius and Lao Tzu. Most of the other similarities build on the second one. But a third similarity is that as the Confucian commentaries advanced, they had to adapt to beliefs that their target audience found plausible. Thus, Zhu Xi spends a lot of time talking about li (not 禮 but 理) which means order because they had to develop a cosmology to respond to the Buddhist missionaries.

Fourth, a key reason why the West likes Confucianism but not Taoism has to do with a controversial set of translations. The Jesuit translations rendered many of the ideas of Confucianism into familiar Western terms:

  1. “Rightenousness” for 義 yi / now sometimes rendered “appropriateness”
  2. “Heaven” for 天 tian / now sometimes left untranslated
  3. “Virtue” for 徳 de / now sometimes rendered “power”
  4. “Benevolence” for ren 仁 / now sometimes rendered “humanity”

(I could expand the list if necessary). But a recent challenge is how accurate these renderings really are to the Chinese context. This is an open area of debate, but part of why it matters is that the Jesuits saw the potential for a synthesis between Christianity and Confucianism but not Christianity and Taoism.

Moreover, the definition of all of these terms is something where Taoists make interesting claims. For instance, they deny that ren should be understood in pedestrian terms. Similarly, they see de as referring to the power to influence. Interestingly, you can reread Confucian texts with these definitions and they still make sense.


To give a parallel, Descartes and Locke take opposite views in epistemology, but it does not mean they have nothing in common. Both write as Christians. Both try to solve problems of perception. Both reject certain features of the classic medieval philosophies while implicitly accepting others.

Answer 2 (score 1)

In my opinion

  • Confucianism, a philosophical treatment, focus on ethics, notably the relation between the government, the ministers and the people, while

  • Daoism, an esoteric speculation, focus on the relation between the individual and his natural environment.

Hence I do not see many similarities.

Added 10.7.2015: According to the definitions stated in the answer of virmaior my answer refers to the works attributed to Confucius and to an English translation of the Daodejing.

Answer 3 (score 1)

In my opinion

  • Confucianism, a philosophical treatment, focus on ethics, notably the relation between the government, the ministers and the people, while

  • Daoism, an esoteric speculation, focus on the relation between the individual and his natural environment.

Hence I do not see many similarities.

Added 10.7.2015: According to the definitions stated in the answer of virmaior my answer refers to the works attributed to Confucius and to an English translation of the Daodejing.

70: What were Plato’s view on slavery, in particular with respect to his proposed Utopia? (score 19714 in 2011)

Question

In the Dialogues(specifically The Republic), when he is discussing his Utopia, At one point, he does say that children should not be treated differently based on their parent’s status in life. However, I did not see any mention of slavery in this discussion, so I want to know what did he propose about the practice of slavery in his Utopia? Also, what were his views of Slavery in general?

Answer accepted (score 10)

There is no special cast called “slaves” in Platos Utopia. Instead all children are given to the state to be brought up and given work as the state sees fit once adult. Hence, almost everybody are slaves. ref Although all people are “citizens” these “citizens” have no rights, only obligations, and no freedom at all except for the absolute upper part that rules the city.

Plato wishes to abolish freedom, and hence in our terminology make everybody slaves. He tries to cover this by claiming that rulers are enslaved (by responsibility and fear of those who rules over) and in pure NewSpeak claim that freedom leads to slavery. (ref: The Republic)

In Platos later ‘Laws’, he describes another city which he thinks is more realistic, and it does have a separate slave cast. ref

Platos view of slavery was therefore quite clearly very positive. But he knew that the word was negative, and therefore tried both to associate things he did not like, primarily freedom, with the word slavery, and also tried to excuse tyrannic rulers by claiming they actually were slaves to the people. But people that we today would call slaves, are people he wanted many of in his ideal states.

Answer 2 (score 5)

A knife is good, he said, when it cuts efficiently, that is, when it fulfills its function.

The assignment of all people to their respective classes would come only after extensive training and only those capable of doing so would progress to the higher levels. Although theoretically all people would have the opportunity to reach the highest level, they would stop in fact the level of their natural aptitudes.

This implied that by nature some would be rulers and others craftspeople, and would provide the basis for a perfectly stratified society. Whereas later societies in Europe assumed that children born into such a stratified society would stay at the level at which they were born, Plato recognized that children would not always have the same quality as their parents. He said, therefore, that among the injunctions laid by heaven upon the rulers there is none that needs to be so carefully watched as the mixture of metals in the souls of children.

If a child of their own is born with an alloy of iron or brass, they must, without the smallest pity, assign him the station proper to his nature and thrust him out among the farmers and craftspeople. Similarly, if a child with gold or silver is born to craftspeople “they will promote him according to his value.”

Most importantly, Plato thought that everyone should agree on who is to be the ruler and agree also on why the ruler should be obeyed.

Answer 3 (score 1)

First of all Plato’s Republic which you mention as Utopia has many parts taken from existing cities and legislations specifically Sparta and Crete.

Concerning your comment on slavery. Broadly speaking Plato belonged to the Athenian Aristocracy. He believed, and is evident in the Republic that not all men are created equal. In the Republic, there is no explicit mention of slaves per se but, men are separated in 3 distinct divisions depending on their personal merits.

One thing to note here, since the concept of slavery in Greece has been misinterpreted by a lot, is this:

  1. The king in Plato’s Republic is the absolute best. The best in all aspects. The best warrior, the best philosopher the most capable person by any standard.
  2. There are no hereditary rights to powers. I.e. the children of the division of the warriors do not necessarily stay in that division. If they are not fit to be warriors, they might fall into the division of farmers.

So to answer your question, division 3 is the division that is somehow the slaves of Republic since all they do in life is work to feed themselves (and families) of course but to feed the other 2 divisions (the republic’s elite) as well.

This is not far from what slaves did in Ancient Greece since their treatment was not as you imagine or happened in Egypt or elsewhere.

Also note on this that slave in Greece is δοῦλος which is the root of today’s word for work in Greek.

71: What is the difference (if any) between “not true” and “false”? (score 19553 in )

Question

A fairly simple question I hope someone can help me with.

Answer accepted (score 55)

“p is false” implies “p is not true”, but not vice verse because p can also be nonsense.

“2 + 2 = 5” is both false and not true.

“2 + 2 > red” is neither true nor false because it is nonsense. If it were false, its negation “2 + 2 ≤ red” would be true, which is not the case.

Source An Inquiry Into Meaning and Truth

Answer 2 (score 14)

In the classical logic something is neither true nor false if it is grammatically malformed to have a truth value, so 2+5 or “x is blue” are not “true”, but not “false” either, they are not truth-apt. The classical assumption was that all truth-apt expressions can be distinguished by syntax alone, i.e. there is a clear way to tell from how they are formed whether it is truth-apt or not, without inquiring into what they mean. However, it is easy to come up with expressions that are grammatically well-formed but problematic semantically, sometimes crudely called gibberish, e.g. category errors like “electrons are blue”. Those are also neither true nor false, at least intuitively. Wittgenstein even suggested that in natural languages there is no clear distinction between syntax and semantics, and there is no way to clearly prescribe what is well-formed, all rules are “grammar”.

There are non-gibberish expressions that have problematic relation to the truth for other reasons, e.g. “such and such will win the election tomorrow”. Is it already true (or false) today? Aristotle and modern intuitionists say “no”. What about undecidable mathematical statements, like the continuum hypothesis? Same idea. There is also another dimension to the difference between true and false. The classical logic assumes for simplicity that that those are the only truth values that truth-apt expressions might take, this is called bivalence, often confused with the law of excluded middle. Multivalued logics remove this assumption. In particular, popular in applications fuzzy logic allows certain claims (usually “vague” ones) take any truth value between 0 and 1, with 0 being false and 1 true. So something like “15 degrees centigrade is cold” will be neither true nor false but have the truth value of say 0.6.

All of these phenomena led to the idea of logics with “truth value gaps”, where we either interpret some expressions as having no truth value at all, or one different from “true” and “false”. Sometimes we are forced to do this by the classical logic itself, e.g. the Liar sentence “I am false” leads to a contradiction if we assume that it has one of the two classical truth values. There is a whole field of semantic paradoxes like tha Liar, to resolve which Kripke specifically developed a whole semantic theory with truth value gaps. Paradoxes of vagueness, like the paradox of the heap (one grain is not a heap, adding a single grain won’t make not a heap a heap, therefore no amount of grains makes a heap) can also be resolved using truth value gaps.

Answer 3 (score 10)

In classical logic these are the same by definition.

But in very tentative logics like Constructivism or Intuitionism, things are only said to be true or false if they meet quite stringent conditions. People using criteria like this require a truth to be proved in a given way, or captured by a certain kind of generalization, and a falsehood to proceed from a clear counterexample that meets the standard for truth. (The idea is that truth is ultimately negotiable, as our intuition improves, or that we should avoid claiming truths we cannot back up with computations.) That means that just not being false is not enough to make them true. There is a vast middle ground of things that remain inaccessible to truth or falsehood.

72: What does “moral agent” and “moral agency” mean respectively? (score 19448 in )

Question

“Agent”, OED says, could mean “an active and efficient cause, capable of producing a certain effect”; “agency” could mean “a thing or person that acts to produce a particular result”. It seems they are synonyms.

“Agent” in “moral agent”, though means simply a subject, an “actor”, can be understood in a sense as a cause of an act. Since “moral agency” is used to mean the inner dynamics of the moral agent, “agency” is to be understood as “cause” too: what makes the agent act morally?

To treat them both as meaning “cause” cannot justify them representing two things different in order. Therefore I doubt that maybe “agent” refers to the causes which are of natural kind, such as human or corporation, while “agency” can mean the inner cause that explains how the causes work as a cause.

Sorry, this may be a very immature thought, please feel free to correct me!

Answer accepted (score 6)

There is a straightforward terminological clarification here, and then a much richer philosophical problem beneath it.

An “agent” in this sense is someone who does things. It derives from the Latin word “ago” which means “(I) do.” Romans used it as their general “doing” verb for a wide range of activities. Our word “agenda” comes from the same word, and literally means “things that ought to be done.” A secret agent is just someone who does things secretly.

A “moral agent” is therefore someone (or something) capable of doing things rightly or wrongly. Typically, this is understood to mean acting with the ability to freely choose (within parameters) what to do. It sometimes also includes the idea of being aware of the concepts of rightness and wrongness, or of what actions are considered right and wrong.

“Agency” is simply the capacity to be an agent, which means the capacity to do things. We see the same relationship between the words “regent” (a monarch) and “regency” (the commission to serve as a monarch or status of being a regent). Moral agency is the capacity to act as an agent. It’s something that every moral agent has by virtue of being one.

Once you get the definitions sorted out, I don’t think there’s anything puzzling there. However, embedded in your question is a challenging question about who or what does the causing when a person acts. Does a person cause his/her actions? Is there some sense in which his/her agency could itself be the cause? This slides us into the longstanding problem of mental causation. It’s the problem of how a mind (which seems at first glance to be a non-physical thing known to us through our conscious experiences, thoughts, and feelings) could ever serve as a cause of physical things like our picking up a rock and throwing it. It’s very difficult to figure out what could be doing the causing when an agent acts. Given your interest, you may want to read more about mental causation.

73: If I said I had $100 when asked, but I actually had $200, would I be lying by omission? (score 19358 in 2019)

Question

If you had $200 cash on you right now, and I asked you if you had $100 on you, would the correct answer be yes (always/no matter what other conditions there are), no (always/no matter what other conditions there are), or it depends on the situation?

My answer would be “yes” (always), because if someone asked me if I had $100 because he/she wanted to borrow it (and I had more than $100), then my reply would be “yes.” I asked this question before (before deleting it because it was off-topic) on another stack exchange (I would think this topic belongs to philosophy SE at least on interpretation of context alone), and most of the answers were “it depends on the situation.” But, I cannot imagine how it could depend on the situation.

If “it depends” if I have $100, that doesn’t even make sense to me? Either I have $100 or I don’t. Is the response “it depends” wrong or is it a situation of semantics?

If someone wanted to know if I had exactly $100, then he/she could ask me if I had exactly $100.

Edit: The responses seem to be a bit divided with most people answering “it depends.” I just thought that “have” can be construed as “in possession of” so usually (depending on context, but in this case it’s a yes) it would mean “at least” while other words like “brought” and “give” don’t have the same implications (and mean “exactly”).

Another example: Do you have a pet dog? (Has a pet dog at home but is not with him/her) I would say the honest response is yes here because of the context and “have” can’t be construed as “in possession of” here, but in the above example with money, I would say it can (no matter what other conditions there are).

Do you have a pet dog? (Has 2 pet dogs at home but they are not with him/her) I would say the honest response is yes.

Do you have one pet dog? (Has 2 pet dogs at home but they are not with him/her) I would say the honest response is no (because “have” doesn’t mean in possession of here).

Answer accepted (score 75)

I would say it depends on the situation. Specifically, it depends on whether the person asking you the question wants to know whether you have at least $100, or exactly $100. The question could literally mean either, and only the context can decide. The former situation is likely much more common, and includes the example you mention, in which the person wishes to borrow $100.

But consider a similar example. Suppose I tell you I have five coins in my pocket, and then I take one out and throw it away. How many coins do I have left in my pocket? Wouldn’t you consider it odd if in fact I have nine, because I had ten to start with?

This is one of those cases where in order to understand the meaning of an utterance you need to judge the speaker’s intention in making it. If I volunteer the information that I have five coins in my pocket, it is a reasonable presumption that I am intending to tell you exactly how many I have. But depending on the context, some other intention may be obvious. If we are standing in front of the entrance to some building that charges five coins for admission, then my saying “I have five coins in my pocket” would more likely express the intended meaning that I have enough money to afford the entrance fee, and hence that I have at least five.

This can be understood as an example of Grice’s theory of conversational implicature. The speaker’s intention may differ from the literal semantic meaning of a sentence, because the utterance needs to be interpreted by its audience in the light of the co-operative principle. In this case, the ambiguity in the question is resolved by what is relevant to the conversation. (“Be relevant” is one of the maxims of the co-operative principle.)

Answer 2 (score 44)

tl;dr- It’s a lie if the speaker intends to deceive the listener(s).

More specifically, it’s a lie-by-omission if the speaker intends to deceive the listener(s) by neglecting to mention something that, absent their intent to deceive, they’d have otherwise said.


Lies are communications intended to deceive recipients.

It depends on if there’s intent to deceive.

Examples:

  1. Not lying:
    Despite having more than $100, Person B can truthfully answer that they have $100.

    Person A: I need to borrow some money! Do you have $100?
    Person B: Yes.
  2. Lying:
    Despite having $100, Person F can deceptively claim to have $100.

    Person A: I got you all presents! Open them up!
    Person B: [Opens present to find $100.] I got $100!
    Person C: [Opens present to find $100.] I got $100!
    Person D:
    [Opens present to find $100.] I got $100!
    Person E: [Opens present to find $100.] I got $100!
    Person F: [Opens present to find $200, then fearing that this was an error that might be corrected, hesitates to say anything.]
    Person E: Hey, Person F, do you have $100?
    Person F: Yes.

Since lying-by-omission means deceiving someone by not saying something, it typically comes up in a context in which a reasonable person wouldn’t expect omission from an honest person.

In the above example, an honest Person F could’ve called attention to the fact that they received $200 while everyone else only got $100, perhaps asking Person A if this was a mistake and offering to return the money. A reasonable person could’ve expected Person F to at least point out that they got $200 when prompted by Person E.

Because Person F crafted their communication in a manner intended to deceive, they lied.

Because Person F’s mode of lying was not providing information that a reasonable listener would’ve expected an honest speaker to provide, it was a lie-by-omission.

Note: Here I’m suggesting that this can be viewed as a lie-by-omission, because it’s certainly a lie, and its deception is describable as being effected by omission. However, the by-omission descriptor is more perspective-subjective. @MichaelS’s answer details another perspective in which the lie wouldn’t be described as being by-omission.


Reference: Related concepts.

There’re a few things that might be confused with lies. Here I’ll mention a few to help draw a line between them.

Not lies:

  1. Honest mistakes.
    A person can say something that creates a false impression in listener(s) without lying if they did so based on their own misunderstanding of what’s true.

    Person A: I just got $100 in my gift envelope! Do you have $100?
    Person B: [Actually received two $100-bills, but thought it was only one.] Yes.
  2. Misunderstandings.
    A person can say something that creates a false impression in listener(s) without lying if they did so without intent to cause that false impression.

    Person A: [Asking in a context in which they mean exactly $100, assuming that Person B understands this.] Do you have $100?
    Person B: [Has $200.] Yes.
  3. Miscommunications.
    A person can say something that creates a false impression in listener(s) without lying if they did so without intent to cause that false impression.

    Person A: [Sends a text-message to Person B.] Do you have $100?
    Person B: [Tries to hit the auto-suggested response for “no”, but accidentally hits the auto-suggested response for “yes”.] Yes.
  4. Non-true statements not intended to deceive (e.g., hyperbole or sarcasm).
    A person can intentionally say something untrue without lying if the untrue statement isn’t intended to deceive.

    Person A: Do either of you have $100?
    Person B: I’ve got a trillion dollars!
    Person C: I’ve got a trillion-zillion-infinity dollars!

    Here, Person B said something that was untrue (and false), but it wasn’t a lie because there was no deceptive intent. Likewise, Person C said something that was untrue (and gibberish), but it also wasn’t a lie because it also lacked deceptive intent.

    Note: Above, I’m distinguishing between two types of non-true statements: falsehoods and gibberish.

    • A false statement asserts something that isn’t true. For example:

      Person B: I’ve got a trillion dollars!
    • A gibberish statement (“not even false”) doesn’t assert anything because it doesn’t mean anything. For example:

      Person C: I’ve got a trillion-zillion-infinity dollars!
  5. Secrets.
    A person can keep a secret without lying by not intentionally causing a false impression in listener(s).

    Person A: Do you have $100?
    Person B: I’m not telling you.

To note it, it’s not generally possible to be honest while keeping secrets. The specific problem is that an asker can craft questions which are just about impossible to respond to without either revealing the secret or responding deceptively.

For example:

  1. Lying to keep a secret:
    Sometimes a secret-keeper doesn’t have the option of honestly keeping a secret since a non-response or refusal to respond would be interpreted in a way that compromises the secret.

    Person A: Did you steal the cookies from the cookie jar?
    Person B: [Did steal the cookies from the cookie jar.] No, not me!
  2. Failing to keep a secret through honesty:
    Sometimes a secret-keeper can fail to keep a secret through honestly not revealing it.

    Person A: Did you steal the cookies from the cookie jar?
    Person B: [Did steal the cookies from the cookie jar.] I’m not telling you.
    This is like a partial break in cryptography: since Person B didn’t confess, Person A may not be certain that Person B is guilty, but they still have more information about the secretive matter than before they asked.
  3. Failing to keep a secret despite lying:
    Sometimes a secret-keeper can fail to keep a secret despite lying in an attempt to maintain the secret.

    Person A: Did you steal the cookies from the cookie jar?
    Person B: [Did steal the cookies from the cookie jar.] No, I was out of town that night!
    Person A: No, you weren’t. We have you on camera near the kitchen that night.
    Here, Person B did lie in an attempt to maintain the secret, but ended up leaking the secret despite their lie.

It’s also possible to lie without deceiving listener(s) if the speaker intended to deceive listener(s). For example, in the above example,

  • Person A: Did you steal the cookies from the cookie jar?
    Person B: [Did steal the cookies from the cookie jar.] No, I was out of town that night!
    Person A: No, you weren’t. We have you on camera near the kitchen that night.

, Person B lied despite not deceiving Person A because they intended to deceive Person A. This is, Person B’s lie was still a lie despite Person A seeing through it.


Conclusion: It’s a lie if it’s said with intent to deceive.

In short, a communication is a lie if-and-only-if there’s intent-to-deceive.

Answer 3 (score 10)

Your question is about lying by omission, and this requires that you define lying. The definition I use is a communication with the intent to deceive. Thus, whether or not you are lying is a function of your intent, more than the actual quantity of cash you have on your person.

Let’s examine two cases. In the first, you have $200, and when asked you state no knowing full well that there is $200 in your wallet. In this case, yes, it is a lie.

EDIT 2019-08-28: But here’s another circumstance. Let’s say while transferring $100 bills from your safe to your wallet, you accidentally grabbed a three instead of two (they were brand new and stuck together). Now, when asked if you have $200, you reply yes, despite the fact that you have $300, you have not lied, omission or otherwise. This is because you committed a mistake and were not aware of this mistake. As your intention is to communicate your sincere belief, then this is simply not a lie.

74: Is it possible to be truly selfless or altruistic? (score 18930 in 2012)

Question

Is it possible for people to actually be selfless? It seems that in many cases where someone is being kind, they are actually performing in a manner that will benefit them. Either the recipient of the kindness will reciprocate, or the act of kindness itself is deemed commendable. Therefore, the one who offered the kindness still benefits personally from their action.

A child shares a toy in the same way. Even with the heroic act of martyrdom, the possibility of knowing one could become a martyr may be uplifting. It seems that every “selfless” act, in some way, may benefit the one being “selfless”.

In light of those observations, is selflessness truly possible? What does philosophy have to say about this question?

Answer accepted (score 13)

First, let’s get the terminology straight. What you are talking about does not appear to be “selflessness” at all, but “self-sacrifice”, or “altruism.”

Now, with that in mind, let’s refine the question. You appear to be asking “Is it possible for someone to act in a manner that is not motivated, directly or indirectly, by self-interest?”

If this is the question, we immediately run into two difficulties:

  1. We need to have a clear idea of what we mean by “motivation” in this sense; unfortunately, this is an extremely difficult problem, as most people recognize the possibility of unconscious motivations– this means that we have no reliable manner of ascertaining precisely what one’s motivations were for any particular act.

  2. We need a good definition of “self-interest.” This is a much more difficult problem than it appears, and a critical analysis of forms the first part of Derek Parfit’s classic Reasons and Persons. I’d recommend this book as a good starting point, if questions like the one you posed interest you.

Finally: if we set aside all of the above, and still try to plow through to an answer, I suppose the answer would have to be “Why not?” Is it possible? I don’t see any reason it should be impossible to believe that at least once in the history of humanity, a single human has taken a single action which offered no foreseeable benefit to the actor. But what does that really tell us?

Answer 2 (score 1)

On a temporary basis and as an ongoing personal philosophy selflessness is possible but a consistent selfless life is unsustainable.

Selflessness perhaps seen “Universally” or collectively could be seen as good because collectivism inherently blanks-out the individual, but in an individual sense selflessness is bad. All values have to be produced by individuals, so altruism requires a sacrifice of time and effort. To see morality in regards to the daily and your long term goals, as individuals, sacrifice is bad. I do consider benevolence and altruism to be separate concepts. Philosophy qua individualism selflessness is bad. It can mean a few things: material selflessness (the giving of possessions), spiritual selflessness (giving up logic and thought; ie religion), and collectivist ethical doctrine towards the state.

Answer 3 (score 0)

1. As far the science explains life is “just” algorithms assembled in a way that they are able to exist at least up to the moment of replication.

2. Such replication mechanism guides life (?)towards(?) homeostasis.

3. To have homeostasis without having “mind as we - sapiens - have” is a kind of “easy”, however, when we - sapiens - become able to build structures we start to be able to leverage.

4. On the process of leveraging we are still looking for homeostasis however we have not capabilities for this (maybe yet) so we use heuristics because at the conscious level at any given time we might be able to play with maximum of (?)7, 8 concepts in our mind.

5. When we are effective and/or efficient we might use concepts such as “selflessness” (or the opposite ☺).

6. Concluding [Is it possible to be truly selfless or altruistic?] if we are a “standard” sapiens we are able and capable to perform much more times as a selfless being than the opposite.

Among references one can start with for example: Why Does the World Exist?: An Existential Detective Story, Jim Holt. And if one likes can go deeper by reading this Quantum theory, the Church–Turing principle and the universal quantum computer by David Deutsch.

After all the purpose of life it seems to be so simple that our "mind" has difficulties to "digest it" ☺

75: Would Kant choose to sacrifice one life to save another? (score 18875 in 2012)

Question

If what I know about Kant is correct- watching the Harvard Justice series on Kant and some summaries of his work- then Kant believes in an absolute morality where everything is either right or wrong in every situation. The act of morality is not dependent on the consequence of the action, but rather the action itself. His mantra, I believe, is, “Act so that your action might be applied as a law of the world.”

We have heard the question of, “What would Kant say to a murderer asking him about the whereabouts of the potential victims.” to which the answer is usually, “Tell him.”, “Close the door.”, or “Twist the truth so that what you are saying is not a lie in itself and therefore respects the categorical imperative.”

I stumbled upon a similar question for which I cannot find an answer that would lie within Kant’s philosophy. I chose this question because the usual one with the murder is classified as exceptional. WW2 was a horrible period in our history and is, hopefully, never to be repeated. However, complications during childbirth are a lot more common and I can see how some people must face this situation. I hope that most of you will agree that this question is more properly grounded in realism than other examples.

Imagine being married and your wife is currently giving childbirth. 
Because of complications both the child and wife are in critical danger, 
and you must choose which individual to save.

My first instinct is that Kant would say, “No, I cannot make this decision for every person has inherent value and to choose one over the other would violate their right to life.” Therefore, in the extreme, Kant would let both people die.

However, I question this, not only for my ignorance of Kant’s true philosophy. The difference between these two questions is that you are the person that must either act or not act. A killer is the agent acting out the evil option. In this scenario, there is no evil agent, only unfortunate circumstances. I cannot help but feel that inaction is an action at the same time, therefore I come to the conclusion:

  1. Sacrificing one individual for the other is not permitted.

  2. Inaction is an action. Therefore, the action itself, not choosing an individual, is morally evil and contradicts the categorical imperative.

The two contradictions are horrible. I believe to have read on Wikipedia that Ayn Rand called Kant a “monster” because of conundrums like these. This makes me understand her objections more clearly, while at the same time believing that Kant’s imperative is still the best possible action to be performed. However, we, as irrational human beings, would choose not to uphold Kant’s philosophy.

Am I correct in assuming that Kant would refuse to act in this situation? More specifically, Kant would let the circumstances unfold and risk losing both his wife and child? I’m also very interested in the reasoning behind the decisions as I am quite possibly misinterpreting or misunderstanding the philosophy of the categorical imperative.

Answer accepted (score 4)

Your specific question (about abortion) has been discussed a fair bit. In general, people seem to agree that when the mother’s life is threatened it’s OK:

This presumption [that abortion is immoral] may be rebutted when the agent’s reasons for abortion have to do with such things as physical risks of pregnancy… abortion is morally problematic, but often permissible - Animality and Agency: A Kantian Approach to Abortion

You can read the full paper for her exact reasoning, but it roughly has to do with considering all the various duties one has and weighing them against each other. (i.e. certain duties are more “important” than others.) This is a very common way to approach it.

Another way, frequently discussed with regards to euthanasia, is the doctrine of double effect:

It is claimed that sometimes it is permissible to cause such a harm as a side effect (or “double effect”) of bringing about a good result even though it would not be permissible to cause such a harm as a means to bringing about the same good end.

So, roughly, killing the fetus is “side effect” of saving your wife and therefore you aren’t using it as a “means to an end.” How exactly one defines a “side effect” versus a “means” is difficult, of course.

As to what Kant would say: who knows? You’re right that in the inquiring murderer case he held fast to his guns and claimed that you shouldn’t lie, but if more extreme examples were brought to his attention would he have changed his mind? I guess we’ll never know.

Answer 2 (score 1)

It is clear that any intelligent human being will proceed under assumption to do whatever is possible to save both. This assumption will be proven wrong only after the fact, when one of them dies, or both die or both live. Since you do not know in advance of the fact, you cannot be morally at fault. This dilemma is just another variant of the “throw grandma under the tram” dilemma, and it is total nonsense to say that the outcome is certain death for one or the other. And not just nonsense, I personally think that people who even pose this to others are morally suspect.

But let me expand on my answer a little bit. A doctor or any other professional may be

  1. morally right and professionally right
  2. morally right and professionally wrong
  3. morally wrong and professionally wrong

BUT, he/she cannot be morally wrong and professionally right. To be morally wrong is wrong, period. It is this idea that Kant is trying to convey to you!

Answer 3 (score 0)

While I’m sure Philip Klocking’s answer in comments is correct, this very either-or question of two lives, in some sort of “lifeboat for one,” say, is often used to criticize the limits of actually applying Kant’s categorical imperative. Does it imply an injunction even against sacrificing one’s own life for two others, in the grand manner of Sidney Carton?

No matter how “dutiful” the good Kantian ought to be, I’m sure it is nonsense to suppose that Kant himself thought such unearthly restrictions must or could govern daily life so absolutely. They furnish “regulative ideals” to judgement along with the actual and actionable circumstances. Kant did say that “ought” implies “can,” that we will not be burdened by providence, as in Greek or Hegelian tragedy, with utterly irreconcilable moral choices. But this is not to say that one can “know” the right choice.

Thus, epistemology plays a role and offers one further appeal. Kant did reject utilitarian and consequentialist ethics of the sort that might “decide” to “save the mother” or whatever. And here his epistemology ties in with his ethics. He argued that such ethical rules assume we can actually predict the outcome, while our knowledge is always limited. This is not an appeal to miracles, but to the realities of induction and the limits of reason.

How certain are we that our decision will actually save anyone? What if the child we decide to save grows up motherless into another Hitler? History is an endless smash-up of unintended consequences. Here Kant, especially in his own historical circumstances, might indeed invoke a necessary “faith” in the moral imperative in the face of unknowns or mere probabilities. Even inaction. This is a plausible moral stance, but one that is strangely eroded by technology and the advance of historical relativism.

76: Why don’t fair coin tosses “add up”? Or… is “gambler’s fallacy” really valid? (score 18811 in )

Question

I have always been perplexed by a seeming paradox in probability that I’m sure has some simple, well-known explanation. We say that a “fair coin” or whatever has “no memory.”

At each toss the odds are once again reset at 50:50. Hence the “gambler’s fallacy.” After 10 heads, the odds of another head are still said to be 50:50. The same after 20, 40, 80… heads.

Yet we also know that the series will converge upon an equilibrium of heads:tails. And indeed this is countable in fairly short order. The convergence appears pretty quickly.

How can both be true? Isn’t there something in the physical series of tosses that “remembers”? Isn’t there necessarily some slightly better chance of a tails after 10 heads?

How does logic resolve this absolute randomness in the particular events with a general law of convergence? I imagine this must be a well-known issue. I suppose it raises the larger issue of what sort of “causality” probability is.

Note that I do not know symbolic logic so, embarrassingly, formal demonstrations are beyond my ken.

Answer accepted (score 109)

Since you have asked for a non-formal answer, I shall try to oblige by not using any numbers or equations.

Fundamentally, your question is, how does it come about that individual events can be completely unpredictable but when you pile a lot of them together, either in a sequence or in a mass, the behaviour of the whole pile becomes, if not totally predictable, at least substantially predictable? The answer is something called the law of large numbers, and it is one of the most fundamental concepts in statistics.

As an illustration of it, imagine something called a Galton box: it is a triangular shaped box standing vertically, with its base on the ground and one vertex at the top. There is hole in the top to allow a ball to be dropped in. A series of pins or pegs are placed such that a ball falls either to the right or left in an unpredictable way until it reaches the bottom. As illustrated in this diagram, when lots of balls are dropped in, the result is a heap in the middle. We cannot predict where a single ball will fall, but put enough balls in and we can be increasingly sure that we’ll get a bell-shaped curve, simply because it is very unlikely that a ball will consistently move left, or consistently move right. One way to think of it is to count the possible paths to a point on the bottom. There is only one possible path to get all the way to the right, or all the way to the left, but lots of paths will take a ball to the middle. enter image description here This means we don’t need to suppose that a ball is remembering the previous falls. Each ball is independent, and the resulting curve (a binomial distribution) emerges spontaneously from it. This is one of many examples of how apparently orderly behaviour can emerge even when there are lots of disorderly things going on at the micro level. Another is radioactive decay: we cannot predict when one atom will decay, but with a large mass of them we can predict very precisely what proportion of them will decay in a given time interval. Another example arises from the kinetic theory of heat: we cannot predict how individual molecules move around, but put enough of them together and we can say all kinds of useful things about their thermodynamic properties.

So the gambler’s fallacy is a real fallacy, even though it is perennially tempting. My favourite way to test peoples’ intuitions about it is to ask them this: suppose I decide to play the lottery every week and my preferred strategy for picking the numbers is to look up the numbers that won last week and choose those. You will find many people who think this is crazy because the chances of the same set of numbers winning two weeks running is tiny. But of course the probability of any set of numbers winning is all equal: it is not affected by the previous week’s win.

Answer 2 (score 42)

If the probability of heads = p , then the probability of tails = 1-p . If it’s a fair coin, then p = 1-p and the probability of either heads or tails is p = 1/2.

Now suppose the number of coin tosses is N, and let’s say that N is getting pretty large. The expected value of the random variable that is the number heads out of the N tosses is going to be around the mean Np, which for an honest coin is N/2.

The variance of the random variable (the total number of heads out of N tosses) is Np(1-p) (which, for the honest coin, is N/4) which is the square of the standard deviation. This means if N is increased by a factor of 4, then the standard deviation only increases by a factor of 2.

So as the number of tosses increases, the deviation of the number of heads (which is sqrt(N)/2)) from the expected mean (which is N/2) does increase, but not as fast as the number of tosses increases. When you divide by N, the percentage of that expected deviation, inside the total number of tosses, gets smaller and gets closer to the expected 50%. This is because it’s (sqrt(N)/2)/N = 1/(2 sqrt(N)) .

From a percentage POV, it looks like you’re getting closer and closer to what is expected from an honest coin.

From a count POV, it doesn’t look exactly the same. If you toss an honest coin 1,000,000 times, the number of heads will likely be some distance away from 500,000. But the percentage of the number of heads out of the total number of tosses will be very close to 50%. And it will get closer to 50% with more and more tosses, but the absolute distance away from the 50% mark will grow at a rate proportional to sqrt(N). But the number of tosses is growing at a rate of N.

Answer 3 (score 25)

The convergence appears pretty quickly.

This is your faulty assumption. It does apear pretty quickly. In most cases. But not at all every time.

There are in some sense two layers of likelyhood: In layer one, every single event has the very same probability as its precedessors. In layer two, the sequence of events as a whole has a probability to occur. And there are (finite!) sequences, although every single event is equally probable, that are as a whole unprobable. Only with infinite length every sequence is equally possible.

The problem is that you never know in “what” sequence you are, it may retrospectavely be the most unlikely sequence possible. That is why, looking to the future, only the probability of the single next event is what counts for the gambler, not the probability for the completed series.

You have to to see what the event (and object of probability) is. In the coin-example, the series so far is an event that has occured. So there was a probability for this series to occur before, but now as it has happened it is only a frequency of occurence left. The only probability is that of the next event, because it lies in the future, or of the upcoming series. But as soon as the next toss is made, there is only the probability of the now next event and the now following possible series.

Probability can only be expressed for future events!

As a sidenote the following “counter-example”: Consider three doors, you choose one that has the “probability” of containing the prize of 1/3. Now one door is opened and you have the choice to change the chosen door. What are the chances? Well, you should definately change, because your door now has the “probability” of 1/3 as before, but the other has 2/3. Here you have to consider the whole series, there is no contradiction. That is because there is no probability anymore: The prize already is behind one door, the event has happened. That is the difference.

TL;DR: Edit and Conclusion

So the fallacy, as expressed by @wedstrom in his comment, is to think that nature will correct itself, will let it happen that the series in progress will converge quickly. But nature is not an actor that does anything. And in the present, there is only past (occured events, frequency) and future (upcoming events, probability). And if the probability is independent, this has to be taken literally as independence from anything that happened in the past, no matter how scarce the occurence of the resulting series will be.

77: Kant and free will (score 18788 in )

Question

Could someone explain Kant’s view on free will to me?

As I understand it, Kant says that it is necessary that we have free will, because otherwise it would be unreasonable to hold people responsible. We hold people responsible for things, even though their decisions appear to us to be deterministic and we do not hold them responsible for the circumstances that led to their decision, but we hold them responsible for having made this decision. This would only be reasonable if their decision was not determined by the circumstances (and the laws of physics etc.) but rather by human reason, uninfluenced by any empirical conditions. Therefore, the reason must be part of the noumenal world, from where it is able to cause our behaviour.

However, I think I am understanding this wrong, because I don’t see how this implies free will or why we would hold people responsible. Kant’s reasoning seems to be:

We hold people responsible->We should hold people responsible->Holding people responsible would not make sense without free will (free from laws of physics/empirical influences)->There must be free will in the noumenal world.

However, I think I am misinterpreting him, because that first implication does not make sense to me.

Could someone help me?

Answer accepted (score 7)

First off, I want to say this is a really good question that reflects real thought on an interpretative issue in Kant studies. Second, I think you’re grasping some major things but also thinking backwards (by which I mean imposing contemporary categories on what Kant is doing).

In terms of your question, one major issue is going to be where in Kant you are reading. The account you are giving sounds closest to the Critique of Pure Reason (Kritik der reinen Vernunft). There, I take Kant’s position to be as follows:

  1. As far as empirical science is concerned, our actions are determined
  2. Moral responsibility requires our actions to be free

This is what Kant calls the “Third Antinomy” (or perhaps that is just what contemporary Kant scholars call it).

But there are some specific details where I think you may be deviating rather sharply from Kant (or at least wording them in some significantly different ways). The background for this is that Kant’s account of human knowledge is predicated on the belief that our knowledge is limited to objects (those things we place under the forms of sensibility and categories of the understanding). To put it another way, Kant is a skeptic about certain types of knowledge claims, because he thinks we never encounter things in an unencumbered way. But the manner in which we relate to objects is called “understanding.”

It turns out that our actions are renderable both through our understanding which places things under the categories of cause and necessity (to name the most relevant two). But our actions take place for ourselves under the auspices of reason which is a different faculty. The easiest way to say this is that I understand your actions, but I reason to my actions on Kant’s account.

So to return to the details in your question, you state:

Therefore, the reason must be part of the noumenal world, from where it is able to cause our behavior.

but at least on my reading of Kant, there’s a minor error here. Viz., Kant does not see this as a conclusion but rather as a premise that relates to what reasoning is. But if you’re just reading the Critique of Pure Reason as your source, then you might come up with this conclusion. The problem is that Kant changes how this works between the 1st Critique, the Groundwork, the 2nd critique, the Metaphysics of Moral, and Religion. Each of them gives a slightly different explanation of how we are morally/noumenally free yet empirically determined.


You’re correct to think this progression is wrong:

We hold people responsible->We should hold people responsible->Holding people responsible would not make sense without free will (free from laws of physics/empirical influences)->There must be free will in the noumenal world.

No Kant work states this.

There’s two important features in Kant’s ethics that do seem similar. I take it that Kant asserts (at least outside the Groundwork this becomes a faktum of reason) we have reason (Vernunft) and this means that we can choose our own actions – in fact, we can choose them despite our empirical natures. Thus, we are responsible for our actions, and thus, we are not determined.

A better but related argument has to do with Kant’s belief that the world is ultimately just / what is called in the literature Kant’s proportionality principle/thesis. Kant, for reasons that are nearly opaque, asserts in his moral philosophy that the world will ultimately turn out to be just. But this is contrary to empirical evidence about this world (evil people die happy; good people die sad; etc). Thus, Kant asserts a God brings about just desserts in an afterlife.


Depending on which work you are looking at in Kant’s moral philosophy, the basis of the claim that we are free is different.

Critique of Pure Reason -> An antinomy shows that we cannot know we are free, but we live that we are free. Reason is free and distinct from understanding. Groundwork -> Part III is an argument that we are somehow free (not well-accepted or understood) Critique of [Pure] Practical Reason -> we are understood to be free as a fact of reason. Metaphysics of Moral: Doctrine of Virtue -> we are free as a condition for a just world. Religion within the Bounds of Reason Alone -> we are free at least insofar as we make a sort of root decision that impacts our later ability to be free.

Notably, it moves from an argument in the first two works to an assertion in the latter three.

Answer 2 (score -3)

Good question. Maybe if Kant would had access to our knowledge today would he would update his thought. He was a genius, no doubt. It seems to me that noumena [(in the sense they are the (supposed/…/presumed) things themselves, which we in lack of better concept so far, we say: reality] impose certain constraints to life which has got thinking ability and life with thinking ability responds to such constraints with a set of heuristics which moral/ethics/…/“what works” are examples and maybe are just byproducts of thinking ability.

So, in the end if we eliminate free will we have to start to find another heuristic to deal with it and moral & cia are not enough for it.

Now, with AI (artificial intelligence, which I prefer to name artificial integration) we just lost ANOTHER degree of freedom… so it is about time to think about the reduction of freedom implies in terms of the “next” heuristic…

After all we are just algorithms producing biochemistry and bio-electrical signs. See for example the book of the picture… ☺

enter image description here

78: What are the important effects of studying logic? (score 18621 in )

Question

I’m working on my assignment in Logic and I’m having a hard time answering this question maybe because my insights about the subject isn’t enough because we’re just getting started, So can you please give me some insights about Logic and an example of important effects of studying logic for me to have an idea on answering the question.

Answer accepted (score 2)

Logic is applied in many different fields, for example:

  • Mathematics
  • Computing Science
  • Electrical Engineering

On a very low level it is needed to understand proof methods such as:

  • Modus ponens / tollens
  • Case distinctions
  • Proof by contradiction

It can also be used to prove basic theorems using truth tables.

But most importantly, learning logic teaches you how to think. It teaches you what is (in)correct reasoning, to recognise fallacies, to check soundness of arguments, etc. These are things that are needed in a variety of real-life situations, for example:

  • Talking about science
  • Arguments with friends, colleagues, others in general
  • Get a better understanding of debates of others (e.g. in politics)

Note: Two years ago there was a question on Electrical Engineering SE which is quite similar to yours, but geared towards digital logic.

Answer 2 (score 1)

  1. It helps you to differentiate between correct and incorrect reasoning.
  2. It helps you to do self-judgement before making any comment.
  3. It also helps to give answers which will make people rethink their statements well.

Answer 3 (score 1)

  1. It helps you to differentiate between correct and incorrect reasoning.
  2. It helps you to do self-judgement before making any comment.
  3. It also helps to give answers which will make people rethink their statements well.

79: What were the government ideals for the Socrates and Aristotle? (score 18599 in )

Question

Winston Churchill famously quipped, “The best argument against democracy is a five minute conversation with the average voter.” Plato also seems to have held such a view, where he wrote in The Republic that “philosopher kings” should lead city-states in a sort of oligarchical system.

Although Socrates, Plato, and Aristotle are often talked about in the same context, their views have historically diverged on various topics. In the case of government ideals, did Socrates and Aristotle differ in their views from Plato? If so, in what ways?

Answer accepted (score 1)

“Plato also seems to have held such a view, where he wrote in The Republic that”philosopher kings" should lead city-states in a sort of oligarchical system." The progression, or lineage, of “cities” (men) in the Republic is as follows: aristocracy, timocracy, oligarchy, democracy, and tyranny. (Remember that it is a degradation.)It seems you meant aristocratic when you said that Plato advocates an “oligarchical system.” This is an important difference. But again, is it Plato or Socrates who seems to advocate this un-decayed political form? Perhaps perusing Plato’s Laws and Xenophon’s Socratic works may help sharpen the boundaries between the political philosophies of Plato’s Socrates, Xenophon’s Socrates, Socrates, and Plato.

The Laws is an oddly pragmatic account of the political (and does not feature Socrates). Xenophon’s Socrates is, perhaps like Plato but not Plato’s Socrates, hyper-concerned with the practical side of life (and politics). The dryness often associated with Xenophon’s writings is largely a factor of his interest in the pragmatic. (The interest is often attributed to Xenophon rather than Socrates.)

Regardless, I’ve heard and read countless interpretations of the Republic that claim Plato advocates aristocracy, democracy, timocracy, etc.. Even if we can disregard the setting of the Republic (it takes place during the height of the Peloponnesian War–probably at the time when Thrace joined sides with Athens, after Pericles’ death, a ruler Thucydides calls the “first citizen” of Athens, and before the rule the thirty tyrants) and the historical situation that Plato lived in (a world without Socrates, a defeated Athens, etc.), we still cannot disregard Socrates’s audience in the dialogue. His main interlocutors are Glaucon and Adeimantus, Plato’s two brothers (all three are sons of Ariston– sons of the best). They, as do Cephalus, Polemarchus, and Thrasymachus, exhibit certain characteristics of the forms of “cities” that Socrates speaks of. But of course, the forms cities are really the forms of men (but they look at cities so as to see a men’s souls more clearly). –Hence the laughter at the beginning of Book V. Plato knows intimately that the multi-colored cloak of democracy can so easily lead to tyranny. It is the very nature of democracy to proclaim liberty and equality for everyone, including giving freedom and equality to tyrants. So it is in the democratic principle of indiscrimination that leads to the demise of democracy. (This is a pertinent question in political philosophy: If a political system guarantees the freedom and equality for all its citizens, what do we do with those citizens who seek to discriminate against others–either in act or speech?) But, of course, in the Republic, the tyrannical Thrasymachus is constrained by the democratic Polemarchus (and others), and defeated by the arguments of Socrates. He is silenced. This interaction takes place entirely in the first book of the Republic. But there is a somewhat parallel sequence in Books VII and VIII. Democracy is dangerous. But is Plato advocating against a democratic form of government? Or is he advocating against a democratic soul? –Or neither. –Or both?

Now what did Socrates think? Do we mean as an historical figure or literary figure? Both questions seem impenetrable. (Similarly, who knows what Plato actually thought?) But, of course, we haven’t even talked about Plato’s Statesman. (Personally, I’m inclined to think that Plato’s own political philosophy is most truthfully presented in the Statesman and the Laws.) But perhaps the most lucid, most practical, expression of Plato’s political philosophy can be found in his Seventh Letter. In it, (if I remember correctly), Plato claims the second best form of government (to the rule by the philosopher king) is the rule of just law. We might call this “second best” form of government a constitutional government.

Aristotle says, in his Politics, that the preferred system of government is of a constitutional form. That is, a government that is ruled by laws. Thus, it seems that Aristotle and Plato shared a fondness for “just” law. How we come to what is a “just law” is another story. But this is where the Republic and Statesman could help us.

Answer 2 (score 1)

“Plato also seems to have held such a view, where he wrote in The Republic that”philosopher kings" should lead city-states in a sort of oligarchical system." The progression, or lineage, of “cities” (men) in the Republic is as follows: aristocracy, timocracy, oligarchy, democracy, and tyranny. (Remember that it is a degradation.)It seems you meant aristocratic when you said that Plato advocates an “oligarchical system.” This is an important difference. But again, is it Plato or Socrates who seems to advocate this un-decayed political form? Perhaps perusing Plato’s Laws and Xenophon’s Socratic works may help sharpen the boundaries between the political philosophies of Plato’s Socrates, Xenophon’s Socrates, Socrates, and Plato.

The Laws is an oddly pragmatic account of the political (and does not feature Socrates). Xenophon’s Socrates is, perhaps like Plato but not Plato’s Socrates, hyper-concerned with the practical side of life (and politics). The dryness often associated with Xenophon’s writings is largely a factor of his interest in the pragmatic. (The interest is often attributed to Xenophon rather than Socrates.)

Regardless, I’ve heard and read countless interpretations of the Republic that claim Plato advocates aristocracy, democracy, timocracy, etc.. Even if we can disregard the setting of the Republic (it takes place during the height of the Peloponnesian War–probably at the time when Thrace joined sides with Athens, after Pericles’ death, a ruler Thucydides calls the “first citizen” of Athens, and before the rule the thirty tyrants) and the historical situation that Plato lived in (a world without Socrates, a defeated Athens, etc.), we still cannot disregard Socrates’s audience in the dialogue. His main interlocutors are Glaucon and Adeimantus, Plato’s two brothers (all three are sons of Ariston– sons of the best). They, as do Cephalus, Polemarchus, and Thrasymachus, exhibit certain characteristics of the forms of “cities” that Socrates speaks of. But of course, the forms cities are really the forms of men (but they look at cities so as to see a men’s souls more clearly). –Hence the laughter at the beginning of Book V. Plato knows intimately that the multi-colored cloak of democracy can so easily lead to tyranny. It is the very nature of democracy to proclaim liberty and equality for everyone, including giving freedom and equality to tyrants. So it is in the democratic principle of indiscrimination that leads to the demise of democracy. (This is a pertinent question in political philosophy: If a political system guarantees the freedom and equality for all its citizens, what do we do with those citizens who seek to discriminate against others–either in act or speech?) But, of course, in the Republic, the tyrannical Thrasymachus is constrained by the democratic Polemarchus (and others), and defeated by the arguments of Socrates. He is silenced. This interaction takes place entirely in the first book of the Republic. But there is a somewhat parallel sequence in Books VII and VIII. Democracy is dangerous. But is Plato advocating against a democratic form of government? Or is he advocating against a democratic soul? –Or neither. –Or both?

Now what did Socrates think? Do we mean as an historical figure or literary figure? Both questions seem impenetrable. (Similarly, who knows what Plato actually thought?) But, of course, we haven’t even talked about Plato’s Statesman. (Personally, I’m inclined to think that Plato’s own political philosophy is most truthfully presented in the Statesman and the Laws.) But perhaps the most lucid, most practical, expression of Plato’s political philosophy can be found in his Seventh Letter. In it, (if I remember correctly), Plato claims the second best form of government (to the rule by the philosopher king) is the rule of just law. We might call this “second best” form of government a constitutional government.

Aristotle says, in his Politics, that the preferred system of government is of a constitutional form. That is, a government that is ruled by laws. Thus, it seems that Aristotle and Plato shared a fondness for “just” law. How we come to what is a “just law” is another story. But this is where the Republic and Statesman could help us.

Answer 3 (score 1)

Plato and Aristotle were reacting against what they believed was a corrupt democracy, which, in their opinion was no better than say a corrupt dictatorship. The chief “end goal” of government—in their opinion—was to produce virtuous citizens, and if a democracy could do that, fine, but an oligarchy of the worthy would be even better.

80: What, if anything, is the difference between ethics and moral philosophy? (score 18497 in )

Question

Are the terms ‘Ethics’ and ‘Moral Philosophy’ different in extension as terms in philosophy? Some Departments of Philosophy have courses with titles like “Introduction to Ethics” and others with titles like “Introduction to Moral Philosophy.” Likewise, one sees both terms used in the titles of introductory textbooks and anthologies. Are they synonymous, or are they tracking a distinction?

I don’t mean ‘ethics’ in the sense relevant to the Code of Ethics of one or another professional discipline (such as Law or Engeneering); I mean to ask about the terms as used in philosophy.

Answer accepted (score 14)

As far as I am concerned, the two terms are completely equivalent.

Formally, “ethics” is the branch of philosophy that addresses questions about justice and morality. Thus, one could also label that branch “moral philosophy” and still refer to the same.

In colloquial usage, “ethics” is often used to mean applied ethics, which is really only a branch of the entire sub-discipline of ethics (or moral philosophy). It refers to specific controversial social or legal issues, such as nuclear war, animal rights, capital punishment, abortion, and so on. The goal there is to apply the ethical principles established in the abstract world of moral philosophy to specific real-world problems.

Answer 2 (score 11)

While their colloquial meaning do significantly overlap, I would argue for a sharp distinction between the two in terms of their philosophical implications.

Morality relies on a transcendent good-evil distinction. Very generally it means a set of constraining rules (a code) consisting in judging actions by relating them to universal values. Morality addresses itself to jurisprudence: “This is right, that’s wrong.”

Ethics indicates a set of optional rules assessing what we do and say in relation to ways of existing. Ethics addresses itself to the “art of living”: the properly ethical question is which mode of existence, which style of living. An ethical analysis would interpret actions and propositions as so many sets of symptoms that express or “dramatize” the mode of existence of the doer or speaker.

An ethical analysis seeks to identify different ways of existing and to judge their worth empirically, rather than weigh them abstractly against a set of “universal” values. So ethics is very different than morality. (I believe this can be seen especially clearly in both Spinoza and Nietzsche, who were both called ‘immoralists’ in their time.)

Answer 3 (score 3)

Etymologically, they both mean the same: ethics comes from the Greek word ethos (ἔθος), which has the same meaning as the Latin word mos (pl. mores) which is the root of the word morality. They both are translated into English with manners or customs.

As Joseph Weissman already pointed out in his answer, the meaning of ethics and morality has shifted, although they both share some parts of their meanings. In general, they are terms describing the assessment of human actions.

Morality seems to be more about the theoretical framework behind, i.e. questions about why there is such a thing as ethics or good and evil, how they come around and why we depend on them.

Ethics is more about answering day to day questions of morality. This is why it is often used in conjunction with other fields of activity, e.g. business ethics: they handle questions of good and evil within a certain context.

Interestingly, in German, the words Ethik and Moral (which have of course the same etymological background as their similar English counterparts) are usually used the other way round, with Ethik more handling the theory, and Moral being more practical. Some authors, though, use them the other way round. This, of course, could be the consequence of influence from the English speaking world.

81: What are/were the main criticisms of logical positivism? (score 18355 in 2011)

Question

Logical positivism, later called logical empiricism, was a school of analytic philosophy famously connected with the Vienna circle and with a significant following up until the 1950’s.

What were the main criticisms that were articulated to refute logical positivism, who articulated them, and why were they so successful in displacing the movement from its previous stature?

Answer accepted (score 12)

The main element of logical positivism was verificationism. This led to the attitude, that only propositions proven by verification are worth to discuss in philosophy. So metaphysics and ontology had to be ignored according to Vienna circle as they cannot be proved by objective methods.

The problem with this view, articulated mainly by Karl Popper, was that for verification of a proposition, one needs absolute truths on which he can base the verification. This is known as the Münchhausen Trilemma. Popper as founder of falsificationism in his critical rationalism, a kind of historic fallibilism, stated that there is nothing we can be 100% sure about, there is ALWAYS the chance of being wrong. We can only falsify current knowledge and develop more exact, consistent and coherent theories. This is the current paradigma believed by most natural scientists.

I foundy only a german diagram of this famous trilemma, use translation if it is not self-explaining

trilemma

Answer 2 (score 12)

The main element of logical positivism was verificationism. This led to the attitude, that only propositions proven by verification are worth to discuss in philosophy. So metaphysics and ontology had to be ignored according to Vienna circle as they cannot be proved by objective methods.

The problem with this view, articulated mainly by Karl Popper, was that for verification of a proposition, one needs absolute truths on which he can base the verification. This is known as the Münchhausen Trilemma. Popper as founder of falsificationism in his critical rationalism, a kind of historic fallibilism, stated that there is nothing we can be 100% sure about, there is ALWAYS the chance of being wrong. We can only falsify current knowledge and develop more exact, consistent and coherent theories. This is the current paradigma believed by most natural scientists.

I foundy only a german diagram of this famous trilemma, use translation if it is not self-explaining

trilemma

Answer 3 (score 1)

The nature of knowledge is different, hence, the tools to verify the knowledge are different. First, this paradigm overstated the notion of verification. Second, it failed to take into account that there are spheres which may not be proven in a scientific way.

Science has a better alternative position. Science does not deal with any notion that can not be scientifically tested. This way, its neutrality towards Metaphysics remains an important contour for its large-scale acceptance even in traditional societies.

On the contrary, Logical Positivism was aggressive and rejectionist and saw attacks on its own since no knowledge, human knowledge, is perfect. It has to go through a constant process of revision. This way an alternative discourse of what things are and how they should be and should not be failed miserably in attracting large subscribers.

82: “I was not; I was; I am not; I do not care” (score 18329 in )

Question

From Wikipedia’s page on Epicurus:

He also believed (contra Aristotle) that death was not to be feared. When a man dies, he does not feel the pain of death because he no longer is and he therefore feels nothing. Therefore, as Epicurus famously said, “death is nothing to us.” When we exist death is not, and when death exists we are not. All sensation and consciousness ends with death and therefore in death there is neither pleasure nor pain. The fear of death arises from the belief that in death there is awareness.

From this doctrine arose the Epicurean epitaph: Non fui, fui, non sum, non curo (I was not; I was; I am not; I do not care) – which is inscribed on the gravestones of his followers and seen on many ancient gravestones of the Roman Empire. This quote is often used today at humanist funerals.

Is there a term that encompasses the belief that there is no awareness in death and that it is not to be feared? Is such a belief peculiar to Epicureanism? If not, which are the other philosophies or religions that share it?

Answer accepted (score 3)

Such a belief is not peculiar to Epicurus. Here are some similar points of view:

With that said, it’s difficult to see who was or wasn’t (exactly) a monist or dualist (in the Cartesian sense) in ancient philosophy. Thales and Empedocles are particularly tricky. They are both Physicalists (this is a very loose categorization), but Empedocles mentions the transmigration of souls.

Aristotle is also difficult to place. He writes (in De Anima):

. . . the soul does not exist without a body and yet is not itself a kind of body. For it is not a body, but something which belongs to a body, and for this reason exists in a body, and in a body of such-and-such a kind (414a20ff).

So there’s a soul which is a property of the body, but we’re not sure how this relates to the nature of the particular thing or whether or not humans as well as dogs have a soul. The soul is matter and it seems to die with the body (just how it happens in Epicurus). I think Epicurus’ argument is pretty airtight. If you believe that there is no existence after death, fearing what happens during the state of death is irrational.

Even stranger is the fact that Socrates actually embraced death (as death would bring one’s soul closer to the forms). So we can actually have a position where the soul is immortal and death is actually welcomed.

83: “That which can be asserted without evidence, can be dismissed without evidence.” - Christopher Hitchens (score 18303 in 2012)

Question

In terms of science, this claim has a pretty straightforward answer. Evidence is the base of all scientific claims, and without sufficient evidence any knowledge claim is meaningless. However, what are the implications of this idea for other subjects like art, history, ethics, etc.? Do these subjects require the same type of evidence that science does? Clearly, art and ethics, subjects that tend to be highly subjective, don’t necessarily require evidence for knowledge claims, do they?

Answer accepted (score 10)

Clearly, art and ethics, subjects that tend to be highly subjective, don’t necessarily require evidence for knowledge claims, do they?

Of course they do.

What would it mean to make a claim completely devoid of evidence?

Do you think there is any philosopher who claims that murder is wrong, or that a work of art is beatiful, for no particular reason?

Answer 2 (score 4)

art and ethics, subjects that tend to be highly subjective,

I agree with Michael D. and I’ll go even further : arts and ethics can be philosophical objects, and as so, they are part of a science. You may not consider philosophy a rigourous science, but it is, for its strengh comes from observations, deductions and conclusions. It’s not always that simple, but that’s how philosophers like Leibniz built their whole theories. Read Spinoza’s Ethics or his Theologico-Political Treatise, and see if his conception of politics, religion and moral is asserted without justification.

Answer 3 (score 1)

Does Christopher Hitchens’ very statement (apparently known as Hitchens’ razor) have any evidence to support it? This is doubtful, because it expresses an ought, and we have the naturalistic fallacy telling us that one cannot reason from is to ought. Unless we can provide ‘evidence’ for ought, upon Hitchens’ argument, every single ought can be dismissed ‘without evidence’. That includes his own statement!

What does this do to ethics and morality? We must be careful to not appeal to people’s feelings or sensations, for those are all is-type statements. In his After Virtue, Alisdair MacIntyre argues that without a telos (final purpose/end goal), morality and ethics collapse into Nietzschean power-plays, sometimes masked by appeals to emotivism. Copying a bit from this answer, MacIntyre says on p53:

Each of the three elements of the scheme—the conception of untutored human nature, the conception of the precepts of rational ethics and the conception of human-nature-as-it-could-be-if-it-realized-its-telos—requires reference to the other two if its status and function are to be intelligible.

If we cannot know man’s telos—final purpose—then where does morality get us? Likely toward some covert telos shared by a subset of the population in the form of their competing-but-cooperating desires for power. If ‘evidence’ is required for knowing anything and this ‘evidence’ is exclusively of the scientific kind, then there are two options (have I missed any?):

  1. Ethics is meaningless because it isn’t based on evidence.
  2. Ethics is built on something other than evidence and must be judged based on something other than evidence.

So I will give a qualified yes to Hitchens’ argument: it is true in a way I don’t think he intended. I think he meant to say:

That which can be asserted without evidence, can be dismissed without evidence.

This form is fallacious, because it is self-defeating, just like Hume’s Fork, which is (arguably?) a similar argument.

84: Why is Nietzsche so against Socrates? (score 18295 in )

Question

Nietzsche recalls the story that Socrates says that ‘he has been a long time sick’, meaning that life itself is a sickness; Nietszche accuses him of being a sick man, a man against the instincts of life, and hence a ‘monstro animo’ (a monstrous soul); Nietzsche is for war.

And because Socrates is the beloved of Plato, his reed-flute which his words and thoughts are refracted through he is against Plato too.

But far from being an ascetic Socrates married late, and then a much younger woman (Xanthippe); he says of her in Xenaphons Symposium:

And that is just my case. I wish to deal with human beings, to associate with man in general; hence my choice of wife. I know full well, if I can tolerate her spirit, I can with ease attach myself to every human being else.

He had three sons, and served in the army in three campaigns and with valour; he was a master-mason and ran a sophist school; and as Epistates (president of the assembly) in the Battle of Arginusae he resisted the judicial murder of the Generals for the failure to rescue sailors during battle.

This is hardly a man it seems who is against the instincts of life; so why Nietszches polemical calumny?

Answer accepted (score 3)

I think Nietzsche uses Socrates as a strawman for all kinds of ideas he wishes to counter.

For instance, in The Birth of Tragedy Out of the Spirit of Music he laments the passing of an archaic time when both Dionysian and Appolonian influences determined art (and life) and he criticizes Euripidean drama, in which he sees the influence of Socrates:

Thus, the Euripidean drama is simultaneously a cool and fiery thing, equally capable of freezing or burning. It is impossible for it to attain the Apollonian effect of the epic, while, on the other hand, it has divorced itself as much as possible from the Dionysian elements, and now, in order to work at all, it needs new ways to arouse people, methods which can no longer lie within either of the two individual artistic drives of the Apollonian and the Dionysian. These methods of arousing people are detached paradoxical ideas—substituted for Apollonian objects of contemplation—and fiery emotions—substituted for Dionysian enchantment. The fiery effects are, to be sure, imitated with the highest degree of realism, but the ideas and emotional effects are not in the slightest imbued with the spirit of art.

Hence, if we have recognized this much, that Euripides was not at all successful in basing his drama solely on Apollonian principles, that, by contrast, his un-Dionysian tendencies led him astray into an inartistic naturalism, we will now able to move closer to the essential quality of Socratic aesthetics, whose most important law runs something like this: “Everything must be understandable in order to be beautiful,” a corollary to the Socratic saying, “Only the knowledgeable person is virtuous.”

I think what Nietzsche has against the strawman is that he (as well as Nietzsche) investigates a domain besides the simply known, the rational (starting from “I know that I know nothing”), but instead of respecting it is as raw and un-knowable (i.e. Dionysian) he wishes to analyze and penetrate it. So as far as I can see, Nietzsche blames Socrates (or “Socrates”) for raising a veil.

Of course and as always there are multiple threads in Nietzsche’s argument, and some of them contradict each other: it is perhaps the author’s way of supporting Dionysian principles against the strawman (who does not like logical contradictions and wants to get rid of them).

Answer 2 (score 2)

Actually, in response to the comment about the rooster, Asclepius is the god of healing in Greek mythology. Neitzsche’s complaint in this respect is that Socrates is basically saying death is his cure. Neitzsche’s interpretation of that is that Socrates secretly hated life and welcomed death. Socrates’ arguments for the virtues of death in other parts of the Phaedo support this as well.

Answer 3 (score 2)

Actually, in response to the comment about the rooster, Asclepius is the god of healing in Greek mythology. Neitzsche’s complaint in this respect is that Socrates is basically saying death is his cure. Neitzsche’s interpretation of that is that Socrates secretly hated life and welcomed death. Socrates’ arguments for the virtues of death in other parts of the Phaedo support this as well.

85: Did Aristotle say “The more you know…” (score 18295 in )

Question

One finds all over the web the words attributed to Aristotle “The more you know, the more you know you don’t know.” Can that be right? It sounds a bit adorable for Aristotle, and the citations are always simply to Aristotle, with no work cited.

Did he say something close to this? Where?

Answer accepted (score 3)

If you view Aristotle’s “Metaphysics” at http://classics.mit.edu/Aristotle/metaphysics.mb.txt and search for the word “know” (including words with “know” n them), you will see that he strongly suggests it several times, though not as a direct quote, at least not in this translation. Perhaps the most relevant paragraph:

“Some of the sensible substances are generally admitted to be substances, so that we must look first among these. For it is an advantage to advance to that which is more knowable. For learning proceeds for all in this way-through that which is less knowable by nature to that which is more knowable; and just as in conduct our task is to start from what is good for each and make what is without qualification good good for each, so it is our task to start from what is more knowable to oneself and make what is knowable by nature knowable to oneself. Now what is knowable and primary for particular sets of people is often knowable to a very small extent, and has little or nothing of reality. But yet one must start from that which is barely knowable but knowable to oneself, and try to know what is knowable without qualification, passing, as has been said, by way of those very things which one does know.”

The point he’s making is the opposite of what the quotation suggests: Aristotle believes that all things are knowable, but, in a specific individual’s journey towards complete knowledge, there will be times that new knowledge shows there are more things to learn.

The last instance of ‘know’ occurs in a somewhat weak sentence “But evidently in a sense knowledge is universal, and in a sense it is not.”

Answer 2 (score 2)

I found cites, though nothing specific, to Socrates, Lao-Tse, George Bernard Shaw, and Aristotle. I did some word searches in the works reprinted at Project Gutenberg. But nothing helpful.

My best guess: the original quote has become so corrupted over time that searches for a source will produce everyone and no one.

Answer 3 (score 1)

Isn’t this quote atributed to Einstein? Probably derived from Socrates quote: I know that all I know is that I do not know anything.

86: What is the Difference Between Human Rights and Natural Rights? (score 18136 in 2014)

Question

When people discuss the human rights and natural rights, there seems to be a distinction between the two, where standard interpretations of human rights (such as the Universal Declaration of Human Rights) are broader and incorporate rights, such as the right to education, which are not seen in explanations of natural rights. Natural rights, such as Locke’s “life, liberty and estate” seem to be more limited in scope than human rights, yet also more fundamental.

What is the difference between natural and human rights? Are natural rights simply a less developed and more concise version of what we now consider human rights? It seems to me that natural rights and human rights both extend from our status as rational humans, but there may be some distinction between the two that I am missing.

Answer accepted (score 7)

Natural rights and human rights originally come from different vocabularies. It’s not fair to construe natural rights as “simply a less developed and more concise version of what we now consider human rights.”

First, I want to start by pointing out an important but crucial ambiguity in the term “natural rights.” Viz., the problem is that “nature” can mean many things. Thus, one meaning of natural rights are those rights granted to an individual by Nature (though in naturalized versions not capitalized); another meaning is those rights granted to a person as a result of their nature. Thus, the US Declaration of Independence speaks of natural rights as those granted by the Creator in making humans the way they are. The second meaning of natural right could be stated in saying people have rights to food, community, and procreation, because these follow from human nature. The senses are somewhat integrated in that most rights granted by Nature (as God, Nature, nature) are generally presumed to accrue as consequences of the sort of creatures that are made.

To make it simpler with an analogy, if God declares we have the right to free tacos, then this would be natural in the first but not the second sense, but if God makes us so that we need food, then we may have a natural right in the second sense to the provision of food [thought not necessarily tacos].

Human rights can differ from both of these senses or overlap. Here, the adjective identifies those things which accrue to humans (an objective genitive) rather than those things that follow from being human. Thus, any right that we think people deserve and/or we grant to people is a human right if we grant it to them because they are human.

To give an example, the UN declares that all humans have the right to a country of citizenship. This can be understood in one of two ways. Either you could believe this follows from the nature of human beings and is thus a human right we should recognize or you believe that this is a right we have given humans by legal force and thus becomes a human right.

Again, we can return to our example. If the UN council passes a bill saying we deserve free tacos, then we have a human right to free tacos in those places where UN pronouncements mean something. The UN might argue that these rights follow logically from what we are in which case they are claiming a natural right-basis for this human right or rather that they are merely pronouncing an already existing natural right and codifying it.

We can now return to the sense in which you viewed human rights as progress over natural rights. This is definitely there in the literature. But this does not mean human rights are necessarily better. What it means is that our idea of rights has shifted. When rights talk first takes off in the natural rights form, it is seen as the perception of a pre-existing right inherent in nature (in the second sense) or a right God/Nature has woven into the fabric of existence (first and second senses together). Human rights talk can also include positive rights – rights that exist only because we’ve decreed that they do. But with this comes the critiques.

Human rights can wind up being arbitrary precisely insofar as they are not necessarily grounded in nature. They also tend to be revocable or if rights with no basis are granted, deleterious to a society. Moreover, this type of rights talk requires a certain Western type of legal system. Thus, we see many critiques from Chinese philosophy (see Confucian Ethics: A Comparative Study of Self, Autonomy, and Community and the work of Theodore de Bary) and from Western communitarians (e.g. Alasdair MacIntyre) and from feminists sand others.

Answer 2 (score 2)

“Natural rights” refer to the right to do (and it is by the nature of being human that humans have rights, which is how “human rights” and “natural rights” get equated). “Human rights” as used in the OPs reference and in most current usages include various rights to have, which is how people can now speak of a right to be provided an education, be provided with potable water, and whatever else you may have encountered.

Answer 3 (score 1)

From my perspective, the difference between Natural (derived from nature) and Human Rights (applicable to humans), is that Human Rights are a subset of Natural Rights. In addition to being applicable to Humans, Natural Rights apply to animals and (possibly) plants, as well. Thus, Natural Rights is a larger set than Human Rights.

87: Is there an alternative to the scientific method? (score 18115 in 2015)

Question

Intro

The scientific method is a key process of how we acquire knowledge and may shape our understanding of the world. If I am not mistaken, this method has been defined several times during our history.

The scientific method is not free from disadvantages. I don’t want to talk about practical issues such as the statistical bias caused by the fact that work that demonstrates effect gets more easily published than work that demonstrates absence of effect. I am asking about the fundamentals of the scientific method.

Broad Question

I am asking whether anything better than the current scientific method could theoretically be achieved? Is there any reason why the current method has to be the best method for acquiring knowledge or can we imagine anything better?

Going into the specific

Typically, I am curious about the process of proving the hypotheses wrong. If I am not mistaken, no hypothesis can be proven correct, we can only prove hypothesis wrong. Such methodology very much follows the method of hypothesis testing in statistics. Taken from my memory of the work I read from Elliot Sober, a long time ago, this method leads to the important issue that one can only discard all the hypotheses that (s)he can imagine, but can never be sure that he would not have missed some hypotheses. One can never prove anything right. One can only prove things wrong.

Can’t we imagine a method that would not be based on rejecting hypothesis? Is there a fundamental restriction in how one can acquire knowledge that forces us to use this seemingly sup-optimal method?

Answer accepted (score 28)

(Edit: this answer is now split into two parts, thanks to a lengthy discussion with Rex Kerr. I made my original answer on a very specific reading of the scientific method. He had a very different reading, which came to a different but very related outcome. I’ve tried to capture that in the first part. The second part is my original answer, for those who wish to use the stricter reading)

There are at least two extremes as to how one can define the scientific method. One is a process, one is more of a set of principles and a goal. The process is well defined as:

  • Observe something interesting
  • Formulate a hypothesis that you think would model this interesting thing better than existing model.
  • Run a series of independent tests of the hypothesis.
  • Statistically demonstrate that the original model (often called the null-hypothesis) predicts the outcomes of the tests to be highly unlikely.
  • Reject the null hypothesis (assuming the data backs your claim)
  • Demonstrate that your new model does a better job of predicting the statistical results.

This is what I was taught the scientific method was in high school. If that is the version you are after, skip ahead to the second part, which explicitly targets that reading.

However, there is another more fluid reading which also exists. The statistical requirement is relaxed, because it can cause trouble. However, there is a focus on both the elimination of hypotheses through testing and the preference towards hypotheses which are testable. This reading of the scientific method is a very general direction, so the alternatives are equally general.

Science is a very deductive learning approach. It depends upon one writing a hypothesis in highly objective terms and then testing it. There are many situations where deductive learning does not work. Procedural learning is often viewed as an alternative approach. Consider the case of an athlete. They collect large amounts of information from scientific approaches, but the final bit that takes them from a “good athlete” to a “great athlete” is all “feel.” There may be no written hypotheses. There may be no statistical testing. Yet, the mind absolutely learns in this way. Thus, procedural learning like this would be a valid alternative method. In fact, many Chinese martial arts focus almost entirely on procedural learning because it is so hard to learn deductively.

Which reading of the scientific method you want to use is up to you. What follows is written entirely from the perspective of a strict statistically valid approach to the scientific method. Much of what has made science great is its ability to build upon previous hypotheses. While statistical rigor is a nicety for rejecting hypotheses, it becomes essential for building hypothesis which can support others.


Finding an alternative to the scientific method depends on you deciding what you want out of a method. You will never find a better tool than the scientific method at it’s game. However, if that game is not what you really want it to be, there are alternatives.

The most visible example of this I have seen is western medicine compared to Traditional Chinese Medicine. They developed very different approaches, and yet both appear to yield results. TCM actually does work on line along your questioning: it is not fundamentally built off of rejecting hypothesis.

Let’s look at the scientific method, and see if we can make some headway. There are two major features of the scientific method which stand out as “interesting” for this line of thinking:

  • The scientific method is highly steeped in the language of statistics.
  • The scientific method seeks objective theories.
  • The scientific method tests theories.

These are traditionally seen as strengths. However, they can also be seen as weaknesses (like all good superheros, their strength is their weakness.. that’s what makes them interesting).

The scientific method is completely and utterly useless without statistics. This means any singular event is completely beyond its reasoning. It cannot provide answers to topics such as “the purpose of your life” because there is only 1 you, and N=1 means there is no statistics.

Related to this, the scientific method strives to be objective. It always tries to remove the observer from the picture. This is very valuable, because it ensures that your discoveries are applicable to others. However, it also proves to be tricky in many situations. Social studies in particular have great difficulties with the scientific method because it is so very difficult to make good tests that keep the observer out of the loop. As an example, TCM claims that acupuncture works. Those who have tried it, claim it works with uncanny success. However, science has had fits trying to find any effect of acupuncture beyond the infamous cop-out “the placebo effect.” The issue is that it is almost impossible to develop an effective control to measure against because the acupuncture practitioner knows if they are doing it right or not. Whether you believe acupuncture works or not depends heavily on whether you accept results which lack a solid control to ensure objectivity.

Finally, science tests its theories. This sounds absurd, because it seems so obvious that you should test them. However, a theory is not accepted at all until it is tested. The result is that anyone with a theory must expend the resources to do the testing before science will do anything with it. Other approaches get away with a different style: you use a theory once you have it, and you test it when you get an opportunity to do so. The tests can also be dangerous. (Edit: I had a reference to the LHC and potential to create black holes here, but it was too contentious. Instead, it has been replaced with a hypothetical example) Consider a hypothetical particle physics experiment. The scientist is rather confident that their theory is correct. They begin experimenting, after calculating that they would like 100 samples to do statistics on. Generally speaking, they are finding their theory holds out for test after test. However, on tests which disagree with their hypothesis (which happens in the scientific method due to noise), the observer notices a burst of energy from the test apparatus. That burst becomes stronger and more dangerous with every data point that disagrees with their hypothesis. At some point, the scientist decides to cut the experiment short, because they are uncomfortable putting their life at risk to finish the test. By the strictest reading of the scientific method, that data cannot be analyzed because it is tainted with the scientist’s choice to cut the tests off early. This might induce biases because the scientist is more likely to cut them off faster if the results look good for their theory. Other methodologies are capable of using this data (including the intuition of that scientist, who will not try the exact same experiment again).

Seeing that the strengths and weaknesses of science are so confounded, it is up to each individual to decide if those are ideal for them. There are many others, none so visibly different from the scientific method than that of TCM. As described to me in a lecture, the difference is in the approach towards healing the human body:

  • Western medicine tears the body apart into components, develops hypotheses about these components, then builds them up. At each step, it develops testable hypotheses, and tests them. From there, it finds things which may provide results, and tests those.
  • TCM starts with the body as a whole, finds things that cause good results, then develops testable theories about why the results occurred.

The end result is that much of TCM is doctor-centric. A doctor finds out what works well for them, and suggests it to others. The focus is less on rejecting bad hypotheses, and more on finding new good hypotheses. TCM relies more on natural attrition to weed out the hypothesis, rather than actively trying to disprove them.

Can I claim one is better than the other? I’m not sure if I can. However, I do feel comfortable claiming that they are different, and that a remarkably large number of individuals consider one better than the other in both directions. It’s simply another way to approach things.

Answer 2 (score 22)

If I am not mistaken, no hypothesis can be proven correct, we can only prove hypothesis wrong.

This is known as “falsificationism”. It viewed with much scepticism by today’s philosophers of science. The author you mention, Elliot Sober, has suggested that it be retired, deriding it as “Popper’s f-word” (referring to Karl Popper, whose own views on this subject shifted somewhat over time).

It is untrue in the exact way you described it, because if a hypothesis A is wrong, then there is another hypothesis, called A-is-wrong, that is correct. So if it is possible to prove some hypotheses wrong, it must also be possible to prove some hypotheses correct.

But the real problem is that you are speaking in terms of absolute certainty, so you are crediting the scientific method with powers no method could possess.

All evidence has multiple (perhaps infinitely many) possible explanations. One interpretation is always available: the evidence might be flawed, in which case it can be ignored.

So all knowledge is conditional. Outside of mathematics, knowledge is conditional on unreliable evidence. But even mathematical deduction produces theorems that are true only because other theorems on which they depend have been shown to be true, and ultimately the whole edifice rests on basic assumptions (axioms) that are simply assumed to be true. Or rather, all mathematical truth is conditional on the truth of the axioms.

So we need a way to compute the certainty of a deduction based on the certainty of the facts that deduction relies on. That’s what probability theory is.

Not for nothing did Laplace observe that the whole system of human knowledge is tied up with probability.

Answer 3 (score 13)

You are starting from the hypothesis that your understanding scientific method is correct and complete, and that everyone else has the same understanding. Neither of which is sustainable from the evidence here.

There are no ‘weaknesses’ with scientific method, publishing and grants are issues of personality not science, as even scientists have personalities. You are correct in suggesting a hypothesis can not be ‘proven’ true but that is its the primary strength, and not an inherent weakness as you imply.

At its simplest, scientific method is a process for producing useful explanations for how the universe works. It is a stunningly simple process with only four steps:

  1. observe a phenomenon
  2. produce hypothesis to explain it
  3. use hypothesis to predict previously unseen phenomena
  4. experiment to observe unseen phenomena.

If phenomena matches predictions, then hypothesis is useful because it describes what is there AND it led to new knowledge. If phenomena not observed or doesn’t match predictions, then either experiment is insufficient or hypothesis is not useful.

It has been suggested that it should be match reality, but this is not correct (or at least is misleading) as demonstrated by both Relativistic and Quantum physics, where the theories were completely outlandish when proposed and it was years before experimentation could confirm their predictions.

It does not matter who comes up with the idea, nor does it matter who performs the experiment, so it is free from bias in that respect. Also, anybody can show that any idea is incomplete if they come up with the appropriate experiment that shows predicted results are not there or not correct, but this does not make the hypothesis false (since it was never ‘true’ to begin with) it simple puts limits on its usefulness. Newtons laws of motion will put a satellite in space, so they are clearly useful, but if you want accurate GPS then you need to use the improvements and refinements proposed by Einsteins theories on relativity.

Footnote - Please notice that there is no mention of the ‘S’ word anywhere in this answer. Contrary to popular belief, it is not a core feature, it is just a very useful tool, one of many.

88: What is it that Leibniz calls a “Monad”? (score 18097 in 2011)

Question

What is a “monad” as introduced by Gottfried Leibniz in his Monadology? What is the purpose of monads for Leibniz’ philosophical arguments?

EDIT: I’ve found basic definitions like those found in the comments, too. However I also found rather confusing properties of monads, like they where “living mirrors of the universe” and can be sleeping or awake. So is the idea really that those monads (of which I think like “atoms”) are like “minds” or living beings?

EDIT: Is there a notion of time / order of actions / cause associated with monads?

Answer accepted (score 9)

However I also found rather confusing properties of monads, like they where “living mirrors of the universe” […] Is there a notion of time / order of actions / cause associated with monads?

Monads mirror – each one under its own aspect – the entire universe, and these mirrorings involve not only the present, but also past and virtual future states. But this must not be understood as causal influence or interaction, because this would contradict their being “fensterlos” (window-less) and their existence as substantial units. What is perceived as effect is really based on an ideal influence established by God, who by thinking through all possible states and courses of the world, accounts for every substance. This is the principle of “prästabilierte Harmonie”.

[…] and can be sleeping or awake. So is the idea really that those monads (of which I think like “atoms”) are like “minds” or living beings?

Leibniz uses the term Monad for individual substances, which have no parts. (“La Monade … n’est autre chose, qu’une substance simple, qui entre dans les composés; simple, c’est à dire, sans parties”) This is a continuation of, and includes, scholastic traditions (“ens et unum convertuntur”).

This is in contrast to Descartes’ dualism. Leibniz takes the modern concept of individuals, well-founded in Descartes’ cogito argument, and lifts the soul-having “I” to being the substance. The infinite number of ideal, individual, and dynamic substances, which all mirror the entire universe, is Leibniz’ answer to Descartes’ dualism, Spinoza’s monism, and Gassendi’s atomism.

Only organic creatures (humans, animals, plants) are such units, not corpora which are mere aggregates and phaenomena. Decomposition of composites will, according to Leibniz, not lead to material units, because continuous division of corpora will not lead to something that could be called “unum per se”. They are “metaphysische Punkte” (metaphysical points), can neither be made or destroyed physically, and are all different.

Monads are dynamic, and only differ by their inner states, their “Perzeptionen” (perceptions). Their dynamicism is due to an inner urge (“appetition”), which advances them from perception to perception. Leibniz calls monads, whose perception is accompanied by recollection souls. (So, in contrast to Descartes, according to Leibniz, animals have souls) Monads which represent “die äußeren Dinge” (the outer things) must be distinguished from “Apperzeptionen”, which means self-awareness or reflexive cognition. Monads capable of this are called “vernünftige Seelen” or “Geister” (intelligent souls or spirits, “animae”). Now, there is a hierarchy of monads with regard to their apperceptions: From “schlummernden” (slumbering) up to the omniscient monad, the “monas monadum” (monad of monads) – God. Somewhere in between are animal and spirit monads. Also, every monad has a dedicated corpus, a body which is itself assembled from parts, which themselves have their dedicated monads, lower in the hierarchy. Only the monas monadum exists without one.

All in all, this is a very difficult topic. I hope I was of any help, and did not add to your confusion. I would not claim that I understand Leibniz’ system myself in its entirety.

Source: Historisches Wörterbuch der Philosophie, Bd. 6: Monade, J. Ritter et al., 1971

Answer 2 (score 7)

The essential characteristic of monads is that they are irreducible; in this way they resemble (philosophical) atoms, which are similarly irreducible to simpler components. Where atoms and monads differ is that atoms are viewed as purely physical, and are therefore tiny. Monads, on the other hand, are immaterial, and are units of “force”; physical attributes (such as matter or motion) are merely phenomenal.

It may be worth pointing out that Leibniz came to this theory after reading Chinese philosophy, and that there are similarities to be found between his thought and that of, for example, Fazang, the 7th century Buddhist philosopher. I think that this element of his project often receives less attention than his mathematical, scientific and logical work.

Answer 3 (score 3)

Others have already commented on the concept itself, and I am certainly not a Leibniz expert, so I’d just like to recommend a look at The Fold: Leibniz and the Baroque, which deals with the concept of the monad in some depth. Google says the words ‘monad’ and ‘time’ show up together on 19 different pages of the text; from the fourth result of that query:

If space-time is not an empty area, but the order of coexistence and the succession of monads themselves, the order has to be marked out, oriented, vectored; in the instance of each monad movement has to go from the more-clear monad to the less-clear monad, or from the perfected accord to the less-perfected accord, for the clearest or the most perfected is reason itself. In the expression ‘preestablished harmony’, ‘preestablished’ is no less important than ‘harmony’. Harmony is twice-established: by virtue of each expression, of each expressant that owes only to its own spontaneity or interiority, and by virtue of the common expression that establishes the concert of all these expressive spontaneities. It is as if Leibniz were delivering us an important message about communication: don’t complain about not having enough communication, for there is always plenty of it. Communication seems to be of a constant and preestablished quantity in the world, akin to a sufficient reason. (Gilles Deleuze, The Fold 154)

89: If you place a pencil in an opaque box and close the box, does the pencil exist? (score 17838 in )

Question

I’m trying to explain to my friends about things existing. I gave them this question: if you place a pencil in an opaque box and close the box, does the pencil exist? They say yes and I ask how do they know and why. All they come up with is “because I put the pencil in there”. I’m having a tough time explaining why the pencil ceases to exist once you close the box.

Answer accepted (score 97)

If you shake the box, it rattles. If you measure its weight before you put in the pencil and after, it will have increased by exactly the weight of the pencil. That’s how you know the pencil still exists in there.

And if you really want to explore the basic meaning of “existence”: how and why do you know the pencil exists before you put it into the box? How and why is this different from the rattling and weight increase?

Answer 2 (score 68)

I’m having a tough time explaining why the pencil ceases to exist once you close the box.

Because you’re trying to explain something which is wrong physically and wrong philosophically. Your friends are correct.

The issue is that you have no proof of its presence or absence once you close the box. That does not mean it ceases to exist. It just means that you cannot prove whether it still exists, or whether it ceases to exist at some point whilst the box is closed, or even whether it ceases to exist at the moment the box is closed and reappears at the moment the box is opened, or flickers in and out of existence, or becomes an alien spaceship when you’re not looking.

Absence of evidence does not mean evidence of absence - that’s a basic logical fallacy. It simply means we don’t know.

Answer 3 (score 51)

The assumption that the pencil continues to exist - even when the box is closed - is the most simple hypothesis which explains all relevant observations. E.g., the observation that the pencil exists when opening the box, as @Mauro ALLEGRANZA explains.

90: What are the major branches of philosophy? (score 17728 in 2013)

Question

What are the major branches of philosophy?

(For instance, as a first-order approximation, mathematics can be sub-divided into three main categories at the first level: Analysis, Algebra, and Geometry.)

What are the first few books that every person attempting to “learn” these areas of philosophy at an undergraduate level read?

Answer accepted (score 43)

The traditional branches of philosophy generally include

  • Aesthetics
  • Epistemology
  • Ethics
  • Logic
  • Metaphysics / Ontology

We can go ahead and add a few contemporary branches on to this (more examples could certainly be adduced):

  • Philosophy of Science [referring to the hard sciences] (Mathematics, Technology, etc.)
  • Philosophy of Politics [referring to the social sciences] (Society, etc.)
  • Philosophy of Religion [actually two things – question related to the existence of God or gods AND questions relating to the sociological phenomenon of religion. Generally, referring to the former]
  • Philosophy of Value (Axiology)
  • Philosophy of Language
  • Philosophy of Mind

Philosophy of language and philosophy of mind are sometimes done in ways that mirror the philosophy of the hard sciences [e.g. cognitive science] and sometimes not.

In addition, we could add many schools and movements (again, more examples could certainly be added). From the 20th century alone, there is:

  • Phenomenology (Husserl)
  • Schizoanalysis (Gilles Deleuze and Felix Guattari)
  • Chaosophy (Felix Guattari)
  • Non-philosophy (Francois Laruelle)
  • Existentialism (Sartre, et al.)
  • Vienna Circle/Logical Positivism

Finally, we can also consider the history of philosophy itself as constituting a series of fruitful conjunctions of cultures and eras where ideas were changing. The structure here is taken from Wikipedia’s article on the History of Philosophy:

  • Western philosophy

    • Ancient philosophy
    • Medieval philosophy
    • Renaissance philosophy
    • Modern philosophy
    • Contemporary philosophy
  • Eastern philosophy

    • Indian philosophy
    • Persian philosophy
    • Chinese philosophy
    • Japanese philosophy
    • Buddhist philosophy
  • Abrahamic philosophy

    • Jewish philosophy
    • Christian philosophy
    • Islamic philosophy

Answer 2 (score 23)

I believe that following list will cover the main areas of Philosophy:

Metaphysics (Study of Existence - What’s out there?)

Epistemology (Study of Knowledge - How do I know about it?)

Ethics (Study of Action - What should I do?)

Politics (Study of Force - What actions are permissible?)

Aesthetics (Study of Art - What can life be like?)

Apparently there is a philosophy of logic and philosophical logic which do differ.

Philosophy of logic is the arena of philosophy devoted to examining the scope and nature of logic.

Philosophy of logic is the investigation, critical analysis and intellectual reflection on issues arising in logic. The field is considered to be distinct from philosophical logic.

Philosophical logic is the branch of logic concerning aspects other than or outside of formal logic.

Philosophical logic is the application of formal logical techniques to philosophical problems

Philosophy of Computer Science - is concerned with philosophical issues that arise from reflection upon the nature and practice of the academic discipline of computer science.

Philosophy of science - is concerned with the assumptions, foundations, methods and implications of science.

Answer 3 (score 6)

Nigel Warbuton (Open University lecturer of Philosophy)’s books cover the basics very well. Philosophy the Basics is the best one to cover the main areas, the books is broken down into:

  • God and ontology
  • Right and Wrong, ethics and morals
  • Politics: democracy and freedom
  • The external world: realism, am I dreaming etc.
  • Science
  • Mind and body: dualism
  • Art

91: Difference between Science and Arts (score 17560 in 2017)

Question

“Science provides an understanding of a universal experience. Arts provide a universal understanding of a personal experience.” Mae Jemison

Here is what Mae Jemison’s says about her claim (source):

The sciences, to me, are manifestations of our attempt to express or share our understanding, our experience, to influence the universe external to ourselves. It doesn’t rely on us as individuals. It’s the universe, as experienced by everyone, and the arts manifest our desire, our attempt to share or influence others through experiences that are peculiar to us as individuals.

On a post on this website, artm gave this explanation of the meaning of this quotation:

science explores and makes available facts about the universe - “universal experience” in a sense that we’re all subject to experiencing them, while arts attempt to exposing to everyone (“make universal”) those special facts that normally are only available to a particular person.

On the same post, ChristopherE said the following :

[..] I am not sure why we shouldn’t think of science as also aspiring to providing understanding for everyone, and I am not sure what to make of art that tries to capture ideal types of things as they’re experienced by everyone. (That is, I am not so sure art necessarily starts with the personal.)

My first question is the following: Does Jemison’s claim seem plausible to you? Does it make sense?

What differences (and similarities) between arts and science do you think is worth to point out?

I think that for the purpose of this question, when using the word “science” we actually use a word that encompass both science and philosophy. Should we split these two notions appart in order to make sense of Jemison’s quotation?

Answer accepted (score 1)

First of all Mae Jemison talks about human creativity and how science and arts are its two manifestations. Not the only ones, but very important ones nevertheless. Her goal, as I understand it, is not to highlight the differences between the two, but to remind the audience of their commonalities: their being the tools of getting to know the world through - among other things - creativity.

Does science “[provide] an understanding of a universal experience”?
Science … is a systematic enterprise that builds and organizes knowledge in the form of testable explanations and predictions about the universe.

(wikipedia)

Do testable explanations and predictions about the universe constitute understanding of a universal experience? I don’t see why not. Universe is our universal experience and to explain / predict it is to understand it.

Do arts provide “a universal understanding of a personal experience”?

This is more problematic as there is no agreement as to purpose of the arts, and even the very definition of arts is controversial. On the other hand this lack of agreement on meaning and purpose of the arts gives Mae free rein to endow the arts with a purpose of her own choosing.

Compare to Dewey’s views on aesthetics:

Aesthetic experience is distinguished from other imaginative experience by the fact that the meanings embodied are especially wide and deep. Although scientific inventions are also products of imagination, works of art do not operate in the realm of physical existence. A work of art concentrates and enlarges immediate experience, directly expressing imaginatively-evoked meaning. It also encourages its audience to carry out a similar imaginative act.

(Notice, how the arts and sciences are again mentioned together, being a two manifestations of imaginative experience).

That immediate experience is what Mae calls personal experience. And the encouragement of the audience to carry out a similar … act is the arts’ way to make this experience universal.

So no, I don’t disagree with the talk in general or the quotation in particular.

Answer 2 (score -1)

I found this thread on a Google search for this thread title. Luckily I found you all having a very tight and nice conversation.

I think it might be worth pointing out how both of the words “arts” and “sciences” are used. They are both used as nouns and verbs. Both as processes, and products. As well as being thought of as two distinct philosophies, and as such- imagining groups of adherers to these philosophies.

I think that the process of science attempts to come up with some objective truth that can be tested and relied upon absolutely.

I rely upon the CD player to play the CD, and that the CD will retain the encoded music that I expect it to, and not some other. It will not play someone else’s music, because of some scientific laws that have been established.

But, I cannot rely upon the music coming out of the speaker to make me feel the same way I first heard it. Nor can i rely on those musicians, or any others, to play that music exactly the same way again. I say that is “art”.

Theoretically, a “robot musician” that uses scientifically tested hardware and software, could play it exactly the same way every time. Further, any deliberate change to that hardware or software, would produce a predictable and reliable result. I say that is “science”.

I hope this adds to the conversation, and does not derail it!

92: What is modal logic for? (score 17446 in 2014)

Question

I understand “pure” logic as a structural description of what a valid proof is but I have never understood the reasons for using modal logic.

What’s an example typical of how modal logic is used?

Answer accepted (score 15)

What is modal logic?

Modal logic is an extension of classic propositional and predicate logic that allows the use of modal operators. In others words, modal logic is everything classic logic is + modal operators. Modal operators express modality, such as:

  • Necessity (denoted by □)
  • Possibility (denoted by ◇)

The above possibilities are the only operators used in modal logic in the narrow sense. However, the term modal logic is often used to include other extensions, for instance temporal logic, that allows for the expression of past or future truths.

Why is modal logic useful?

Classic logic is great for mathematics, but for the analysis of daily language and arguments, it lacks certain operators. There are many sentences that you can’t express in classic logic that can be expressed in modal logic. Example: “I may get burned if I lie in the sun for too long”. In classic logic, you can say: “I get burned if I lie in the sun for too long”, but you can’t express the possibility of getting burned. In classic logic, it’s either true or false. In modal logic, you can also express the possibility or impossibility of a proposition being true or false.

Further reading:

Answer 2 (score 9)

Extending previous answers by ChaosAndOrder and Dennis…

You seem to appreciate why “pure” logic (I take it that you mean classical first order logic) is useful in the context of mathematical logic, but you don’t see the point in formalizing other modal notions in ordinary language. While presenting you the many applications of modal logic might convince you, it may be easier to indicate how modal logic was important in the development of the very field of mathematical logic that you seem to appreciate.

The formalization of modal logic has been a breakthrough in the development of model theory, one of the four branches of mathematical logic (the other being set theory, recursion theory and proof theory). See Kripke semantics for more details.

Modal logic has found some very natural applications in metalogic, such as provability logic (where □ means ‘it is provable that’). A milestone in the analysis of provability is Solovay’s arithmetical completeness theorem published in 1976.

Another interesting (but very advanced) example is the use of modal logic to study the relations between forcing extensions in set theory. See Hamkins & Loewe, The Modal Logic of Forcing, 2005:

What are the most general principles in set theory relating forceability and truth? As with Solovay’s celebrated analysis of provability, both this question and its answer are naturally formulated with modal logic. We aim to do for forceability what Solovay did for provability. A set theoretical assertion psi is forceable or possible, if psi holds in some forcing extension, and necessary, if psi holds in all forcing extensions. In this forcing interpretation of modal logic, we establish that if ZFC is consistent, then the ZFC-provable principles of forcing are exactly those in the modal theory known as S4.2.

Answer 3 (score 7)

So, I will tailor my response to your comment to ChaosAndOrder.

The reason we want to utilize modal logic is to precisify ordinary language. Ordinary language is notoriously ambiguous and the analysis of ordinary language modal operators is fraught with difficulty.

By regimenting our discourse into formal (quantified) modal logic we can eliminate some of these ambiguities. We can distinguish between modality de dicto (applying to a whole proposition) and modality de re (applying to predicating a property of an individual). Ordinary English is almost systematically ambiguous between the two.

In general, the reason to formalize any natural language statements is to achieve a greater precision in our discourse and to remove (as much as is possible) indeterminacy and ambiguity.

As an aside, classical logic is not great for analysis of mathematical discourse. The model theory of mathematics bears a striking resemblance to possible worlds semantics for modal logic. While it is true that within a model mathematics behaves classically, when we assess claims like categoricity (all models are identical up to isomorphism) we are assessing claims that are quite plausibly modal.

Also, the characterization of validity in classical logic is often modal in some sense. How do we explain validity? It is impossible for the premises to be true and the conclusion false (or, if the premises are true the conclusion MUST be true as well). What this (arguably) shows us is that validity/logical consequence is a modal notion.

93: Was Aristotle an Empiricist? (score 17385 in )

Question

When I was taught about Aristotle and Plato, the picture I got was very much like this image from a Raphael fresco:

Plato gestures to the heavens and Aristotle gestures to the Earth.

Usually Plato is said to be pointing to the heavens, which represent abstract Forms, while Aristotle gestures to the Earth (or possibly breaking the fourth wall and indicating the viewer) to symbolize his commitment to Empiricism. Certainly in an epistemological landscape dominated by Pythagoras and Plato, Aristotle emerges as practically David Hume in relation.

But reading Aristotle directly and especially the book he is shown to be holding in the painting (Ethics), I find Aristotle’s epistemology to be far more balanced. While he certainly prefers inductive arguments when he can make them, he isn’t reluctant to argue deductively from a priori principles that we might not agree with. In fact another popular characterization of Aristotle is that his philosophy held back science by insisting on claims that were not evident in nature. Some of these claims (such as heavier objects falling faster than light ones) could easily have been rejected with very straightforward experiments.

So ought we to think of Aristotle as a man who was born two thousand years too early to be a part of the scientific revolution or as a man who wished to moderate Platonic dogma?

Answer accepted (score 7)

Empiricism and its opposite Rationalism are positions about the nature and origin of knowledge. Empiricists say that knowledge comes from experience. (Empeirea is the Greek word for experience.) Rationalism on the other hand says that we have at least some knowledge innately, i.e. prior to experience. In this sense Aristotle is definitely an empiricist. He says explicitly in a number of places “all knowledge begins with the senses.”

However, Aristotle isn’t exactly a proponent of natural science in the sense that we think of the natural sciences today because he hasn’t got the idea of an experiment in the sense of a repeatable, precisely measurable, controlled trial. Aristotle’s scientific method does depend upon observation, and sometimes his observations are quite keen. He is the first person we know of to run the experiment where you trace the development of a chicken embryo by cracking different eggs at different days from fertilization. Also he describes a large number of marine species and insects. However there are some egregious errors as well: Aristotle believed that women have fewer teeth than men, for instance.

So is Aristotle the first scientist? I think it depends what you mean by a “scientist”. If you mean somebody who has the idea of the experimental method and who performs rigorous experiments, then no he doesn’t. Although, by that criterion, I’m not sure Galileo or Francis Bacon would have been “scientists” either. On the other hand, it is clear that Aristotle is the first person to set about giving a systematic explanation of nature in terms of simple principles based upon observation. And that is an important intellectual achievement.

Answer 2 (score 5)

My familiarity with Aristotle is mostly secondary, in particular the Jewish philosophers of the middle ages who had to reckon with it as the prevailing “scientific” opinion of the day [and even then my familiarity is imperfect]. Nevertheless my impression is that whatever empirical tendencies he may have had must have remained largely in embryonic form. Off the top of my head perhaps this position is illustrated by his view that heavenly bodies traveled only in perfect circles, while this was known not to be the case even among later Greek thinkers.

And while it may not have been his intention, my impression is that his acceptance as an authority led to stagnation in areas of research which he had spoken on exasperating his on non-empirically sound conclusions.

I think the following quote kind of sums up Aristotle’s influence on science, “I said this before, but I want to emphasize it. In spite of the fact that the details of Aristotle’s logical theory no longer have value for us, the fundamental value of his contributions must never be forgotten: He created a science of logic where before there was nothing.” http://ocw.mit.edu/courses/linguistics-and-philosophy/24-241-logic-i-fall-2005/readings/chp01.pdf Nevertheless, I’m not so sure we can give him a lot of credit as an empiricist.

94: Do animals know that they are going to die? (score 17228 in 2013)

Question

This is maybe the wrong website for this type of question, but I’ll try it anyway.

I’ve somehow experienced this patterns mostly in cats (not scientifically proven, but pretty common also talking to other people):

  • sometimes cats or dogs feel pain about their sons/loved ones losses. They feel depressed and tend to sleep more after a beloved loss.

  • when the death is coming: they tend to isolate themselves and reach a quite place, like if they know they are going to die within an hour or so.

  • cats seems to know if something is wrong with another cat in the same house (one of my cat decided out of nowhere to never eat from the food that was sharing with the other cat and felt forced to go out and hunt birds instead; later that month we discovered that the other cat had a terrible contagious disease pretty similar to human HIV and died in a week; after that the other cat simply began to eat the cat food again).

So here’s my final question: Do animals know that they are going to grow old and die? Is there some philosopher who talked about the awareness of death in animals and the difference with us? How do they possibly live their life in case the answer is no?

Answer accepted (score 3)

Side note: This could be thought of a philosophy of mind question but as it reads, it seems more like an ethology question, or (animal) psychology question which may or may not be fit for CogSci. That is, it seems you are asking for scientific evidence/research that indicates animals display the same kind of behaviors humans do which indicate an understanding of death. As a philosophy question, it is even more challenging than doing a few research experiments, specifically because it’s unclear what it means “to have an understanding of something”, let alone an understanding that we are going to die one day.

E.G., Does knowing that when you (or a cat) touch an electric fence that you will get shocked equate with an “understanding of electric fences”? Sure, you understand that fencey = shocky, but nothing about electricity, thermal conduction, grounding, etc. But let’s say you believe that such a basic understanding counts as “understanding electric fences”. Does it then seem to you that a cat seeing many other cats grow up and die would then understand the concept of death? Does this cat really know that the cat’s heart ceased pumping blood to its body for one reason or another? That these final moments of other cats’ lives is not just a super long sleep from which it cannot be awoken?

You can see very quickly the difficulty here with your question, and the real philosophical problem behind it:

What does it mean to understand something?

Some notes I have on this subject to get you started, which I wrote down in my initial preparations to write a sentient computer program (that would ultimately be able to understand concepts):

In logic, the comprehension of an object is the totality of intentions, that is, attributes, characters, marks, properties, or qualities, that the object possesses, or else the totality of intentions that are pertinent to the context of a given discussion. This is the correct technical term for the whole collection of intentions of an object, but it is common in less technical usage to see ‘intention’ used for both the composite and the primitive ideas.

To understand something is to have conceptualized it to a given measure. The use of concepts is necessary to cognitive processes such as categorization, memory, decision making, learning and inference.

  • Concepts as mental representations, where concepts are entities that exist in the brain.
  • Concepts as abilities, where concepts are abilities peculiar to cognitive agents.
  • Concepts as abstract objects, where objects are the constituents of propositions that mediate between thought, language, and referents.

–Mostly from wikipedia and other internet sources

Personal addendum: As a physicalist, I believe that “what it means to understand something (as a human)” can be so precisely described that it could be quantified (written in a mathematical formula or computer program), but we are not definitively there yet (at any rate, I don’t have the answer yet, but maybe someone else does).

Answer 2 (score 3)

Side note: This could be thought of a philosophy of mind question but as it reads, it seems more like an ethology question, or (animal) psychology question which may or may not be fit for CogSci. That is, it seems you are asking for scientific evidence/research that indicates animals display the same kind of behaviors humans do which indicate an understanding of death. As a philosophy question, it is even more challenging than doing a few research experiments, specifically because it’s unclear what it means “to have an understanding of something”, let alone an understanding that we are going to die one day.

E.G., Does knowing that when you (or a cat) touch an electric fence that you will get shocked equate with an “understanding of electric fences”? Sure, you understand that fencey = shocky, but nothing about electricity, thermal conduction, grounding, etc. But let’s say you believe that such a basic understanding counts as “understanding electric fences”. Does it then seem to you that a cat seeing many other cats grow up and die would then understand the concept of death? Does this cat really know that the cat’s heart ceased pumping blood to its body for one reason or another? That these final moments of other cats’ lives is not just a super long sleep from which it cannot be awoken?

You can see very quickly the difficulty here with your question, and the real philosophical problem behind it:

What does it mean to understand something?

Some notes I have on this subject to get you started, which I wrote down in my initial preparations to write a sentient computer program (that would ultimately be able to understand concepts):

In logic, the comprehension of an object is the totality of intentions, that is, attributes, characters, marks, properties, or qualities, that the object possesses, or else the totality of intentions that are pertinent to the context of a given discussion. This is the correct technical term for the whole collection of intentions of an object, but it is common in less technical usage to see ‘intention’ used for both the composite and the primitive ideas.

To understand something is to have conceptualized it to a given measure. The use of concepts is necessary to cognitive processes such as categorization, memory, decision making, learning and inference.

  • Concepts as mental representations, where concepts are entities that exist in the brain.
  • Concepts as abilities, where concepts are abilities peculiar to cognitive agents.
  • Concepts as abstract objects, where objects are the constituents of propositions that mediate between thought, language, and referents.

–Mostly from wikipedia and other internet sources

Personal addendum: As a physicalist, I believe that “what it means to understand something (as a human)” can be so precisely described that it could be quantified (written in a mathematical formula or computer program), but we are not definitively there yet (at any rate, I don’t have the answer yet, but maybe someone else does).

Answer 3 (score 0)

I believe that animals like us most certainly know they are going to die , from birth the instinctively know they have to outsmart predators , if they did not know that they can die why run from a lion , or a bare ,

95: Does the impossibility of an infinite regress prove God exists? (score 17016 in 2017)

Question

I’m strictly discussing one aspect of God: God as the First Cause. I am excluding all other qualities of God defined by any religion or belief system – including the notion of God as a sentient being. For the scope of this question God could be nothing more than an inanimate construct that is the source of all things. My question rephrased: “Does the impossibility of an infinite regress prove the universe has an ultimate source?”

Thomas Aquinas decided the answer to this question was Yes ( as can be seen is his Argument of the First Cause ). Yet he makes the faulty assumption that this being must be God as defined by religion. Notice how my question differs.

There have been many similar arguments throughout history such as The Cosmological Argument and Kalam’s Argument. My question, however has nothing to do with the identity of this First Cause. For that, see this question. I am only asking if we can logically determine if there is one, or not.

For my argument to be successful an Infinite Regress must be impossible. If anyone has a valid argument against this, please share. My argument supporting the impossibility of an infinite regress is as follows:

An infinite regress proposes an explanation, but the mechanism proposed stands just as much in need of explanation as the original fact to be explained. It is literally an infinite series of propositions where each proposition relies on the previous proposition. It is equivalent to saying:

“Each and every single human being was created by a human being before them”

According to this logic there can be no “first” human or even an origin for the species. Is this not a logical fallacy that is impossible? If this is, in fact, impossible as I suppose then the law of causality and the causal chain itself breaks down at a type of singularity that must be outside of it. With what we know from science and The Big Bang this would possibly be the influence that caused the Big Bang. This is in very much agreement with science because we now believe time itself started with the Big Bang, and causality depends on time existing. The only way to arrive at a timeless state is to go “before” the Big Bang.

Special pleading is an argument with an unjustified exception. Given the above, is this exception justified? Does the very fact that an infinite regress is impossible justify this exception?

Answer accepted (score 15)

It’s no solution to postulate a primordial source as a remedy against infinite regress. The concept of a primordial source prompts at once the question for its cause. To say it is “causa sui” - the answer of Christian philosophy - does not answer the question but rejects it.

My conclusion: We must not overestimate the power of pure reasoning. Instead, we must restrict ourself to the insight that today certain cosmological questions have no answer.

Answer 2 (score 15)

It’s no solution to postulate a primordial source as a remedy against infinite regress. The concept of a primordial source prompts at once the question for its cause. To say it is “causa sui” - the answer of Christian philosophy - does not answer the question but rejects it.

My conclusion: We must not overestimate the power of pure reasoning. Instead, we must restrict ourself to the insight that today certain cosmological questions have no answer.

Answer 3 (score 13)

This is very similar to Thomas Aquinas’ second ‘way’. Aquinas was a 13th century catholic philosopher and wrote Quinque viæ, five ways to God. The second is The Argument of the First Cause.

The five ways are controversial and very debatable. Richard Dawkins famously criticised them in The God Delusion. He claims it is a form of special pleading to say that everything except god must have a cause. This is a fallacious argument where something is an exception to a general rule, while the exception isn’t justified. In your case, you don’t justify why god can be an exception to your first premise that everything is caused.

Also, you don’t justify your second premise, that an infinite regress is impossible. It doesn’t seem to be very compatible with the Big Bang, however, it isn’t impossible once you refute that theory. For more info, see eternal return.

96: Is it immoral to download music illegally? (score 16699 in 2017)

Question

I should first point out that the title is more to capture a common occurrence of the broader idea I want to address in this question. It is also somewhat incorrect in that—at least in the US—I’m not sure it’s actually illegal to download music without paying (per se), but rather to share it. But the law here is irrelevant; this question is about the moral status of illegal downloading, whether or not it is or should be illegal.

My question, stated in a relatively broad, almost all-encompassing manner:

Is it immoral to acquire goods or services which are generally intangible1 for free when the original owners of such goods or services would have otherwise profited with such an exchange?
  • On one hand you are depriving the original owner of money they could have potentially made
  • On the other hand you are not actually taking anything physical from their possession.

1By intangible here I mean simply something that is incapable of being perceived by the sense of touch, as incorporeal or immaterial things. A digital version of a song, for example, is tangible in the sense that it is a real, existing product that has value, but intangible in that it occupies essentially no physical space (other than a handful of electrons in the capacitors of a memory module in a computer, etc.).

I think it’s easiest to demonstrate my reasoning so far through thought experiments:

Example 1:

If I walk into a movie theater (having paid), and it just so turns out that I have extremely good photographic memory such that I can rewatch the movie I saw in my head with perfect precision, does that means I am immoral for rewatching it over and over in my head without paying the owner each time? I don’t think anyone would seriously say that I am acting immorally in this case. But I don’t see this as qualitatively different than having taken a video camera into the theater and then rewatching the movie at home as many times as I please without re-paying the owner. Are people with perfect memory not allowed to go to the movies? You may laugh, but this is becoming a reality sooner than you think with technologies like Google Glass recording every moment of our lives.

Further, the same logic that applies to the above example also applies to many other cases; for example, if I hear a song on the radio and I just happen to be an acoustic genius who—after hearing a song once—possesses the ability to replay a song in my head with extraordinary precision after I’ve heard it once (or I was simply able to play it whenever I please using my own musical instruments / music mixing software at home). Normally, the cost of a song on the radio is 1) you have to listen to advertisements and 2) you have to wait to hear it again. But in my special case I could eliminate all those costs after hearing the song once. Am I immoral for not subjecting myself to the advertising of the radio station (and therefore reducing the money the radio station makes)?

These examples becomes even more interesting when we consider the inevitable eventuality that humans will integrate technology like cameras and hard drives into our bodies and actually possess perfect memory and recording capabilities as part of our being.

Example 2:

I am a master tailor with near God-like knitting skills. I see a person walking down the street with a beautiful purple scarf. I whip out my knitting tools and fashion myself the exact same scarf in 30 seconds, directly copying every aspect of its style and design. Am I acting immorally by not paying the original owner of the scarf for using his or her exact ideas in creating my own scarf?

I don’t see this example as far off from the idea of copying music. With tools which are readily available to us, we are all “Gods” when it comes to copying files. Ctrl+C, Ctrl+V; it’s that easy — just as easy as it was for the master tailor. Should I pay the owner of a song each time I copy the song on my computer, say if I want it in my iTunes folder but also in my music folder? That doesn’t seem very reasonable. So it seems that the action of copying something itself does not seem to be the problem. It seems to be a problem when the action of copying could result in a loss for someone else. However, if we were required to pay a fee for each copy of a song on our computer, each illicit copy action would result in a loss for someone. However, it’s not quite a “loss”, is it? The owner is not losing anything. They are simply not gaining some money they could have earned. In the case of the scarf, if I was not a master tailor, I might’ve otherwise bought the scarf, but you can’t honestly say it’s immoral for me to be a good knitter, can you? Stated another way:

Is aiding in the loss of a potential sale a moral wrong?

Clearly, what someone does not earn (a “non-gain”) is not the same as losing money, because your current wealth is not affected. Non-gain’s affect only potential wealth. Is it immoral to negatively affect someone’s potential wealth? I negatively affected the potential wealth of cigarette companies by convincing my friend to quit smoking. Was that immoral? It seems ridiculous to think so. But are there cases where it is immoral to negatively affect someone’s potential wealth? You tell me.


Note:

I want to try to avoid people’s subjective opinions on whether stealing non-tangible goods is immoral or not (i.e. in one persons particular opinion rather), and focus on whether it would be considered immoral in the moral community of today (globally, or “developed nations” if that suits you better). Also, note that this question is specifically designed to avoid references to statute; it is not asking whether downloading music is stealing but rather it is morally justifiable. Lastly, I’ve actually not read any philosophers who have written on this subject at any length, so links to articles would be useful.

Answer accepted (score 29)

I do not think that there is a single answer to the question of “is it immoral to negatively affect someone’s potential wealth” in the context of the moral community of today. For example, if I have a great and inexpensive widget for sale, and you have a lousy expensive widget for sale, my advertising of my widget is going to negatively affect your potential wealth. This seems to be viewed as not only perfectly okay but even desirable in market economies (which have particularly nice properties when one can assume consumers have near-perfect knowledge). Or if, for example, I am particularly annoyed with the customer service on United and encourage a friend to fly on Delta instead, I would not expect that friend to criticize me for negatively impacting United. Likewise, if I discouraged someone from taking a bus tour of Washington D.C., and encouraged them to walk instead because it would be a better experience, they’d probably appreciate it (if it was good advice). So it seems that in the context of market-based competition and engaging in exchange of goods and services it is absolutely okay to aid in the loss of a potential sale.

(As an aside, companies and individuals do often try to use copyright to head off these sorts of losses; if an embarrassing document comes to light, for example, it’s common that the entity will at least try to assert its copyright to avoid having the negative information disseminated.)

If we instead restrict ourselves to the main question–is it immoral to acquire intangible goods for free–then I think there is no single answer because there is not a single community. I’ve heard the following argument in various guises, mostly from thoughtful but non-affluent people, specifically with regard to music:

  1. I love such-and-so music.
  2. I cannot afford to buy more than this much of it, and I do buy this much.*
  3. Downloading the rest for free benefits me.
  4. Downloading the rest for free only formally, not actually, reduces the artists’ income, as I cannot afford more. There was no more potential for sales, so there is no actual harm.
  5. Therefore, this activity is morally permissible.

*The “the artists who created this get almost none of the money” argument often modulates this claim–that is, they’re happy to buy things that support the artists, but not necessarily to download music from iTunes where an overwhelming majority of the money goes to people other than the artists.

I’ve also heard the following argument in various guises, mostly from affluent people who rely on the current system for their affluence:

  1. Music is the work of artists (and producers and so on); that is the output of their labor.
  2. Music has a tangible value (i.e. people are willing to exchange money in order to be able to listen to it).
  3. Taking items of tangible value without the consent of its creators robs the creators of their livelihood.
  4. Therefore, this activity is both immoral and counterproductive.

From what I have seen, there is disagreement because the premises differ (both based on self-interest): the latter group takes the value for granted and reasons from there; the former takes the creation of the work for granted and reasons from there.

Of course, neither is true in some deep sense. If we could monetize and restrict the supply of oxygen, it would become extremely valuable, but that does not mean it would be beneficial; one should not simply take for granted the value of a song under the current scheme where distribution is restricted. Likewise, even for music, but especially for expensive propositions like software creation and movies laden with special effects and stars, it is clear that models that cannot fund the creators will be destructive, and therefore that being a freeloader is aiding oneself at the expense of others (which is pretty widely considered immoral).

I have not seen any philosophically sophisticated treatment of the various considerations. Lawrence Lessig has written some of the more carefully-reasoned material taking the side that copyright as used now is having a sizable negative impact, but he is a professor of law, not a philosopher. I am unaware of an equally eloquent proponent of an opposing stance.

Answer 2 (score 15)

Over several decades, I’ve paid the music industry many times more than the average consumer, but in recent years I’ve almost exclusively obtained my recordings from “illegal” use of P2P and torrents on the internet.

I don’t see my current behaviour as immoral, nor do I think the fact that I paid a lot in the past, or that I’ve had no income (and dwindling savings) recently, have any real bearing on the moral question.

Most professional musicians don’t earn huge sums, but what they do earn comes mainly from live performances. When I go to a gig, I’m grateful if I know one of the band and can blag a free ticket - but I don’t expect this; I respect a person’s right to be paid if they’re working, while I’m just there to enjoy myself.

For a top-notch album, there are always enough people in the world willing to pay, which will cover the (relatively) low costs of recording and mastering. Distribution over the net is bordering on free anyway, and if people still want the physical medium they’re obviously going to have to pay for that.

The people who lose out from music piracy are primarily those in the music distribution business, not the musicians themselves. I’d be happy to see their entire business disappear, since I think they’re at best dinosaurs, and at worst leeches.

Returning to the moral issue, just because a relatively small number of top entertainers (including some musicians) do in fact become staggeringly wealthy, doesn’t mean an aspiring musician should feel he’s being “robbed” if he doesn’t receive maximum income all the way through his journey to the top (which in most cases he’ll never reach). Specifically, I see no moral justification for musicians being paid again and again for repeat sales of a recording which - once completed - requires no further effort on their part.

Answer 3 (score 9)

Is it immoral to negatively affect someone’s potential wealth?

This is actually a very good question if we replace “wealth” with “income”.

The the answer goes like this:

  1. In a free market economy, high profits (and, hence, income) can be made only by saitisfying urgent demand for things that are very scarce. For example, while water is essential to sustain life, its price is usually quite low (at least outside deserts). In certain regions water is actually a free good. OTOH, diamonds and gold, while per se useless or at least not necessary to sustain life, command high prices and those who dig and sell them make high profits.
  2. Therefore, one way to possibly “negatively affect” high profits in a free market economy would be to persuade people not to demand that what is scarce. As long as this is done without force or threadening of force, there is nothing to object to that.
  3. Another way would be to actually reduce the scarcity of the item in question. For example, in a town where hunger reigns, one that sells grain will make high profits. These profits will decline drastically as soon as fresh grain supplies are brought in from outside and sold.

But (3) is actually the definition of increasing wealth! By producing something people need and want, you create wealth and, inevitably, reduce future profits of all producers of comparable items.

Hence, the answer to your question is a resounding “No, it is not immoral.”

If this were so, then the following activities would be immoral: quit smoking (reduce income of tobacco industry); build a house next to another (as far as free standing houses with better scenic view command higher prices); bake your own bread (think of the poor baker, dude!); inventing, building, selling and using automobiles/computers/washing machines; not using automobiles/computers/washing machines; … you get the picture.

It is a certain caste of rent seekers that want to make us believe that “their” future profits actually are already their property right now, so that taking them away is “stealing”. They have a natural alley, that helps them to pursue their special interests using force - the state.

97: Is “time” an abstract mental construct or does it exist independent of human consciousness? (score 16561 in 2015)

Question

When I consider my own existence with respect to time I can imagine three possibilities:

  1. Time extends infinitely into the past. In this case, how can the present, with me in it, exist, since there would be an infinite period of time that existed before the present. That seems illogical.

  2. Time had a starting point from which everything in our universe evolved to the present (t=1). That possibility would require something (God) to start the clock ticking (t=0). But in this case we’re back to square 1 with the problem of an infinite past. So the only possibility that makes sense to me is a third possibility

  3. Where time is some sort of abstraction in human consciousness that we need in order to relate cause and effect.

Answer accepted (score 5)

A popular saying is: “Time doesn’t exist, only clocks exist.”

Time, like all units of measurement, are abstractions.

In physics, the purest way of comparison is the direct one: How long is this thing compared to that thing. It yields a fraction of some sort, which most importantly is invariant.

This fraction has no units, is directly bound to the original question and never varies, no matter our frame of reference or unit of measurement.

The thing is, if we can’t measure one of the things before tomorrow, and we can’t measure the other thing after today. That is when we introduce units of measurements, because units of measurement co-vary.

When we change our frame of reference, say by pretending everything is twice as big (effectively cutting all our reference units in half), all off our measurements go up by a factor of two.

Therefore, comparing the measurements to one another still yields an invariant fraction!

Time is measured in seconds, which is defined as some high number of periods in microwave radiation emitted by Cesium atoms cooled to some low temperature under some special conditions, in an atomic clock.

But atomic clocks are subject to time dilation courtesy special and general relativity.

If you put an atomic clock on a spacecraft (which has been done, by the way) and send it into orbit, the lower gravity will make the space clock disagree with an identically calibrated twin left on earth.

So when we say “it takes N seconds” we really mean “if I put an atomic clock next to this thing, the dial on the atomic clock would show the number N when this process has reached the end state.”

Humans feel time passing because of some physical processes in our brains behave like (very inaccurate, compared to atomic ones) clocks.

But what does time then abstract? Entropy.

The universe is continually descending into informational chaos, usually in the form of heat. Entropy starts in the laws of quantum mechanics where a certain ubiquitous interaction causes loss of information; this can be proven to be an instance of Liouville’s Theorem.

Some cosmologists and epistemological mathematicians currently believe that the final formulation of quantum mechanics will not include “time” as a parameter anywhere, and that time will be a derivable quantity.

Answer 2 (score 4)

Makes sense. (2-3) are popular ways to go. I’d like to address your worry in (1).

Time extends infinitely into the past. In this case I don’t understand how the present, with me in it, could exist, since there would be an infinite period of time that existed before the present.

Consider the domain of nonpositive integers (Z = {0, -1, -2, …}). Interpret 0 as ‘now’, and precedence as ‘<’ on Z. If we use Z to model time, we can satisfy your hypothesis that:

  1. Time extends infinitely into the past: ¬∃i ∈ Z ( ∀j∈ Z ( i ≤ j ) ).

Now, while this means that an infinite period of time precedes the present moment, we still have it that:

  1. The present exists: ∃i ∈ Z: i = 0.

Usually rational or real numbers are used to model time, but this model is sufficient for our purposes.

Answer 3 (score 3)

  1. Definitely.

I’d liken it to a crystallization process. Or maybe even condensation.

Time does not exist. Only change exists. We can observe change by comparing the now to the what we remember.

Relativistic physics talks about “time” speeding up or slowing down depending on how fast you travel. All that changes is the speed of change of the object that travels faster or slower.

If you take an analog clock for example. If you don’t watch it for “some time” it will have made several 24h rotations. If you don’t know if it’s day or night outside you might not even know what “time” it is or if the time is correct. It’s just some made up instrument to give us something to hold onto in the ocean of change.

98: Why, according to Nietzsche, is becoming a monster by fighting to overthrow monsters a bad thing? (score 16526 in 2016)

Question

Nietzsche states:

Whoever fights monsters should see to it that in the process he does not become a monster. And if you gaze long enough into an abyss, the abyss will gaze back into you.

Source: Beyond Good and Evil

In fiction (and in real life at times) people embrace the inner monsters (sometimes literal, sometimes figurative) to gain the power they need to defeat their foes, get to the next level, or grit through something they’d rather not do. If this is something that seems to work, why are we specifically warned against it?

In any war, in any problematic situation where horrible things are happening, it can be impossible (or virtually so) not to use the selfsame tactics of those that oppose you. The saying goes “ALL’S fair in love and war” and even if you’ll pay for doing it in the long run by being seen as a monster, by doing these horrible things (or at times even MORE horrible than those that oppose you), you’ll have the satisfaction that you won, even though at day’s end it was a morally a pyrrhic victory. But if you accomplish what you set out to do, sometimes it’s worth it. But such things can’t help but drag you through the soil of morality, dirtying you in the process.

Answer accepted (score 1)

Surely it’s about the genealogy of morals. e.g. when you are hurt, not to take on a moral system which devalues whoever has that power over you. Or rather, he’s cautioning against it.

i.e the first use of ‘monster’ refers to the powerful or noble, and the second the christian (etc.).

Whether or not this theory of ressetiment even can be thought without ressentiment. Or even if you can escape the double bind of only ever valuing what seems good for you.

Answer 2 (score 1)

Surely it’s about the genealogy of morals. e.g. when you are hurt, not to take on a moral system which devalues whoever has that power over you. Or rather, he’s cautioning against it.

i.e the first use of ‘monster’ refers to the powerful or noble, and the second the christian (etc.).

Whether or not this theory of ressetiment even can be thought without ressentiment. Or even if you can escape the double bind of only ever valuing what seems good for you.

99: What did Hegel mean by ‘world spirit’? (score 16468 in 2016)

Question

I understand that Hegel intends to justify the history of mankind by stating that every town and society that has existed has been a test indirectly arranged by the ‘world spirit’ in order to eventually create a town in which people would live ethically, and that this process has been done in order for the ‘world spirit’ to become aware of its freedom. But what exactly is this ‘world spirit’?

In the last quiz of a course I’m taking (I´ve taken this year a philosophy course at college), I wrote that the ‘world spirit’ was some kind of parasitic entity which inhabits every human being that has existed and is going to exist. Does this seem right?

Answer accepted (score 7)

I wouldn’t go so far as to say that wikipedia can answer the question completely, but it’s not a bad start. The meaning of Spirit (Geist) is complicated and a matter of contested interpretation, but I will just explain to some extent what happens with spirit in Phenomenology of Spirit (or Mind), hereafter PhG.

To summarize very quickly, the PhG is a story about knowledge and consciousnesses. The story begins in the preface with naive realism that asserts “what I see in front of me is real” under the heading “SENSE CERTAINTY.” The story then advances through problems with different accounts of knowledge, i.e. how we know things as “CONSCIOUSNESS” of objects. It then turns to topic of the self that is able to know under “SELF-CONSCIOUSNESS.” From there, it discovers “REASON” which has to do with the process of thinking itself. Then thinking itself rotates back into the world as the activity of thinkers under “SPIRIT” which is the self that understands and realizes it is spirit and takes specific form as “RELIGION” and finally “ABSOLUTE KNOWING.”

Thus, we could say at one level Spirit is a section in the Phenomenology. But the real answer is more complicated. This is because (giving away the ending) it turns out that the agent of knowing all along has been Spirit – even before it knew it was conscious, self-conscious, reason or spirit. Largely, this is a journey of increasing self-awareness. Spirit is thus the active element in consciousness.

In the section called “RELIGION”, Hegel will argue that the death and resurrection of Jesus and the reception of the Holy Spirit is the recognition in history that we are the divine Spirit that is consciousness engaged in the task of knowing itself. For Hegel, Jesus is a man who realizes he is God, who dies, and then is “resurrected” as Spirit’s self-consciousness in all men that they are Spirit – meaning they are the conscious part of the universe that makes what surrounds us a “universe” (as a concept for us) and gives things meaning).

For Hegel, this turns out to be necessitated. You can understand this necessity either as a contingent necessity built into the nature of consciousness or as an absolute necessity built into the inevitability of everything that happens in the world. I would tend towards the latter as an interpretation of Hegel’s own view.

It’s mentioned in one of the answers above, but Spirit is ultimately panentheist or pantheist insofar as it turns out that God is Spirit and we are Spirit and all we do is Spirit. But this is because the objects, etc., we know and perceive are already being imbued with Spirit through our acting and perceiving.

Im not sure if that’s helpful for you, but it’s a brief sketch of what happens in PhG as it relates to Spirit.

What about “world Spirit”? Well, it turns out World Spirit is the recognition that consciousness is ultimately non-individual. The cultural backgrounds, etc., in which we think make it so that the agency of understanding is not localized but rather occurs within societies and cultures as their agency. For Hegel, this also includes their destiny. World Spirit is the necessity of the unity of rational consciousness that Hegel believed happens inevitably (whether this is contingent or necessary inevitability is a matter of debate).

Answer 2 (score 2)

First encountered on Philosophy Forums, the following discusses Hegel’s ‘World Spirit’ (ie: ‘Weitgeist’) and summarises Hegel, but is NOT my own work. Since this summary aided me, but suffers from many grammatical errors, I decided to emend it, and not only copied and pasted it; so please observe any differences. Please notify me of any errors and suggest improvements.

Author: ‘Tobias’    Posted Aug 27, 2005


I made a short summary of core themes in Hegel’s philosophy that may help people wishing to get to know him better. It is no professional summary though, not the be-all-and-end-all of Hegel. I would like to know what you think; so please point out mistakes or things you think I have missed. Regards in advance, Tobi

  1. Starting point of Hegel’s thought

German philosopher Hegel starts as a follower of Kant, but quickly becomes involved in a movement that tries to surpass Kant, without losing the improvements made by Kant and his Copernican turn.

The main reason that Hegel feels Kant’s philosophy cannot be the final articulation, is that according to Hegel, Kant divides our relation to the world in dualisms, such as into: sensibility and intelligiblity, noumena and phenomena, into intuition and concept. All these dualisms cannot be related to each other, and so (according to Hegel) makes our knowledge of the world always incomplete and detaches our knowledge from reality. Reality is the noumenon (Ding an Sich) which we can never know. Kant seems to pull apart what belongs together for Hegel: thinking and existing. Kant’s divisions have grave consequences for metaphysics, because they doom our attempts to think the great metaphysical question: the self, being, God as something real, which (with Kant) instead become simple regulatory principles which have no bearing on reality. We cannot think Kant’s philosophy above and that which is given in experience, without losing ourselves in antinomies, unsolvable paradoxes.

Hegel instead feels impossible not to accept a unity in which all these seemingly opposite notions have their place. If we find an opposition between concepts, than we must be able to conceive them as opposition so in relation with each other. Even the classic opposition A and not-A belong together on a plane of A-ness where they are opposite. So for each opposite we find a higher identity in which they belong. If we accept the consequence of this thought, than there must be a totality, all reality in which there oppositions have their place. Hegel called that totality the ‘Absolute’.

  1. The Absolute

The Absolute is one of the most provocative ideas of Hegel, but I feel the idea is rather simple: It is in principle the relation between two different things that ties these things together. If I lay my hand on the table, I have the absolute. Why? Because though the table and my hand are different objects, it is my hand which I lay on the table and I laid my hand for a reason; so my hand and table are connected as parts in that complex movement (of laying my hand on the table). My hand and the table are no strangers toward each other; in fact, a table is a good place to rest my hand, and laying my hand on the table shows well my immediate relation to my surroundings.

More generally speaking, the world is no stranger to me. I live in it; I do things with it; I presume the world has some order which I can know. Even though I am often surprised and bewildered by the world, I still accept the surprise as part of my world, not as some alien thing. In fact, is not lack of surprise only possible when my idea is that I know the world intimately? After all, I can only be surprised if I expect something else.

So the absolute is that simple relation we have to the world; but is it really that simple? Well it is a little bit more complex, because our relation towards the world and the relations within the world itself, always change. 2.1. To exemplify the previous sentence physically, I might lay my hand on a table and assert that this is absolute: this assertion might be right. 2.2. As another example, suppose that I put a glass on the table (for instance). Then the relation is no less absolute. But my reasons for laying my hand on the table differ from my reasons for putting my glass on the table, because the first action exemplifies the topic sentence of this paragraph, whereas the second action is necessary for easy access to my drink.

I hope that this clarifies that the relation is always absolute, but is also always different; the relations within this totality change from minute to minute, from moment to moment.

This relation is not an immovable relation, but an always changing one. The absolute is always the same, but posits itself always as different. Always there are different opposites being connected; in fact the absolute can only exist because it unifies differences and because there are always new differences evolving from the absolute. The absolute is no substance; it is a process, because no matter how these relations change and what the new states of affairs will be, the change is understandable. Some changes may seem irrational, but when cogitating them we always find a reason for what happened. Relations evolve in a reasonable process, which Hegel calls ‘Spirit’.

  1. Spirit

Spirit is the idea that things ‘logically’ follow from each other. We cannot think of something other than being in a development to something else. I examine 2.2 further: When I put my glass on the table, I do so with the idea that I will drink from it again, which means the glass will become empty at some point, and so which means that I will have to remove it from the table and put it in the dishwasher. My thinking thinks things in a certain procedure that is basically orderly, which procedure is common to the whole totality from which no part is exempt. We can understand this as follows:

We find Spirit most clearly in human intentional activity. Spirit is goal oriented and is the human thinking that ‘molds’ all development and gives human thinking that orderly label. How is Spirit in nature? We see nature as what differs from ourselves: that force all around us with floods, earthquakes and seemingly blind coincidence. Nature doesn’t seem rational; it just seems ‘the way things are’, being blind and unreasonable. Yet this opinion of nature is a mistake, thinks Hegel, because we discover laws in nature: regularities. Nature is not fundamentally other than spirit; it is the ‘other’ of spirit. Recognises Nature as Spirit’s other, mirror image, Spirit recognises that even nature is only thought of as ‘Other’ by Spirit. It is only Other in Spirit’s own understanding. The same goes for religion and God. God is already a very advanced form of, and is a definition more or less of, thinking the totality; yet God is still a symbol. God is still not an inwardly understood concept, but an outward symbolising of it. Even the unlimited God, has its limit because it is a symbol and is something that is not understood properly. Yet when Spirit realises that this god is nothing but a symbol of this totality, Spirit will recognise God as Spirit’s own creation as if God were Spirit’s projection on a wall. If Spirit reflects on itself like that and understands that there is literally nothing beyond, Spirit recognises that it alone is all reality and that all reality is reasonable.

In the previous paragraph, I projected quite some human characteristics on Spirit: Spirit that reflects on itself, that recognises things, etc. For Hegel, this characterisation of Spirit is not only metaphorical; for Hegel Spirit is indeed a kind of rationality. This rationality shows itself in our human ratio; we as humans are not unconnected atoms, but instead are connected in practices and shared goals that have their own rationality, beyond what a single human understands. Spirit is that rationality that overarches all these practices in its turn, in that order that is the world itself. Hegel sometimes calls this the Absolute Idea, because Idea expresses this rational idea and expresses that it is not a substance, but a moving relation all encompassing relation, rather than a thing.

  1. Dialectic

Of course this is all well and good, but is there any proof for this overarching totalistic rationality and if so, how can we find that proof? Hegel tried to show that this Absolute Idea exists via a method he first called speculation and later dialectic. Hegel’s probably most influential work is supposed to be a ‘deduction’ of this Idea. In this book Hegel tries to show that we must accept a rational totality because all other viewpoints will find themselves in insurmountable contradictions. In fact Hegel shows us that we can never find rest in any kind of metaphysical fundament: not in the idea that what I immediately see before me is most true, because if I look a moment later I see something else; but again also not in the opposite view, that everything extant is just in the here-and-now. Why not? Because for these ideas to be real, they must have some concrete content. We cannot be happy with an essentialist account, e.g. that salt is salt because of its saltiness. We can discern lots of different things about salt, but one of them is not saltiness. Is salt then a combination of different qualities? No, because then all these qualities would be in thin air, with nothing to bind them, and so on and so on… We find ourselves in oppositions that constantly urge us to review our theories of reality, which make us doubt and find some new explanation. The new explanation is always a new standpoint, which we will find can also not hold, but through this constant review, we do learn something. We learn how we are related to reality: that it is basically our thinking that orders the world. We learn that being conscious is essentially being self-conscious and that if we get confronted by other self-consciousnesses, some order will evolve and shape our roles in that hatchling community. We learn the problem of freedom: that we can only be free within certain boundaries. We learn that that the world of laws is itself controlled by the real world of occurrences, which in turn had to be explained by the world of laws etc… All this ends with the recognition that what we relate to, is not something strange, but something linked with us: what we have called the Absolute or Spirit. Ending with nothing less than absolute knowledge, Spirit knows that it is all there is. That is the dialectic: Hegel’s conviction that all development proceeds by encountering opposition, which encounter then produces a new idea or new judgement, which in turn will be opposed and so on. Each new opposition steps towards a higher insight into the totality.

In the Logic (Hegel’s second influential work), he dialectically tries to extrapolate all the determinations of: thought, because reality depends on, at least does not differ from, thought; and of reality too. This work resembles Kant’s; Hegel tries to deduce the categories of thought as Kant before him did. But Hegel does so not self-reflectively as Kant tried, but dialectically, moving along with the flow of thinking which creates all oppositions. But since these oppositions are oppositions in a unity, a higher concept can always be found, after the logic is completed and our thought process is totally mapped. All other sciences can be worked out on the basis of this dialectical model. This task he tried to perform in the Encyclopedia, the Religionsphilosophie and the Outline for a Philosophy of Law.

To quickly recapitulate the dialectic: When posited as some final notion that explains a part of reality, every notion will be countered by its opposition, and this counter will force both oppositions to come together in some higher notion (that contains within the higher notion) the tensions of the first two oppositions. I exemplify with the following dialogue of two sentences:

  • I say, ‘we deal with each other on the basis of justice’.
  • ‘No’, you will immediately counter, ‘we deal with each other on the basis of power’.

Then the above tension will be overcome by the idea that we deal with each other on the basis of law, justice armed with power.

  1. Conclusion

Absolute, Spirit and Dialectic: I think these three notions are the main building blocks of Hegel’s philosophy. I also hope to have shown that Hegel with the conception of his philosophy is thoroughly indebted to Kant.

Books I found very helpful understanding Hegel were: R. Pippin Hegel Idealism, Cambridge 1999 (?)
H. Marcuse Hegels ontologie und die Theorie der Geschichtlichkeit,
W. Jaeschke Hegel Handbuch Metzler Verlag 2003.
(In Dutch) Hegel een Inleiding ed. Arie Leijen en Ad Verbrugge, Boon 2002

Answer 3 (score 1)

Carl G. Jung, a Kant follower, used ideas from the Germanic culture that, in turn, Herman Hesse used in “Steppenwolf”: there is no such thing as a ‘unified I’ or self. We are “many selves,” which Jung calls “complexes.” Then, a “false I” is the so-called “personality”; which in the bourgeois thinking that first Hegel, then Marx criticized is equated with “self,” but it is not.

I find Hegel’s idea of a self that is so to speak in flow fascinating. It is brought to these times from Heraclitus

100: Examples of universalizable maxims in Kantian ethics (score 16197 in 2019)

Question

I’m taking a two year course on philosophy and ethics. At the moment, we’re studying Kantian ethics. I understand Kant’s theory of “act by a maxim that you would want universalized” and the three steps to see whether a maxim can become a categorical imperative, such as universalization, seeing humans as an ends mean and kingdom of means.

I am currently stuck at figuring out what maxims could be universalized apart from not lying and ‘prisoners of war should not be killed’. What are other examples of maxims that could be universalized?

Answer accepted (score 2)

Here are two maxims:

  1. Do not steal

Because if everybody stole, the very practice of property rights would break down.

  1. Keep your promises

Because if everybody broke their promises, the very practice of making and taking promises would break down.

Answer 2 (score 0)

  1. Don’t (try to) win at chess (with a human opponent).

  2. Don’t (try to) commit suicide (for someone else).

  3. Don’t (try to) steal other people’s property (if they / we have rights to property).

etc. the CI makes sense, you just need a little nuance to get them.

There is the related question of whether we may deceive ourselves into not really following the CI, for repugnant examples (lying to stop the child dying). That would depend, at a minimum, upon us not being able to deceive ourselves at will, etc.. I’m unsure of the metaphysics of the argument, so whether self deception changes the CI. Seems safe to say, yes, but it would have to be for a moral reason.